Welcome to
Science a GoGo's
Discussion Forums
Please keep your postings on-topic or they will be moved to a galaxy far, far away.
Your use of this forum indicates your agreement to our terms of use.
So that we remain spam-free, please note that all posts by new users are moderated.


The Forums
General Science Talk        Not-Quite-Science        Climate Change Discussion        Physics Forum        Science Fiction

Who's Online Now
0 members (), 181 guests, and 2 robots.
Key: Admin, Global Mod, Mod
Latest Posts
Top Posters(30 Days)
Previous Thread
Next Thread
Print Thread
Page 1 of 10 1 2 3 9 10
#54098 07/04/15 07:55 AM
Joined: May 2011
Posts: 2,819
O
Orac Offline OP
Megastar
OP Offline
Megastar
O
Joined: May 2011
Posts: 2,819
Since we have an abundance of alternate theory experts lets try and see if we can get some real conversation going. I want you to read the article by Ethan Siegel

https://medium.com/starts-with-a-bang/as...le-5ef156bf048d

It actually harks back to a discussion I was having with Bill G/Bill S when they were trying to use a rubber sheet and ball weights as a simulation of gravity and get them to see the problem. Lets just say they didn't get it or the implications but lets try the alternative theory guys.

The key point is that any valid theory is going to have to explain

1.) How does gravity get out of a black hole.

Background: Gravity interacts with itself and everything attracts everything else. So if you create a black hole that stops even light leaving how is it gravity itself is getting out?


2.) Why is the speed of gravity restricted to the speed of light

Background: Gravity has been measured a number of different ways and all put it's speed at between 0.8c to 1.2c and that is consistent with the most likely that it is in fact c (https://en.wikipedia.org/wiki/Speed_of_gravity)


3.) What is opposing the black hole so that the black hole gravity does not go to infinity (What limits the collapse).

Background: There has to be something opposing gravity otherwise whatever your theory the universe is going to collapse or at the very least start shrinking into every black hole. Worse still try and create the universe if gravity is not opposed.

I don't want crazy other details of your alternative theory but just deal exclusively with the 3 points asked.

Last edited by Orac; 07/04/15 08:24 AM.

I believe in "Evil, Bad, Ungodly fantasy science and maths", so I am undoubtedly wrong to you.
.
Orac #54100 07/04/15 11:16 PM
Joined: Aug 2010
Posts: 3,570
B
Megastar
Offline
Megastar
B
Joined: Aug 2010
Posts: 3,570
I’ve not read the article yet, but here’s a quick shot at the questions.

“1.) How does gravity get out of a black hole?”

It doesn’t. Gravity is not a force, it is a feature of the geometry of spacetime. Spacetime is influenced by the presence of the mass of the black hole; not by anything that has to escape from it.

“2.) Why is the speed of gravity restricted to the speed of light?”

Gravity does not travel. What travels is information about the presence/nature of the mass in question. Exchange of information is limited to “c”.

“3.) What is opposing the black hole so that the black hole gravity does not go to infinity (What limits the collapse)?”

If the centre of a black hole is a singularity, this is defined as a point where spacetime curvature is infinite, so gravity is infinite. Obviously, spacetime could not become more curved, and the area of infinite curvature must be infinitesimally small, so it is self limiting.


There never was nothing.
Bill S. #54101 07/05/15 03:14 AM
Joined: May 2011
Posts: 2,819
O
Orac Offline OP
Megastar
OP Offline
Megastar
O
Joined: May 2011
Posts: 2,819
Remember: I am not giving an answer and there is no right/wrong from me. I am simply looking at the logic of your responses and ultimately whether I can believe your theory.

Originally Posted By: Bill S.
It doesn’t. Gravity is not a force, it is a feature of the geometry of spacetime. Spacetime is influenced by the presence of the mass of the black hole; not by anything that has to escape from it.

So how does it interact with matter and know when to deform ... divine perception(god tells it)?

Being facetious but it highlights the problem there has to be an interaction which you refer to as influence.

Go back to your rubber sheet and ball example the deformation is created by the mass. The mass is opposed by the rubber tension of the runner stretching and that gives you the proportionality that a heavy mass deforms more than a light mass.

Now go back to the gravity example there is proportionality. You may care to think about what are the other possibilities besides proportional and why is proportional selected?

So what you deem as an influence must (a) come out of the black hole and (b) create a proportional response to mass/distance which we write as a formula.

Originally Posted By: Bill S.
Gravity does not travel. What travels is information about the presence/nature of the mass in question. Exchange of information is limited to “c”.

If gravity does not travel how does it act at distance and how can we even do the experiments to measure its speed?

I sort of get the information bit as that is very QM but you lost me with how something that doesn't travel have a speed at all.

I am guessing you are going to equate "information" to energy along the lines of QM?

Originally Posted By: Bill S.
If the centre of a black hole is a singularity, this is defined as a point where spacetime curvature is infinite, so gravity is infinite. Obviously, spacetime could not become more curved, and the area of infinite curvature must be infinitesimally small, so it is self limiting.

Think more carefully about what you have said gravity is going to reach infinity smile

So we are back to our problem why doesn't the universe get instantly sucked back into the infinite gravity, what stops it?

I guess I could say the universe is a positive infinity and try and balance it but even then I am seeing an issue. Supposedly how many black holes are there in the universe?

To me the logic ends up at one extra big infinity is offset by lots of small infinities ... not sure I like this sort of maths smile

The fact your value can drop to infinity still leaves me questioning proportionality, why doesn't a bit of matter create gravity that instantly drops to infinity? Why does it require a lot of matter to produce the infinity?

I have a much deeper more technical problem with your answer but lets first clear the simple ones.

Last edited by Orac; 07/05/15 04:05 AM.

I believe in "Evil, Bad, Ungodly fantasy science and maths", so I am undoubtedly wrong to you.
Orac #54102 07/05/15 12:32 PM
Joined: Aug 2010
Posts: 3,570
B
Megastar
Offline
Megastar
B
Joined: Aug 2010
Posts: 3,570
I was trying to frame the argument in terms of what seems to be accepted scientific (hitch-hiker level) terminology, and at the same time stay off my usual “hobby-horse” about infinity.

Quote:
To me the logic ends up at one extra big infinity is offset by lots of small infinities ... not sure I like this sort of maths.


I’m glad you don’t like it. If it makes any sense at all, it is only in terms of mathematical infinities, which are simply bookkeeping conveniences.

Let’s try to dispense with infinities, at least for the time. A black hole exists in time; it has a beginning and, as far as we know, an end. It is finite. Nothing finite can become infinite, so the most we can say about the centre of a black hole is that its curvature is able to increase “infinitely”. This brings us back to the distinction between infinite and boundless.

In the interests of taking one step at a time, can we agree on what we mean when we talk of something becoming infinite? Would this be the same as what we mean if we say that something goes to infinity?

My response to those questions would be that the first is meaningless, and the second refers to boundlessness.


There never was nothing.
Bill S. #54107 07/06/15 07:38 AM
Joined: May 2011
Posts: 2,819
O
Orac Offline OP
Megastar
OP Offline
Megastar
O
Joined: May 2011
Posts: 2,819
Realistically I don't get hung up on infinity like you seem too, it is a description of a behaviour to me nothing more or less.

My issue is gravity increases to some incredible value and there are 2 obvious issues

1.) What controls the rate of drop off to whatever you want to call that bottom value ... infinity/bound?

This is the issue of proportionality ... here is a plot of Earths gravity well.


Now that is a steep drop why doesn't it keep plummetting to the infinity/bound condition?

GR has an explaination for the rate of dropoff and even why the value doesn't go to infinity/bounds and so any competing theory needs to do the same.


2.) What is the nature of the interaction between matter and gravity?

Ideally an alternative theory would be able to shed light on this although for its part GR does not describe what causes the interaction. GR merely places a curvature relationship in time on all space coordinates radiating out to infinity/universe bounds by some unknown process.

Last edited by Orac; 07/06/15 07:40 AM.

I believe in "Evil, Bad, Ungodly fantasy science and maths", so I am undoubtedly wrong to you.
Orac #54112 07/06/15 09:17 PM
Joined: Aug 2010
Posts: 3,570
B
Megastar
Offline
Megastar
B
Joined: Aug 2010
Posts: 3,570
Originally Posted By: Orac
Realistically I don't get hung up on infinity like you seem too, it is a description of a behaviour to me nothing more or less.


Pushed for time at the moment, but coming back. Thought I should say I have no objection to using "infinity" in this way; just as long as we all understand how we are using it.


There never was nothing.
Orac #54127 07/07/15 06:18 PM
Joined: Aug 2010
Posts: 3,570
B
Megastar
Offline
Megastar
B
Joined: Aug 2010
Posts: 3,570
Originally Posted By: Orac
1.) What controls the rate of drop off to whatever you want to call that bottom value?


Looking at your diagram (let’s call it Fig. 1 in case there are more) it is easy to imagine that something needs to act on the Earth to stop it falling further down that well, possibly to infinity. Remember, though, that this is just a small fraction of a 2D model of the real thing. To be a complete 2D illustration there would have to be an infinite number of gravity wells surrounding the Earth. Even without attempting the impossible, you could draw enough gravity wells around the Earth to get rid of that apparent, but totally wrong, view that something – gravity? – is pulling the Earth “downward” in Fig.1.

Gravity is, if it is to be thought of in terms of gravity wells, an infinite number of wells, radiating in 3D from the Earth.

There is no downward direction, except towards the Earth, at the centre of this infinite number of gravity wells.

Why would this be seen as a situation in which any force is trying to move the Earth in any direction from its apparently stable position at the centre of this setup?

If there is such a force, would it not be balanced by an equal force acting on the Earth from an infinite number of directions?


There never was nothing.
Orac #54129 07/07/15 08:31 PM
Joined: Aug 2010
Posts: 3,570
B
Megastar
Offline
Megastar
B
Joined: Aug 2010
Posts: 3,570
Originally Posted By: Orac
2.) What is the nature of the interaction between matter and gravity?

You might need to be more specific in what you are asking here. There could, for example, be a Q/A sequence that went something like this:

Q. What is the nature of the interaction between matter and gravity?
A. Mass distorts spacetime which results in what we interpret as gravity.

Q. How does mass achieve this?
A. Simply by being there.

Q. Why does this work?
A. “Why” questions take us outside science, and into philosophy.


There never was nothing.
Bill S. #54133 07/08/15 05:57 AM
Joined: May 2011
Posts: 2,819
O
Orac Offline OP
Megastar
OP Offline
Megastar
O
Joined: May 2011
Posts: 2,819
Originally Posted By: Bill S.
Why would this be seen as a situation in which any force is trying to move the Earth in any direction from its apparently stable position at the centre of this setup?

If there is such a force, would it not be balanced by an equal force acting on the Earth from an infinite number of directions?

Ummmm ... so again exactly what is curving and how are you measuring it?

You are going to have to do that one from the top for me, I am missing something


I believe in "Evil, Bad, Ungodly fantasy science and maths", so I am undoubtedly wrong to you.
Bill S. #54134 07/08/15 06:09 AM
Joined: May 2011
Posts: 2,819
O
Orac Offline OP
Megastar
OP Offline
Megastar
O
Joined: May 2011
Posts: 2,819
So lets extend your Q & A session

Orac) How does mass achieve this?
Bill.S) Simply by being there.
Orac) So you have a correlation but no causation?


Orac) Why does this work?
Bill.S) “Why” questions take us outside science, and into philosophy.
Orac) No Bill S if you don't have a "why" you can have a science law but not a science theory.

Big difference between a science law and a science theory.

https://en.wikipedia.org/wiki/Scientific_law
https://en.wikipedia.org/wiki/Scientific_theory

So is what you telling me is you have Bill's law of gravity rather than a theory?

I can't see how Bill's law of gravity is any different to Newton's law of gravity, explain differences please?

You may care to read some quotes of Newton written in 1713 and ponder modern education standards.

Originally Posted By: Newton
"That one body may act upon another at a distance through a vacuum without the mediation of anything else, by and through which their action and force may be conveyed from one another, is to me so great an absurdity that, I believe, no man who has in philosophic matters a competent faculty of thinking could ever fall into it."

Originally Posted By: Newton
"I have not yet been able to discover the cause of these properties of gravity from phenomena and I feign no hypotheses... It is enough that gravity does really exist and acts according to the laws I have explained, and that it abundantly serves to account for all the motions of celestial bodies."

Newton man of science knew the difference between a law and a theory and what he was required to do

You will hence forth be known as Bill "hypotheses non fingo" smile

Hmmm wonder how does GR get from a law to a theory? smile

You have it easy all the answers are probably written out on the internet all you have to do is know how to join them up.

First principles is fun isn't it Bill S keeps you on your toes now can you walk the walk laugh

Hint: Hmmmm lets think of a question you once answered, Bill S does a photon create a gravitional pull, does a photon have mass and do you consider it matter? Is this consistent with your laws and statements above?

Last edited by Orac; 07/08/15 09:13 AM.

I believe in "Evil, Bad, Ungodly fantasy science and maths", so I am undoubtedly wrong to you.
Orac #54139 07/08/15 09:14 PM
Joined: Aug 2010
Posts: 3,570
B
Megastar
Offline
Megastar
B
Joined: Aug 2010
Posts: 3,570
Not ignoring your latest questions/comments; I'm still a few steps further back.

Originally Posted By: Orac
If gravity does not travel how does it act at distance and how can we even do the experiments to measure its speed?


Wikipedia says: “…the speed of gravity is the speed at which changes in a gravitational field propagate. This is the speed at which a change in the distribution of energy and momentum of matter results in subsequent alteration, at a distance, of the gravitational field which it produces.”

“For static fields, such as …… the gravitational field connected to a massive object, the field extends to infinity, and does not propagate.”

Would it not follow from this that gravity is a static field, and that what is being measured is changes within a gravitational field, rather than any intrinsic motion of the field itself?


There never was nothing.
Bill S. #54142 07/09/15 02:43 AM
Joined: May 2011
Posts: 2,819
O
Orac Offline OP
Megastar
OP Offline
Megastar
O
Joined: May 2011
Posts: 2,819
Originally Posted By: Bill S.
“For static fields, such as …… the gravitational field connected to a massive object, the field extends to infinity, and does not propagate.”

Would it not follow from this that gravity is a static field, and that what is being measured is changes within a gravitational field, rather than any intrinsic motion of the field itself?

There is a problem we are going to confront and you haven't yet got to the point where we have introduced the problem.
So for a change I will sort of give you the problem we are about to hit and you decide what you want to discuss.

In the above you suddenly introduce the concept of a field which your idea and laws haven't even dealt with yet.
I will put that aside as I think you started it as a joke and now got buried in your own joke.

Fields were initially described in classical terms and the man that did the greatest contribution is James Clerk Maxwell. The part you need to read is the history of the equations he derived.

https://en.wikipedia.org/wiki/History_of_Maxwell's_equations


Again I will be brief and just jump thru it just to sequence things for you. The key point about the equations is they connect electricity, magnetism, and the speed of light.

The problem with the equations and what we call "Maxwell's equations" really are not exclusively Maxwell's at all and its covered in that history. What you need to do is add in the concept of lines of force and vector mathematics to Maxwell's work.

The evolution is well covered in
https://en.wikipedia.org/wiki/A_Dynamical_Theory_of_the_Electromagnetic_Field

It is actually discussed in that link that we are going to hit another complication. All that early work is classical and suddenly we are going to need to consider motion and especially speeds getting near the speed of light itself.

Quote:
Albert Einstein used Maxwell's equations as the starting point for his Special Theory of Relativity, presented in The Electrodynamics of Moving Bodies, a paper produced during his 1905 Annus Mirabilis.

What we are describing is the connections of how and why GR (via it's connection to SR) isn't a law but is indeed a theory and Einstein makes it clear he is seeking to do that by this statement .. sigh I am doing your homework again smile.

Originally Posted By: Einstein
"the same laws of electrodynamics and optics will be valid for all frames of reference for which the equations of mechanics hold good"

Remember the requirements of a new theory to replace an old theory or laws it must explain all that the old covers and he makes that claim.

Finally we can address your question above and the problem rolls around motion. So do you think you are going to be able to use static field ideas on universal bodies that may well be moving at speeds a good fraction the speed of light? What they sort of hide in the statement above is they expect large objects not to be moving very fast and they sort of wave there hands and say we can thus treat them as static ... that is classically ... it's a justification not a statement of truth. You run into this problem with black holes, a spinning black hole vs a static black hole are very different beasts.

So the statement may be valid in a context of use, where classical physics and static fields would provide a correct answer and you can justify ignoring Special Relativity.
So for a more precise personal view I would need to see the full context. If you are asking is that some sort of "absolute truth" then no Special Relativity is the correct answer in all situations Einstein told you that ... don't you believe him smile

Bonus hint: In the second link was a hint for your journey, let me isolate it "Maxwell's equations can also be derived by extending General Relativity into five physical dimensions."

Last edited by Orac; 07/09/15 04:15 AM.

I believe in "Evil, Bad, Ungodly fantasy science and maths", so I am undoubtedly wrong to you.
Orac #54143 07/09/15 04:46 AM
Joined: May 2011
Posts: 2,819
O
Orac Offline OP
Megastar
OP Offline
Megastar
O
Joined: May 2011
Posts: 2,819
If you were trying to be cute and got caught let me show you the problem

GR equates local spacetime curvature (expressed by the Einstein tensor) with the local energy and momentum within that spacetime (expressed by the stress–energy tensor).

You have got the first bit you are totally not getting the second bit and haven't introduced it.

The answer starts in the Maxwell link under the section Electromagnetic stress–energy tensor under a curved spacetime (the bit you have got)
https://en.wikipedia.org/wiki/Maxwell's_equations_in_curved_spacetime

As part of the source term in the Einstein field equations, the electromagnetic stress–energy tensor is a covariant symmmetric tensor and that tensor is trace-free or in other words ZERO. Don't fall into the hole that Bill G and Rede did that is very important you need other theories if it isn't true. The justification for the ZERO is electromagnetism propagates at the invariant speed (being c) and conservation of energy in interactions.

You should get the joke when I was asked to define the GR stress-energy tensor and I said Zero. It went straight over some people heads smile

So you can't directly calculate a stress value and you probably can guess why because we actually have nothing to relate it to and so we have to create an indirect measure or a pseudotensor.

In lovely technical language we say this
Quote:
By the equivalence principle gravitational stress–energy will always vanish locally at any chosen point in some chosen frame, therefore gravitational stress–energy cannot be expressed as a non-zero tensor; instead we have to use a pseudotensor.

It does lead to a problem people may and can choose a different tensor definition and again technically we say this
Quote:
In general relativity, there are many possible distinct definitions of the gravitational stress–energy–momentum pseudotensor.

Don't get hung up on it you have seen this everyday or your layman life think metric/imperial smile

If someone uses a different pseudotensor they are describing the same thing in different units and some conversion exists just like metric/imperial.

So you can see my answer of zero was both accurate and a joke and they made it funnier by saying I was wrong and didn't know what I was talking about smile

If Bill G/Rede were more switched on I would have done the GR chicken stress tensor (a play on horse/chicken power) to make a point the choice isn't important laugh

Ask a layman there weight in cartons of beer, sexy girls, sheep manure, feathers etc no problem, but take the concept into a more abstract form and they don't get it smile

Humour aside generally you try a choose a pseudotensor that is not contentious and easy to replicate by anyone and most in the field would know several some examples Einstein, Papapetrou, Bergmann and Møller pseudotensors. If you say look at the Einstein pseudotensor it has a problem it is not symmetric and runs into problems with angular momentum but the name makes sure it is still around today. You really need someone with deeper GR knowledge than mine to give you pro's and con's or each.

Here is the whole lot I have run thru with less detail in layman style
https://diracseashore.wordpress.com/2009/01/21/everyday-physics-the-stress-tensor/
See how you go.

Last edited by Orac; 07/09/15 07:47 AM.

I believe in "Evil, Bad, Ungodly fantasy science and maths", so I am undoubtedly wrong to you.
Orac #54150 07/10/15 02:33 PM
Joined: Aug 2010
Posts: 3,570
B
Megastar
Offline
Megastar
B
Joined: Aug 2010
Posts: 3,570
May be a couple of days before I can get back to this, but I'm still here.


There never was nothing.
Orac #54152 07/11/15 05:59 PM
Joined: Aug 2010
Posts: 3,570
B
Megastar
Offline
Megastar
B
Joined: Aug 2010
Posts: 3,570
Originally Posted By: Orac
If you were trying to be cute and got caught


We should clarify a few things.
1. I’m not proposing a theory.
2. I’m not trying to be clever.
3. I’m not trying to score points.
4. I am trying to learn.
5. If I suggest something might be the case, I am happy to have it confirmed or shot down. Either way, I learn.
6. If you are trying to bog me down in maths or obscure science, don’t bother. It’s too easy so it's a waste of time with.

I have enough training and experience in teaching to know that answering a question with a question is a standard didactic technique, but to treat it as though it were the only technique often results in missing important points. I’m too old, and too short of time to take the long route to everything, so a few straight answers would be appreciated.


There never was nothing.
Orac #54153 07/11/15 06:14 PM
Joined: Aug 2010
Posts: 3,570
B
Megastar
Offline
Megastar
B
Joined: Aug 2010
Posts: 3,570
I’ve had a look at your links and suspect you may be trying to drown me in maths and tech stuff. smile
Let’s step back a bit.

Originally Posted By: Orac
How does gravity get out of a black hole?


Originally Posted By: BS
It doesn’t. Gravity is not a force, it is a feature of the geometry of spacetime. Spacetime is influenced by the presence of the mass of the black hole; not by anything that has to escape from it.


Originally Posted By: Orac
So how does it interact with matter and know when to deform ... divine perception (god tells it)?


If you want me to try proposing an idea (I will not call it a theory), I’ll have a go, and try something a bit more imaginative than "God tells it".

QM indicates that zero is not a possible value of electromagnetic energy. The Casimir Effect supports this.
QM tells us that matter is composed of wavelike energy. We might conclude from this that empty space and matter are the same thing, differing only in their energy states. We might continue this line of thought to say that matter/energy and space/time are the same thing. Changes of energy in matter may lead to detectable changes; e.g. expansion/contraction. The presence of matter/energy in space/time may involve a transition between matter/energy and space/time which results in transitional phenomena, such as expansion/contraction. One might expect that the expansion/contraction would effect both matter/energy and space/time, and that it would be more perceptible in the latter.

The “classic” example of what the Earth would feel (and when) if the sun just vanished is obviously just a thought experiment which relies on a physical impossibility. Matter does not spontaneously appear and disappear.

Excluding individual particles, matter becomes present in masses of increasing size by a process of accretion; thus, the interaction between matter/energy and space/time is an ongoing complex relationship in which the expansion/contraction/deformation of space/time – giving rise to what we experience as gravity – is an integral part.

There is no external influence, the process is integral to the formation and evolution of the Universe.


There never was nothing.
Orac #54154 07/11/15 06:46 PM
Joined: Sep 2012
Posts: 1,209
N
Megastar
Offline
Megastar
N
Joined: Sep 2012
Posts: 1,209
Point 1)

1.) How does gravity get out of a black hole.








...p1....p2....p3....................M >>> motion





^ F
m

(mass m = We right now are registering Grav. signal from past )


Many years ago ( 1000.......00000 Light Yers) Big Mass M was in past in point 1..2...3...

small mass m ( we right now ) are registering very very old position big mass M


WHY WE CAN NOT SEE big MASS M why it is BLACK HOLE ?


light must hit black hole and back to observer ??? Yes
please use HUGE DISTANCE and INVERSE SQUARE LAW


What is very intresting ! small mass m is registering force but
big mass M is not registering any force that is can slown down motion ( I invented this effecte - it is ACTION without reaction model NEWTON III law is too short to be real )



I MADE VERY IMPORTANT DISCOVERY ( easy to prove )
NOT EXIST IN UNIERSE LINE MOTION ( mass m1 is not falling down on mass m2 MASS m1 IS FALING DOWN ON POINT WHERE m2 was in past )

m1=m2

..1...2...3...m1 >>> motion


..1...2...3...m2 >>> motion


mass m1 is faling down on point p1,p2,p3 ....









Point 2)

2.) Why is the speed of gravity restricted to the speed of light

To understand what is Gravitation we must study very old Fact for physics


short theory :

James Clerk Maxwell, in 1861–64, published his theory of electromagnetic fields and radiation, which shows that light has momentum and thus can exert pressure on objects.


Engineers are using Si units system not Idiotics theory


ENERGY / SPACE = Joul/ cubic meter

N*m/m^3 = N/m^2 ( PASCAL !!!! )


http://2.bp.blogspot.com/-C1cgolGV6r0/VUCw2IWncwI/AAAAAAAACWQ/8RhssZGNGKA/s1600/22.JPG



GALILEO NATURAL FALL LAW = MISTAKE

we remember famous experiment on Moon


please compare natural fall down time

1 kg steel ( 7850 kg/m^3 ) and 1 kg EPS ( 58 kg/m^3)

we can compare steel and wood ( remember about Maxwell 1861)

http://4.bp.blogspot.com/-AOVx_ljzTqQ/VP_xutwjrUI/AAAAAAAACQw/HzanrOGF4mA/s1600/CIMG3701.JPG





EXIST TWO TYPES Electro-Magnetic Pascal

Dynamic and Static similar like
Static and Dynamic Air's preasure

We must find ANALOGY !!!( below my interpretation )





IN UNIVERSE exsist many many stars !

Each star is pushing Earth !!! How heavy is Earth

http://4.bp.blogspot.com/-BHgIIFU8ON8/VP_zX3KcL3I/AAAAAAAACQ8/5nk25md2wEk/s1600/CIMG3705.JPG




WE HAVE IN PHYSICS NORMAL AIR PREASURE ., TEMPERATURE and ...

FUTURE MY OWN CLAIM

http://3.bp.blogspot.com/-G8XXBln--6k/VP_2zF35VwI/AAAAAAAACRg/oFtCjI5NMtE/s1600/CIMG3703.JPG






GRAVITATION it is ELECTRO MAGNETIC CAVITATION

Earth is like a big truck
PLACE WHERE EARTH WAS IN PAST = LOWER DENSITY AREA chamber

( photons from stars are hiting Earth - dark colour ground or water ( mirror) energy is changing form

http://2.bp.blogspot.com/-XpgMRvGldH4/VFTgIuJjM3I/AAAAAAAACC8/sBIWRXC5fF4/s1600/CIMG3377.JPG


http://2.bp.blogspot.com/-iNHYy6u9bRg/VF9En99DNmI/AAAAAAAACFA/vOX3fZYchx4/s1600/em.jpg




VERY IMPORTANT PROBLEM FOR PHYSICS
MOTION !!!!



m-----M------m >>> motion ???

how big force is registering left /right m ?

very important is place where Gravitation started life

http://3.bp.blogspot.com/-fEJmNXHYWEE/VUOIBqAuAGI/AAAAAAAACXM/9gbCFna4fb0/s1600/333.png


http://2.bp.blogspot.com/-bi__UJHCX-Q/VUmXV-mvsvI/AAAAAAAACYs/w8VIgCYDGfQ/s1600/grav.jpg





NEW PHYSICS YES WHY NOT !!!

MICHELSON MORLEY BUT !!!! FORCE EDITION

http://3.bp.blogspot.com/-mfX1X9yDqDw/VLuo2Th_6_I/AAAAAAAACNQ/YaGD36e4PRk/s1600/mar.jpg


https://youtu.be/K8aJHtvlBzw

























Last edited by newton; 07/11/15 07:07 PM.
Bill S. #54157 07/12/15 03:15 AM
Joined: May 2011
Posts: 2,819
O
Orac Offline OP
Megastar
OP Offline
Megastar
O
Joined: May 2011
Posts: 2,819
Originally Posted By: Bill S.
I have enough training and experience in teaching to know that answering a question with a question is a standard didactic technique, but to treat it as though it were the only technique often results in missing important points. I’m too old, and too short of time to take the long route to everything, so a few straight answers would be appreciated.

Ok you are dealing in a complex area and I don't like to say "this is the answer just trust me", it is important you make sure the answer you arrive at you are happy with the logic. I can give you what I think but I will draw short of saying this is the answer.

Last edited by Orac; 07/12/15 03:15 AM.

I believe in "Evil, Bad, Ungodly fantasy science and maths", so I am undoubtedly wrong to you.
Bill S. #54159 07/12/15 04:36 AM
Joined: May 2011
Posts: 2,819
O
Orac Offline OP
Megastar
OP Offline
Megastar
O
Joined: May 2011
Posts: 2,819
Originally Posted By: BS
QM indicates that zero is not a possible value of electromagnetic energy. The Casimir Effect supports this.

Ok you have bought in QM which technically isn't need to do GR but lets go with where you want to go. QM and Electromagnetism are formally joined in the theory of Quantum Electrodynamics and it is one of the most tested and precise theories in our science knowledge. It is a relativistic quantum field theory and that means it incorporates relativity (specifically Special Relativity), QM and electromagnetics and so it embodies 3 other sets of understanding within it. Yes along with the casmir effect you therefore arrive at a vacuum expectation value of energy in each point in spacetime.

There are however some problem worth noting, if spacetime is infinite in it's points in space your universe vacuum energy is an infinite quantity. When you use the value you get as a quantum gravity theory the value you get for the cosmological constant is massive compared to it's measured value.

Originally Posted By: BS
QM tells us that matter is composed of wavelike energy. We might conclude from this that empty space and matter are the same thing, differing only in their energy states. We might continue this line of thought to say that matter/energy and space/time are the same thing.

No there are extensions of QM that do that but not all, is it the favoured possibly but again science isn't a consensus vote you can take a personal view on that issue.

You may care to review the tenets of quantum mechanics which says none of that except the wavelike behaviour bit.

Originally Posted By: BS
Changes of energy in matter may lead to detectable changes; e.g. expansion/contraction. The presence of matter/energy in space/time may involve a transition between matter/energy and space/time which results in transitional phenomena, such as expansion/contraction.

Ok now you are getting in to trying to describe interactions and we need to get into details and perhaps I will give you some.

What Einstein noticed is that forces can arrive centred on a point in space based solely on motion you know these as centripetal forces in classic physics. There are several key points to this so lets deal with them

1.) The "fictional force" is focused to a single point in spacetime
2.) The force seems to be transparent to the focal point in spacetime. That focal point in spacetime does not see the force like a media would and you can't seem to create a force so large it crushed or shatter the point. This goes to the point that GR locally takes it's stress tensor to zero and you can't directly measure the force directly on a single point in spacetime well at least one that non zero.

Originally Posted By: BS
One might expect that the expansion/contraction would effect both matter/energy and space/time, and that it would be more perceptible in the latter.

One might expect it based on the matter/energy but the above detail casts a lot of doubt on space/time because at the interface we see and measure nothing.

However what Einstein also realized is he had seen behaviour before in hydraulics. He also knew a basic fact that hydraulic systems will work most efficiently if the hydraulic fluid used has zero compressibility and zero viscosity. Think about the behaviour above. What he ended up at was very different but it was a useful starting point. I am not sure how much you want to go into that so leave that up to you.

Originally Posted By: BS
The “classic” example of what the Earth would feel (and when) if the sun just vanished is obviously just a thought experiment which relies on a physical impossibility. Matter does not spontaneously appear and disappear.

As far as we know all that is correct smile

Originally Posted By: BS
Excluding individual particles, matter becomes present in masses of increasing size by a process of accretion; thus, the interaction between matter/energy and space/time is an ongoing complex relationship in which the expansion/contraction/deformation of space/time – giving rise to what we experience as gravity – is an integral part.

Yes but there is a problem does the gravity come first or the particles, you have a chicken and egg problem.

Originally Posted By: BS
There is no external influence, the process is integral to the formation and evolution of the Universe

Simply no way to answer that we have no evidence there is an external force but let me give you a stupid answer which at present you cant refute. Gravity is because a big green giant has the whole universe in his hand and is swinging us around and it is really the centripedal force of that motion.


I believe in "Evil, Bad, Ungodly fantasy science and maths", so I am undoubtedly wrong to you.
Orac #54161 07/12/15 02:02 PM
Joined: May 2011
Posts: 2,819
O
Orac Offline OP
Megastar
OP Offline
Megastar
O
Joined: May 2011
Posts: 2,819
If you want to review the basics of QM Lubos has just put up a new article on the basics of QM which is worth a read.

http://motls.blogspot.com.au/2015/07/boolean-logic-is-sufficient-to-work.html#more

Again all this is up to you but as you will note is doesn't say the sorts of things you say above.


I believe in "Evil, Bad, Ungodly fantasy science and maths", so I am undoubtedly wrong to you.
Orac #54162 07/12/15 05:08 PM
Joined: Sep 2012
Posts: 1,209
N
Megastar
Offline
Megastar
N
Joined: Sep 2012
Posts: 1,209
Acion without reaction !
[b]F1 = F2 = Gm*M/ R^2 ?




mass M was in past in point p1..p2..p3..
How small m is able give the same reactions to big M ???



EN https://youtu.be/tEv6GxLPJRA

PL - https://youtu.be/vMJjuYJSxDA

Last edited by newton; 07/12/15 05:13 PM.
Orac #54164 07/12/15 08:51 PM
Joined: Aug 2010
Posts: 3,570
B
Megastar
Offline
Megastar
B
Joined: Aug 2010
Posts: 3,570
Orac, perhaps we should examine my earlier post a bit more closely.

1. QM indicates that zero is not a possible value of electromagnetic energy.
2. QM tells us that matter particles behave as waves.
3. This is sometimes interpreted as saying that matter might be composed of wavelike energy, with particles appearing as disturbances in the energy field.
4. This matter/energy duality accords with the concepts of relativity – E=mc2.
5. If statement 1 is correct the vacuum always has non-zero energy, which takes the form of (virtual) particles and waves.
6. Relativity indicates that space and time are linked in such a way that they cannot be thought of as individual entities.
6. This leads us towards a scenario in which a qualitative similarity appears to exist between matter/energy, on the one hand, and space/time on the other.


There never was nothing.
Bill S. #54165 07/13/15 03:20 AM
Joined: May 2011
Posts: 2,819
O
Orac Offline OP
Megastar
OP Offline
Megastar
O
Joined: May 2011
Posts: 2,819
I agree with everything except the last statement

In general I agree with you to a points 1-5 about QM. However QM is not clear about the structure of spacetime other than Space and time are continuous. QM research has considered the possibility that space or even time are discrete but such theories are inevitably inconsistent.

In fact there are two alternate ways to deal with spacetime in QM which contrast to position space you are familiar with in classic physics.

a) Momentum space (https://en.wikipedia.org/wiki/Position_and_momentum_space)
b) Phase space (https://en.wikipedia.org/wiki/Phase_space_formulation)

So sometimes for ease of analysis spacetime is turned into something classically we would not recognize but the treatments are valid under QM.

People view this many different ways from a simply mathematical thing to it is implying something physical and I leave that as an exercise for you if interested and I am happy to answer questions.

Statement 6 directly link space and time in GR correctly.

So I can't in good faith say I agree with the last statement (which should be 7) because whether the two concepts smoothly join is not clear at least to me.

However this is not about me, if you are happy that all connects I won't strenuously object and lets see where you go next.

Last edited by Orac; 07/13/15 03:31 AM.

I believe in "Evil, Bad, Ungodly fantasy science and maths", so I am undoubtedly wrong to you.
Orac #54166 07/13/15 05:04 PM
Joined: Aug 2010
Posts: 3,570
B
Megastar
Offline
Megastar
B
Joined: Aug 2010
Posts: 3,570
Originally Posted By: Orac
....and lets see where you go next.


I wish I knew! smile Possibly good to get this distraction out of the way first.

QED describes the strong, weak and electromagnetic forces in terms of exchanges of messenger particles. It seems reasonable, therefore, to use the same description for gravity, although the graviton, which would be the messenger particle, has not yet been found.

A quantum theory of gravity would necessitate the reality of gravitational waves which, like gravitons, have not yet been observed. There is indirect evidence of the existence of gravitational waves, and if they exist, it is reasonable to consider the graviton as an excitation of a wave in the gravitational field.

An electron is considered as being surrounded by a retinue of virtual photons, constantly appearing and disappearing, and the energy of this combination is calculated as going to infinity in the case of each electron. In the same way, every matter particle must be considered as having a similar retinue of gravitons, the energy of which would go to infinity.

A serious consideration here is that, unlike photons, gravitons would interact with other gravitons. This means that any situation involving loops escalates into multiple loops-within-loops. The energy of each of these loops goes to infinity, so in each case infinities are multiplied - infinitely.

Is this an impediment to finding a quantum theory of gravity?

Renormalization is the standard method of dealing with infinities in these calculations, but that is nothing more than the dubious practice of dividing “both sides” by infinity. As a non-mathematician I can do no more than acknowledge that renormalization seems to work, and is valuable in making progress, but how far can that progress go?


There never was nothing.
Bill S. #54169 07/14/15 04:14 AM
Joined: May 2011
Posts: 2,819
O
Orac Offline OP
Megastar
OP Offline
Megastar
O
Joined: May 2011
Posts: 2,819
Originally Posted By: Bill S.
QED describes the strong, weak and electromagnetic forces in terms of exchanges of messenger particles.

Yes it's a Quantum field theory and so that is how it works.

Originally Posted By: Bill S.
It seems reasonable, therefore, to use the same description for gravity, although the graviton, which would be the messenger particle, has not yet been found.

Yes but nature may not care to be reasonable smile

Originally Posted By: Bill S.
A quantum theory of gravity would necessitate the reality of gravitational waves which, like gravitons, have not yet been observed. There is indirect evidence of the existence of gravitational waves, and if they exist, it is reasonable to consider the graviton as an excitation of a wave in the gravitational field.

Ok you are mixing up things here a gravitation wave is a ripple in spacetime metrics in GR theory.
A graviton is particle or virtual particle in a gravitational field theory.

So we have seen indirect evidence of a gravity wave but no indirect evidence of a graviton.

Originally Posted By: Bill S.
An electron is considered as being surrounded by a retinue of virtual photons, constantly appearing and disappearing, and the energy of this combination is calculated as going to infinity in the case of each electron. In the same way, every matter particle must be considered as having a similar retinue of gravitons, the energy of which would go to infinity.

Yes, No, Maybe you have so many different concepts mixed up it's hard to work smile

Lets start with what a Field theory says, so start here
http://profmattstrassler.com/articles-and-posts/the-higgs-particle/the-higgs-faq-2-0/

The key points as Matt underlines

a) A field is present everywhere in space and time,
b) The field can be, on average, zero or not zero, and
c) Can have waves in it.
d) And if it is a quantum field, its waves are made from particles.

So a GR gravity wave is (c) while a graviton is (d) if that helps above.

He further distinguishes by referring back to earlier articles that a "virtual particle" is simply a disturbance in the field, a "real particle" is a resonant wave that persists in the field. There are other tricky things about virtual particles that they do not have to precisely obey particle behaviour because they are short lived
http://profmattstrassler.com/articles-an...-what-are-they/

So under a Quantum field theory your whole statement is misguided and/or wrong the correct field version he expresses as

Originally Posted By: Prof Matt Strassler
Even to say a particle like an electron is a ripple purely in the electron field is an approximate statement, and sometimes the fact that it is not exactly true matters.

It turns out that since electrons carry electric charge, their very presence disturbs the electromagnetic field around them, and so electrons spend some of their time as a combination of two disturbances, one in in the electron field and one in the electromagnetic field. The disturbance in the electron field is not an electron particle, and the disturbance in the photon field is not a photon particle. However, the combination of the two is just such as to be a nice ripple, with a well-defined energy and momentum, and with an electron’s mass.

In itself the statement won't cause you any issue as a layman, until I point out you and your entire classical universe is therefore nothing more than resonant waves in the fields in spacetime. At which point you usually run for the door, you saw the movie Matrix and didn't like it smile.

So no quantum field theories dispose of classic physics and are very different to the concept of considering virtual particles as real (classical objects) and popping in and out of existence. You do that and then try to borrow calculations from QM field theory, and put simply you can't. You either accept your classical world is gone (IE we have a quantum universe) or rework your theory/ideas from scratch with classical objects and new classical calculations. You are not alone Bill G/Rede struggle a lot with the same issue.

Quote:
Renormalization is the standard method of dealing with infinities in these calculations, but that is nothing more than the dubious practice of dividing “both sides” by infinity. As a non-mathematician I can do no more than acknowledge that renormalization seems to work, and is valuable in making progress, but how far can that progress go?

That is not what renormalization is but it probably isn't important if you get the message above about mixing different ideas.

Sorry for length I tried to shorten responses up as much as possible.

Last edited by Orac; 07/14/15 05:12 AM.

I believe in "Evil, Bad, Ungodly fantasy science and maths", so I am undoubtedly wrong to you.
Orac #54170 07/14/15 05:45 AM
Joined: May 2011
Posts: 2,819
O
Orac Offline OP
Megastar
OP Offline
Megastar
O
Joined: May 2011
Posts: 2,819
I debated whether to discuss real vs virtual particles in QFT so I will put this bit as a new post, it may interest you.

In a practical sense deciding if a particle is real or not is very problematic especially to try and do it classically.
Wikipedia does a terrible job, there is some discussions going to try and clean it up and it ends up at this statement

https://en.wikipedia.org/wiki/Virtual_particle
Quote:
However, all particles have a finite lifetime, as they are created and eventually destroyed by some processes. As such, there is no absolute distinction between "real" and "virtual" particles. In practice, the lifetime of "ordinary" particles is far longer than the lifetime of the virtual particles that contribute to processes in particle physics, and as such the distinction is useful to make.

They have done what you did combined many different ideas and tried to join them into an encompassing statement.

Under Quantum Field Theory that statement is blatantly wrong there is a clear definition like so
Quote:
Virtual particles can violate special relativity as they are not real and special relativity only puts restrictions on real particles. These violations by virtual particles is explained by the uncertainty principle.

You can actually prove the statement by a rather long calculation.

So QFT puts a clear definition under the decision, you find a particle violating SR and it is virtual or just a disturbance in the field using Matt's terminology.

In this regard QFT is unique in that is explains why and what Virtual Particles are.

Last edited by Orac; 07/14/15 05:54 AM.

I believe in "Evil, Bad, Ungodly fantasy science and maths", so I am undoubtedly wrong to you.
Orac #54172 07/14/15 08:03 PM
Joined: Aug 2010
Posts: 3,570
B
Megastar
Offline
Megastar
B
Joined: Aug 2010
Posts: 3,570
Originally Posted By: Orac
Ok you are mixing up things here a gravitation wave is a ripple in spacetime metrics in GR theory.
A graviton is particle or virtual particle in a gravitational field theory.


In order to move towards a QT of gravity, would we not have to consider what Strassler refers to as “quantum waves” in a gravitational field?
How else would we explain gravitons, which would seem to be essential to quantum gravity?


There never was nothing.
Orac #54173 07/14/15 08:20 PM
Joined: Aug 2010
Posts: 3,570
B
Megastar
Offline
Megastar
B
Joined: Aug 2010
Posts: 3,570
Quote:
In itself the statement won't cause you any issue as a layman, until I point out you and your entire classical universe is therefore nothing more than resonant waves in the fields in spacetime. At which point you usually run for the door, you saw the movie Matrix and didn't like it .


I saw all the Matrix films, and enjoyed them. smile

The idea that the “classical universe is therefore nothing more than resonant waves in the fields in spacetime” certainly doesn’t cause me to run for the door. It fits beautifully into my ideas about infinity, but let’s not go there! :P

BTW, I’ve read Matt Strassler’s article about virtual particles a couple of times; it’s probably time to read it again.

I'm very happy with long posts, as long as they say something, which (in this thread) yours do.


There never was nothing.
Bill S. #54175 07/15/15 02:05 AM
Joined: May 2011
Posts: 2,819
O
Orac Offline OP
Megastar
OP Offline
Megastar
O
Joined: May 2011
Posts: 2,819
Originally Posted By: Bill S.
In order to move towards a QT of gravity, would we not have to consider what Strassler refers to as “quantum waves” in a gravitational field?
How else would we explain gravitons, which would seem to be essential to quantum gravity?

Yes everyone has tried but there are some very obvious problems and the reasons.
I will offer a general view on the underlying issues, consider it personal not a science consensus view.

QM mathematics and formulation is very exact and there is only one solution you saw that in above discussion.
However the issue is the solution doesn't resemble the classic world so it cause interpretation issues with the classical world.
You just ran thru that above where the world under QFT looks very different to our perceived world.

GR unfortunately is not so exact there are actually many solutions to it's equations.
I would strongly recommend you looking at exact solutions to GR article
(https://en.wikipedia.org/wiki/Exact_solutions_in_general_relativity)
The point is that GR may be considered a kind of compatibility condition not a specific solution.

I was trying to get you to look at the stress-energy-tensor because it shows you the problem of multiple solutions.
It isn't well covered in science media and the same with dark energy, I think largely due to resistance to GR in the past.
You can have fun with those who rely on Science magazines and don't know the background if you are mean smile
I was hard on Bill G/Rede because they made out what they said was scientific and it simply wasn't, it was more like Marosz.
I may have been brutal to someone else over infinity smile

If you want a comparison here is a list of the current wiki exact solutions in QM
https://en.wikipedia.org/wiki/List_of_quantum-mechanical_systems_with_analytical_solutions
There are probably many more in literature and on the internet.

QM can not be refined it gives just one answer, GR gives many and in a nutshell that is the problem to me.
So to me we are only going to progress by finding the exact solution to GR or some observation pops up to show us the way.
QM science can and does construct many theories that are compatible with GR but it would come down to luck in finding the right one.
I think string theory is an expression of that and why it offers seeming limitless candidates.

It's up to you but armed with that background it might be interesting to look at traditional GR gravity waves under different stress-energy tensors.

Last edited by Orac; 07/15/15 07:00 AM.

I believe in "Evil, Bad, Ungodly fantasy science and maths", so I am undoubtedly wrong to you.
Orac #54176 07/15/15 12:26 PM
Joined: Aug 2010
Posts: 3,570
B
Megastar
Offline
Megastar
B
Joined: Aug 2010
Posts: 3,570
Originally Posted By: Orac
I may have been brutal to someone else over infinity smile


I can't imagine who that would have been. If he even noticed he probably thought "What does he know about infinity?" laugh


There never was nothing.
Orac #54177 07/15/15 12:33 PM
Joined: Aug 2010
Posts: 3,570
B
Megastar
Offline
Megastar
B
Joined: Aug 2010
Posts: 3,570
Originally Posted By: Orac
It's up to you but armed with that background it might be interesting to look at traditional GR gravity waves under different stress-energy tensors.


I would probably need some very basic background on tensors before I could do much with that.


There never was nothing.
Bill S. #54180 07/15/15 04:15 PM
Joined: May 2011
Posts: 2,819
O
Orac Offline OP
Megastar
OP Offline
Megastar
O
Joined: May 2011
Posts: 2,819
You are over thinking it .. start where we all started including Einstein smile

You have a perfect fluid consider the pressure on an individual molecule in the fluid.

http://www.zweigmedia.com/diff_geom/Sec12.html

You don't even have to really follow the mathematics just follow they are describing the pressure at a specific point and trust there mathematics.

The key point here is it is a calculation in general you can never actually measure but it doesn't mean it's useless smile

In the real world the fluid would be less than ideal so it may compress, change viscosity etc which is the reason the calculation was first done.
Do you remember the post I calculated the pressure at the centre of the earth ... very similar smile
I smile because good luck ever measuring it.

To make electomagnetism relativistic you give it the same treatment
https://en.wikipedia.org/wiki/Electromagnetic_stress-energy_tensor

You will see it ends up having the same properties smile

What we are dealing with has a scientific name called a body force.
https://en.wikipedia.org/wiki/Body_force

Quote:
A body force is a force that acts throughout the volume of a body, in contrast to contact forces. Gravity and electromagnetic forces are examples of body forces. Inertial spin forces such as the Centrifugal force, Euler force, and the Coriolis effect are also examples of body forces.

It's that issue that leads to most of the complications from what many did with contact forces at school.
When you throw the frame of reference stuff in on top it gets very complicated.

Last edited by Orac; 07/15/15 04:57 PM.

I believe in "Evil, Bad, Ungodly fantasy science and maths", so I am undoubtedly wrong to you.
Orac #54181 07/16/15 09:21 PM
Joined: Aug 2010
Posts: 3,570
B
Megastar
Offline
Megastar
B
Joined: Aug 2010
Posts: 3,570
As we have strayed into fields and particles, I think this would be a good point at which to check some of my thoughts/questions on that subject.

1. So far, what I think I have is that fields permeate the cosmos. (Is this theory, or is there any way in which it has been/could be established?)
2. On average, fields may be measured as zero or non-zero, at any point in space. (Is that their energy?)
3. We detect fields by observing disturbances in them.
4. I assume this also applies to the gravitational field, because, although gravity waves have not yet been observed directly, we can detect it only because it is disturbed by the presence of matter and/or energy.
5. The normal form taken by disturbances is that of waves.
6. In quantum field theory the waves are [also] particles. (Are there any fields that cannot be described as quantum fields?)
7. All particles are associated with fields, but only at quantum level is this of significance in the present state of our knowledge.


There never was nothing.
Bill S. #54185 07/17/15 03:24 AM
Joined: May 2011
Posts: 2,819
O
Orac Offline OP
Megastar
OP Offline
Megastar
O
Joined: May 2011
Posts: 2,819
That is all pretty much spot on what science says, I will expand a couple of points.

1. So far, what I think I have is that fields permeate the cosmos. (Is this theory, or is there any way in which it has been/could be established?)

QFT goes a little further and says fields make spacetime and consider it to be a physical entity.
GR doesn't go that path because it is based on a classic field in it's inception.

Originally Posted By: John Archibald Wheeler
In the modern framework of the quantum theory of fields, even without referring to a test particle, a field occupies space, contains energy, and its presence eliminates a true vacuum. The fact that the electromagnetic field can possess momentum and energy makes it very real... a particle makes a field, and a field acts on another particle, and the field has such familiar properties as energy content and momentum, just as particles can have. If a particle is real then a field must be considered equally real.

Do you see the subtlety you can expand/contract spacetime, and there is an outside spacetime.
So QFT definitely takes an approach some other areas of science might not agree with.
You may want to consider your favourite infinity under that.

2. On average, fields may be measured as zero or non-zero, at any point in space. (Is that their energy?)
Agreed

3. We detect fields by observing disturbances in them.
Agree and science detected the higgs field in exactly that way.
If you produce a certain collision it reacts with the field and you see the disturbance as a
short lived virtual particle we call the Higgs boson.

4. I assume this also applies to the gravitational field, because, although gravity waves have not yet been observed directly, we can detect it only because it is disturbed by the presence of matter and/or energy.
Agreed and science is uncertain yet if gravity is a quantum field or a classic field.

5. The normal form taken by disturbances is that of waves.
Agree and extend.
The waves are the exchange of energy and the maximum speed of the waves is the speed of light.

6. In quantum field theory the waves are [also] particles. (Are there any fields that cannot be described as quantum fields?)
Agree and gravity may not be a quantum field.
Our current best theory to described gravity is a classic field theory.
Ultimately if gravity is quantum it would follow the normal QFT transformation done on other fields.

7. All particles are associated with fields, but only at quantum level is this of significance in the present state of our knowledge.
[/quote]

Disagree even in a classic sense you have a problem of the chicken and egg scenario, I sort of bought this up with the electron.
So an isolated electron produces and has a field around it at school you may have drawn it like this



See the problem is the electron producing the field or merely visible because of the field.

Classic physics assumes the electron is producing the field and it radiates out to infinity.
QFT says the field was always there you just made it visible by putting a particle at a point.
QFT also says it is correct because of the discovery of the higgs you can't see or even deduce that field in classic physics.
So science answered the chicken and egg question conclusively .. classic physics lost smile

However definitions on "the universe" got a little more interesting didn't they smile
You may care to think of a particle moving vast distances without losing energy in that scheme smile
Which scheme does QM entanglement make more sense in smile

If you ever wanted in a nutshell why classic physics is incompatable with QM there it is.
Bill G doesn't like it but classic physics can lead to really wrong answers ... like above smile
You see classic physics isn't approximately giving the right answer or understanding, and we are going to need to rethink teaching this stuff.

The take home memo is we are never going back to classic physics and QM is never going to disappear, the higgs told you smile

Last edited by Orac; 07/17/15 04:57 AM.

I believe in "Evil, Bad, Ungodly fantasy science and maths", so I am undoubtedly wrong to you.
Orac #54186 07/17/15 06:06 AM
Joined: May 2011
Posts: 2,819
O
Orac Offline OP
Megastar
OP Offline
Megastar
O
Joined: May 2011
Posts: 2,819
Here is your classic physics smile


I believe in "Evil, Bad, Ungodly fantasy science and maths", so I am undoubtedly wrong to you.
Orac #54187 07/17/15 06:30 AM
Joined: May 2011
Posts: 2,819
O
Orac Offline OP
Megastar
OP Offline
Megastar
O
Joined: May 2011
Posts: 2,819
On cue for you today the announcement of the discovery of the Weyl fermion by two different groups

http://phys.org/news/2015-07-year-massless-particle-next-generation-electronics.html
http://phys.org/news/2015-07-weyl-years.html
http://spectrum.ieee.org/tech-talk/semic...ter-electronics

Background how the Weyl fermion fits into QFT.

http://www.quora.com/Quantum-Field-Theory/What-is-a-Weyl-fermion

They have long been sought after because they give you the equivalent to superconductivity at room temperature.

There is no indication of whether these things are likely to show up in the wider universe.
Everyone is hassling the experts in the "standard model and beyond" for that answer as it is outside the realm of QFT/QCD smile

Lubos Motl covered it superficially back in an article in 2010
http://motls.blogspot.com.au/2010/12/beauty-of-dirac-equation-and-its.html

Quote:
The Weyl fermion interpretation is preferred if we want to preserve the symmetry that rotates the phases of the left-handed and right-handed fermions differently; such a symmetry prohibits the Majorana mass terms.

Watch what insights the theorists can give but in general massless particles must be massless for a reason and of coarse there are versions of string theory that predict it, which will get a lot of interest now.

Using the tools of QFT it was always known you could not show that massless charged fermions did not exist and hence the search so a hint of physics beyond the standard model.

Given you know the basics as a layman now, I would be interested in your evaluation of the different science media articles on it.

Last edited by Orac; 07/17/15 07:46 AM.

I believe in "Evil, Bad, Ungodly fantasy science and maths", so I am undoubtedly wrong to you.
Orac #54189 07/17/15 01:45 PM
Joined: Aug 2010
Posts: 3,570
B
Megastar
Offline
Megastar
B
Joined: Aug 2010
Posts: 3,570
Interesting looking links, all I need now is time to read them.

In the meantime, I assume you know about the pentaquark.

http://www.nature.com/news/forsaken-pent...=NzIxOTI5MjIzS0


There never was nothing.
Orac #54190 07/17/15 01:51 PM
Joined: Aug 2010
Posts: 3,570
B
Megastar
Offline
Megastar
B
Joined: Aug 2010
Posts: 3,570
My apologies, AR11, I,ve just realised you beat me to the pentaquark.


There never was nothing.
Bill S. #54195 07/19/15 11:12 AM
Joined: May 2011
Posts: 2,819
O
Orac Offline OP
Megastar
OP Offline
Megastar
O
Joined: May 2011
Posts: 2,819
So a small challenge Bill S. Using your new understanding describe the good and bad in this article

https://medium.com/starts-with-a-bang/embracing-your-fifth-dimension-2e02d9a0572c

Do you think it successfully portraits an accurate representation of science?

My personal view is it basically comes across as an anti-string rant, which isn't really even relevant to many holographic theories. It then tries to put together a mumbo jumbo argument around what it refers to as a bulk spacetime which actually it doesn't ever define. I know what a spacetime bulk is in string theory but it doesn't make sense in this article which basically tries to avoid the stringy stuff.

The key point is it completely misses the fact in physics we are now forced to treat particles as excited states of an underlying physical field that is always already there. That is what makes it look holographic and is at complete odds to the classic physics where the field arise and expand out into space. Many people who don't believe in string theory accept and believe in a holographic scheme.

You said it yourself under QFT you are forced to see an electron or any "real" particle in classic physics as a sort of hologram of the field (Matrix like) ... it's not complicated and nothing to do with string theory or space bulk or the rest of the trash in the article.

Again it looks like another one of those situations that is an article by someone completely out of there depth. If I was being honest it looks like a case of google "holographic principle" and try and write an article using lots of catch phrases and google only picks up the story when the idea is formalized into string theory.

The article as written should have been called AdS/CFT correspondence and basically follows this story
https://en.wikipedia.org/wiki/AdS/CFT_correspondence

Perhaps had they googled a little better they would have found
http://www.thedailybeast.com/articles/2014/08/31/is-the-cosmos-just-a-big-hologram.html

Quote:
The holographic principle isn’t a theory that describes our Universe yet: it’s more a fascinating conjecture that might let us solve some thorny problems in fundamental physics someday. However, a few researchers think we might be able to detect some discrepancies between the three dimensions of space we perceive and a lower-dimensional hologram in the structure of spacetime on the very microscopic level.

You will also find conflicting claims of who created the concept of holographic principle. The earliest reading I know of the claim is 1947 by Denis Gabor who actually invented Holography and want a guess what made him think of the idea smile
https://en.wikipedia.org/wiki/Dennis_Gabor

Perhaps I could suggest a reasonable read
https://books.google.com.au/books/about/...amp;redir_esc=y

I graded the article D- as misdirected or misunderstanding.

Last edited by Orac; 07/19/15 03:43 PM.

I believe in "Evil, Bad, Ungodly fantasy science and maths", so I am undoubtedly wrong to you.
Orac #54196 07/19/15 03:56 PM
Joined: May 2011
Posts: 2,819
O
Orac Offline OP
Megastar
OP Offline
Megastar
O
Joined: May 2011
Posts: 2,819
BTW Bill S if you want a funny read on holographic principle without string theory

http://arxiv.org/abs/1004.1285

Solves the whole thing in only 9 pages and 7 formulas smile

I love this bit
Quote:
I am not saying that string theory is dead. What I am saying is, that string theory cannot be a theory of the fundamental gravitational interaction, since there is no fundamental gravitational interaction.


I believe in "Evil, Bad, Ungodly fantasy science and maths", so I am undoubtedly wrong to you.
Orac #54197 07/19/15 09:06 PM
Joined: Aug 2010
Posts: 3,570
B
Megastar
Offline
Megastar
B
Joined: Aug 2010
Posts: 3,570
Looking back through my notes from a few years ago I found:

“When scientists, such as David Bohm, and authors, such as Michael Taylor describe the Universe as a hologram I very much doubt that they are suggesting that some other-worldly being is projecting laser images with an incredibly gigantic projector to produce what we experience as the Universe. I suspect that it would be more appropriate to say that the three-dimensional images we can produce with laser technology are as near as we can come to producing an effect that, to some extent, mimics the way in which our Universe works.”

I found quite a bit of Taylor’s book interesting, but from what I remember of it, I would have thought it would have been too much like “quantum flapdoodle” for your liking. smile

Still trying to find time for the other links, but will get there.


There never was nothing.
Bill S. #54198 07/20/15 01:13 AM
Joined: May 2011
Posts: 2,819
O
Orac Offline OP
Megastar
OP Offline
Megastar
O
Joined: May 2011
Posts: 2,819
Ah yes now you mention it, I remember David Bohm and yes a different type of holographic universe. I think that also underlines the general idea has been around in many forms for a very long time. I sort of suspect the concept goes back even further than 1947 probably into the 1920's.

For example Kaluza–Klein theory published in 1921 was a classic theory with 5 dimensions and later adapted into string theory
https://en.wikipedia.org/wiki/Kaluza-Klein_theory

I always viewed it as a hidden dimension theory but using your definition above I think I could argue it qualifies as a holographic theory.

Last edited by Orac; 07/20/15 01:54 AM.

I believe in "Evil, Bad, Ungodly fantasy science and maths", so I am undoubtedly wrong to you.
Orac #54199 07/21/15 01:48 PM
Joined: Aug 2010
Posts: 3,570
B
Megastar
Offline
Megastar
B
Joined: Aug 2010
Posts: 3,570
https://medium.com/starts-with-a-bang/embracing-your-fifth-dimension-2e02d9a0572c

“These dualities are said to be “holographic” because they tell us that everything allowed to happen in the bulk space-time of the gravitational theory is encoded on the boundary of that space.”

I find this boundary concept puzzling. Are they saying that our Universe is part of “the bulk”, bounded by 2-brains that form a 2-D box?

Alternatively, could it be that our Universe is considered as having a boundary, beyond which there is nothing?


There never was nothing.
Bill S. #54200 07/21/15 03:34 PM
Joined: May 2011
Posts: 2,819
O
Orac Offline OP
Megastar
OP Offline
Megastar
O
Joined: May 2011
Posts: 2,819
It's branes not brains BTW smile

As I said the whole article reads as like mumbo jumbo to me, a bit like when science magazines try to describe quantum entanglement.

I simply don't get anything that means holographic out of "everything allowed to happen in the bulk space-time of the gravitational theory is encoded on the boundary of that space."

I am not a wordsmith but if I had to try I would go something like this

"The universe in classic physics is described by 3 dimensions and time. Brane models in string theory can use the hidden dimensions of space to project a 3D world onto a normal 2D plane in the same way as computer games do it. Electromagnetic, Strong and weak nuclear forces play out only on the 2D plane while time and gravity play out in all the dimensions but project onto the 2D plane. The 3rd dimension is an illusion and hence this is called a holographic theory".

Hard to work out what boundary is being discussed in that mumbo jumbo. If I had to guess, I think the Brane/other dimensions boundary.

All theories that comply with QM/QFT make a definition of fields that propagate a translationally invariant spacetime. So there are a number of possible boundaries in that definition not just one smile

Now you have progressed I will give you a more interesting boundary to ponder smile

Emmy Noether demonstrated that conservation laws are linked to symmetries, specifically the validity of the energy conservation law is equivalent to the time-translational symmetry of the laws of physics. So outside our universe must be somewhere time-translational symmetry doesn't hold laugh

Last edited by Orac; 07/21/15 03:53 PM.

I believe in "Evil, Bad, Ungodly fantasy science and maths", so I am undoubtedly wrong to you.
Orac #54201 07/21/15 05:12 PM
Joined: Aug 2010
Posts: 3,570
B
Megastar
Offline
Megastar
B
Joined: Aug 2010
Posts: 3,570
Originally Posted By: Orac
It's branes not brains BTW smile


An outstanding example of a Freudian slip!

Quote:
So outside our universe must be somewhere time-translational symmetry doesn't hold.


Of course it is! If there is anything outside our Universe it is the cosmos which is infinite. No time, no change, and it's not really outside, because our Universe is "part" if it, and every part is the whole...."and I am that" laugh

Where's Rev when you need him?


There never was nothing.
Bill S. #54203 07/22/15 03:30 AM
Joined: May 2011
Posts: 2,819
O
Orac Offline OP
Megastar
OP Offline
Megastar
O
Joined: May 2011
Posts: 2,819
Originally Posted By: Bill S.
Of course it is! If there is anything outside our Universe it is the cosmos which is infinite. No time, no change, and it's not really outside, because our Universe is "part" if it, and every part is the whole...."and I am that" laugh

Where's Rev when you need him?

Haha you got the joke ... you have come a long way smile
Didn't some scientist once give you that answer?

If we were religious we just proved we are in hell and that outside our universe is heaven.
Outside here they get everything, land of milk and honey and all that and I did say I was the devil laugh
See you always suspected it was bad here and now you know why smile

I am guessing you are ready to put it all together and Lubos did it really well in an article 5 years ago, you should be able to follow it now.

http://motls.blogspot.com.au/2010/08/why-and-how-energy-is-not-conserved-in.html

You will meet all the key points again from GR
- Why and how the Energy-Stress-Tensor is zero AKA Vanishing Energy
- Reparameterizing
- Energy conservation

Finally you should see the whole lot collapses like a deck of cards the moment you introduce an actual infinity.

Hence you now know anyone who makes a statement about GR and includes an infinity hasn't got a clue. Some who do it will surprise you smile

Anyone who tries to use consensus or claim authority, be comfortable that the janitors of science don't care laugh

Last edited by Orac; 07/22/15 06:26 AM.

I believe in "Evil, Bad, Ungodly fantasy science and maths", so I am undoubtedly wrong to you.
Orac #54204 07/22/15 04:00 AM
Joined: May 2011
Posts: 2,819
O
Orac Offline OP
Megastar
OP Offline
Megastar
O
Joined: May 2011
Posts: 2,819
I put this in a seperate post as it's up to you if you want to look at it.

You may be ready to follow the next extension now as well, if you write all the above out you get this.
https://en.wikipedia.org/wiki/Vacuum_solution_(general_relativity)
See if you can follow it

It answers a couple of problems with gravity you introduced
- Why determining the precise location of this gravitational field energy is technically problematical in general relativity
- Gravitational field energy itself produces more gravity.

The last one at the very least should raise your eyebrows about perpetual motion and why doesn't it go into self induced runaway.

Remember in your first answers I was quizzing you about this and how your theory stopped it smile

BTW if you want a good laugh
http://phys.org/news/2015-07-line-quantum-classical-world.html
You should easily spot the problem, at 80 I think it might be time for the professor to retire smile

Last edited by Orac; 07/22/15 07:00 AM.

I believe in "Evil, Bad, Ungodly fantasy science and maths", so I am undoubtedly wrong to you.
Orac #54205 07/24/15 09:34 PM
Joined: Aug 2010
Posts: 3,570
B
Megastar
Offline
Megastar
B
Joined: Aug 2010
Posts: 3,570
I have tried thinking as I did about infinity a few years ago, taking one step at a time and seeing how far I can go. This time I worked backwards, trying to understand the origin of a black hole’s gravity. Where does it start?

In the case of a black hole that originates as a collapsing star, the star’s gravity is already there, and measurable, before the collapse. The gravitational field of the black hole can be calculated in terms of the gravitation of the original star.

Black holes are generally presented, at least to lay audiences, as things of enormous gravitational attraction. I think the next step in my line of thought would have to be to ask if the gravity at the event horizon is any stronger than the gravity would have been at the surface of the original star. If it is not, then there is no “escaping” gravity to consider; no one/nothing outside the event horizon experiences gravity beyond that of the original star.


There never was nothing.
Bill S. #54207 07/25/15 01:51 AM
Joined: May 2011
Posts: 2,819
O
Orac Offline OP
Megastar
OP Offline
Megastar
O
Joined: May 2011
Posts: 2,819
Originally Posted By: Bill S.
Black holes are generally presented, at least to lay audiences, as things of enormous gravitational attraction. I think the next step in my line of thought would have to be to ask if the gravity at the event horizon is any stronger than the gravity would have been at the surface of the original star. If it is not, then there is no “escaping” gravity to consider; no one/nothing outside the event horizon experiences gravity beyond that of the original star.

It's more complex and interesting than that consider the first part of this
https://en.wikipedia.org/wiki/Supermassive_black_hole

You didn't touch the gravity creates more gravity why doesn't it go into runaway collapse on itself? smile

Last edited by Orac; 07/25/15 01:51 AM.

I believe in "Evil, Bad, Ungodly fantasy science and maths", so I am undoubtedly wrong to you.
Orac #54208 07/25/15 10:16 PM
Joined: Aug 2010
Posts: 3,570
B
Megastar
Offline
Megastar
B
Joined: Aug 2010
Posts: 3,570
I appreciate that supermassive black holes present their own problems, that's why I said: "In the case of a black hole that originates as a collapsing star..."

My question still stands: Is the gravity at the event horizon any stronger than the gravity would have been at the surface of the original star?


There never was nothing.
Bill S. #54209 07/25/15 10:40 PM
Joined: Dec 2010
Posts: 1,858
B
Megastar
Offline
Megastar
B
Joined: Dec 2010
Posts: 1,858
Yes the gravity at the horizon would be greater than it would be at the surface of the original star. That is based on the fact that the horizon is the surface at which the escape velocity would be greater than the speed of light. Obviously the escape velocity at the surface of the star would be less than the speed of light, so the gravity would have to be smaller. However, the gravity at the same distance from the center of the black hole as the surface of the star would be from the center of the star would be the same as it would be the same distance from the center of mass. I'm not sure if this would be affected by GR variations. My first thought is that it wouldn't, because the total mass would not change much. If anything the total mass of the black hole would probably be less than the mass of the star because a lot of the star would be ejected during the collapse.

Bill Gill


C is not the speed of light in a vacuum.
C is the universal speed limit.
Bill S. #54210 07/26/15 04:11 AM
Joined: May 2011
Posts: 2,819
O
Orac Offline OP
Megastar
OP Offline
Megastar
O
Joined: May 2011
Posts: 2,819
Originally Posted By: Bill S.
My question still stands: Is the gravity at the event horizon any stronger than the gravity would have been at the surface of the original star?

Yes/No/Unchanged depends on who and what you believe smile

So look at your options

GR & Bill G it seems: Within the event horizon matter has to move outward faster than the speed of light in order to remain stable and avoid collapsing to the centre. However there is an issue with this that a black hole must then inevitably collapse to a singularity and you saw the problem with infinities above, GR itself collapses under that. Different scientist will resolve that problem different ways the common ones

1.) Ignore it and wave hands that will understand it better one day
2.) Treat things inside the Schwarzschild radius in a special way and make there own pet theory up.
3.) Admit it's a problem and currently no solution

There are hints as to how to resolve this but you seem reluctant to look at it .. if you want to try read next bit smile

Your problem in a nutshell

The fact gravity creates more gravity (I think you got that bit yourself and agreed) means it has to be opposed by something or the moment you created one bit of gravity that point would self collapse to a black hole at that point in a self sustained collapse. In classic physics that is an unavoidable consequence of the mathematics that goes like this

X + a percentage X = Y (Y = 1+X%)
Y + a percentage Y = Z (Z = 1+Y%)
.... etc etc
INFINITY

Your problem above is you want to talk about something without considering what is opposing it laugh

Under your current thoughts any point that creates gravity must instantly collapse to a gravitational singularity don't you think it might be important to fix this up first before dealing with the black hole event horizon smile

So how do you want to deal with this?

Last edited by Orac; 07/26/15 04:19 AM.

I believe in "Evil, Bad, Ungodly fantasy science and maths", so I am undoubtedly wrong to you.
Orac #54212 07/26/15 05:52 PM
Joined: Aug 2010
Posts: 3,570
B
Megastar
Offline
Megastar
B
Joined: Aug 2010
Posts: 3,570
Originally Posted By: Orac
There are hints as to how to resolve this but you seem reluctant to look at it .. if you want to try read next bit

Your problem in a nutshell

The fact gravity creates more gravity (I think you got that bit yourself and agreed) means it has to be opposed by something or the moment you created one bit of gravity that point would self collapse to a black hole at that point in a self sustained collapse. In classic physics that is an unavoidable consequence of the mathematics that goes like this


My only reluctance is that I like to think I understand a point, as well as possible, before moving on to the next. So I need to be able to answer questions like: Can we measure gravity at an event horizon of a stellar BH? If so, how would that compare with gravity at the surface of the original star?

As far as the idea of gravity creating gravity goes, I had reached the point of thinking “does gravity create more gravity? That raised a few questions, like:

Wouldn’t it have to have mass/energy to create more gravity?

If it’s not a force, can it have energy?

Manifestly, gravity is not a run-away phenomenon. Is that because something counters it, or because gravity does not spontaneously create gravity. Which is it?

What physical/experimental evidence do we have for saying that gravity creates gravity?


There never was nothing.
Orac #54213 07/26/15 10:43 PM
Joined: Aug 2010
Posts: 3,570
B
Megastar
Offline
Megastar
B
Joined: Aug 2010
Posts: 3,570
Quote:
Yes the gravity at the horizon would be greater than it would be at the surface of the original star. That is based on the fact that the horizon is the surface at which the escape velocity would be greater than the speed of light. Obviously the escape velocity at the surface of the star would be less than the speed of light, so the gravity would have to be smaller. However, the gravity at the same distance from the center of the black hole as the surface of the star would be from the center of the star would be the same as it would be the same distance from the center of mass. I'm not sure if this would be affected by GR variations. My first thought is that it wouldn't, because the total mass would not change much. If anything the total mass of the black hole would probably be less than the mass of the star because a lot of the star would be ejected during the collapse.


Thanks Bill, I think all that makes sense.


There never was nothing.
Bill S. #54214 07/27/15 02:04 AM
Joined: May 2011
Posts: 2,819
O
Orac Offline OP
Megastar
OP Offline
Megastar
O
Joined: May 2011
Posts: 2,819
Originally Posted By: Bill S.
My only reluctance is that I like to think I understand a point, as well as possible, before moving on to the next. So I need to be able to answer questions like: Can we measure gravity at an event horizon of a stellar BH? If so, how would that compare with gravity at the surface of the original star?

The problem is you have only half the GR theory worked so far.
Your half creates a blackhole immediately any gravity forms, so I can't answer what will happen.

You have a curvature in spacetime in the presence of energy/matter but something has to invert time back to what it was originally. You can make it whatever you like space is elastic, there is a back pressure, the green alien inverts it back but something has to as you have spacetime as passive currently ... see the issue smile

The problem is obvious if time wasn't restored back there would be this big time curvature tracks carved across space where planets/suns moved thru. It would be like a boat moving thru water and the water not levelling the wake left behind it.

I guess you could go for there is no time in space and things with gravity tow time around with them?
So time would be like electric charge under that scheme I guess intrinsic to matter/energy and space is passive.

Both space and time being passive gives huge issues I need a solution from you smile

Originally Posted By: Bill S.
As far as the idea of gravity creating gravity goes, I had reached the point of thinking “does gravity create more gravity? That raised a few questions, like:

Wouldn’t it have to have mass/energy to create more gravity

Energy and it does remember its covered here
https://en.wikipedia.org/wiki/Vacuum_solution_%28general_relativity%29

Originally Posted By: Vacuum GR solution
But the gravitational field can do work, so we must expect the gravitational field itself to possess energy, and it does.


Originally Posted By: Bill S.
If it’s not a force, can it have energy?

Well radiation has energy is it a force?

See this is where it gets tricky for me I need you to define force are talking classic physics/QM/your own?

Originally Posted By: Bill S.
Manifestly, gravity is not a run-away phenomenon. Is that because something counters it, or because gravity does not spontaneously create gravity. Which is it?

How can you get a black hole if it doesn't go into runaway, that is like having an atomic bomb without critical mass ... think about it laugh

History lesson
http://www.physicsoftheuniverse.com/scientists_schwarzschild.html
Quote:
Schwarzschild’s solution identified a radius for any given mass, known as the Schwarzschild radius, where, if that mass could be compressed to fit within that radius, no known force or degeneracy pressure could stop it from continuing to collapse into a gravitational singularity or black hole.

You don't have any back pressure so any gravity at all will collapse into a black hole as it has to overcome ZERO smile

As I said I am open for anything you believe lets just see where it goes smile

Originally Posted By: Bill S.
What physical/experimental evidence do we have for saying that gravity creates gravity?

Einstein and scientists did some tests on a 1919 solar eclipse for a reason to check the curvature of light from the sun smile

Here is the problem in it's conceptual form
https://en.wikipedia.org/wiki/Two-body_problem_in_general_relativity

So you end up with gravity as a radiation although it is still unclear to me if you want to treat it as a force or not.

Last edited by Orac; 07/27/15 07:36 AM.

I believe in "Evil, Bad, Ungodly fantasy science and maths", so I am undoubtedly wrong to you.
Orac #54215 07/27/15 02:41 AM
Joined: May 2011
Posts: 2,819
O
Orac Offline OP
Megastar
OP Offline
Megastar
O
Joined: May 2011
Posts: 2,819
I will put this as it's own comment because it is a statement from me not answering your questions Bill S.

The central premise of GR is that the curvature of space-time is directly determined by the distribution of matter and energy contained within it. What complicates things, however, is that the distribution of matter and energy is in turn governed by the curvature of space, leading to a feedback loop.

That is the nature of the feedback loop in GR, how you want to describe it I leave with you.

Again a statement what opposes gravity in GR is dark energy, it is well covered here in layman terms from the NASA site
http://science.nasa.gov/astrophysics/focus-areas/what-is-dark-energy/

They give you the way Einstein thought of it ("empty space" can possess its own energy), the QM answer (which is wrong) and some other alternates but at the end of the day you end up at this point

Quote:
something that fills all of space but something whose effect on the expansion of the Universe is the opposite of that of matter and normal energy

Whichever answer you take you end up with something creating a tendency to return spacetime to some equilibrium.

I consider Ethan Siegel did two terrible jobs on it but it might work for you and they end up with a negative pressure on any patch of spacetime
http://scienceblogs.com/startswithabang/...verse-combined/
http://scienceblogs.com/startswithabang/2011/12/02/dark-energy-accelerated-expans/

Sean Carroll does all that in a few tight sentences
http://preposterousuniverse.blogspot.com.au/2004/09/dark-energy-equation-of-state.html

If we have dark energy perhaps one might ask what happens to it in a black hole as it might be important smile

Some funny things to probe Bill G about his answer but as he isn't talking to me the point is moot laugh

Last edited by Orac; 07/27/15 08:34 AM.

I believe in "Evil, Bad, Ungodly fantasy science and maths", so I am undoubtedly wrong to you.
Orac #54217 07/27/15 03:15 PM
Joined: Aug 2010
Posts: 3,570
B
Megastar
Offline
Megastar
B
Joined: Aug 2010
Posts: 3,570
OK. Let’s look at three things I think we have said about gravity.

1. Gravity has energy.
2. Gravity distorts spacetime.
3. Gravity creates more gravity.

In achieving 2 & 3, is the energy of gravity diminished, dissipated or exchanged in any way?


There never was nothing.
Bill S. #54219 07/27/15 04:55 PM
Joined: May 2011
Posts: 2,819
O
Orac Offline OP
Megastar
OP Offline
Megastar
O
Joined: May 2011
Posts: 2,819
Originally Posted By: Bill S.
OK. Let’s look at three things I think we have said about gravity.

1. Gravity has energy.
2. Gravity distorts spacetime.
3. Gravity creates more gravity.

In achieving 2 & 3, is the energy of gravity diminished, dissipated or exchanged in any way?

Yes absolutely it is smile

Read the section labelled work done by gravity
https://en.wikipedia.org/wiki/Work_%28physics%29

That is the classic physics version or you can use the more technical GR version
https://en.wikipedia.org/wiki/Energy%E2%80%93momentum_relation

Your current answer means your theory is inconsistent with even classical physics smile

You theory describes a universe other than I know and is the same league as Marosz at the moment.

Last edited by Orac; 07/27/15 04:57 PM.

I believe in "Evil, Bad, Ungodly fantasy science and maths", so I am undoubtedly wrong to you.
Orac #54220 07/27/15 05:04 PM
Joined: May 2011
Posts: 2,819
O
Orac Offline OP
Megastar
OP Offline
Megastar
O
Joined: May 2011
Posts: 2,819
Perhaps I can ask are you trying to work this out and why googling it gives weird answers.

"What is the surface gravity of a black hole" smile

The wikipedia article is accurate but sort of funny
https://en.wikipedia.org/wiki/Surface_gravity

The Surface gravity of a black hole section leads to

surface gravity of a black hole ..... k = 1/4M

Therefore the bigger a black hole mass the smaller value of surface gravity ... no really laugh

Is that what this is all about as you don't really seem to be interested in understanding only the value?

It's very different to the answer Bill G gave but he isn't entirely "wrong" either laugh

Last edited by Orac; 07/27/15 05:17 PM.

I believe in "Evil, Bad, Ungodly fantasy science and maths", so I am undoubtedly wrong to you.
Orac #54221 07/27/15 05:40 PM
Joined: Aug 2010
Posts: 3,570
B
Megastar
Offline
Megastar
B
Joined: Aug 2010
Posts: 3,570
Originally Posted By: Orac
Your current answer means your theory is inconsistent with even classical physics


We seem to be going round in unproductive circles. Could you identify the "answer" to which you refer?


There never was nothing.
Bill S. #54222 07/27/15 11:59 PM
Joined: May 2011
Posts: 2,819
O
Orac Offline OP
Megastar
OP Offline
Megastar
O
Joined: May 2011
Posts: 2,819
Lets take the easy one kinetic energy of a non-rotating object moving at velocity

In classic physics you know it by a simple equation

Kinetic Energy = 1/2 Mass * Velocity * Velocity

Lets change the velocity terms into a different form for you

Velocity = DISTANCE / TIME

So Kinetic Energy = 1/2 Mass * DISTANCE / TIME * DISTANCE / TIME

You still think you can randomly change distance(space) and/or time?

Kinetic energy of a moving object going near your version of spacetime distortion is going to be fun isn't it smile

You might need to be careful changing DISTANCE and TIME or you break all energy equations smile

KE under Bill.S theory:
The Kinetic energy of an object can be anything as it is dependent on the metric of the space it's in and the time factor of that piece of space. For this reason we have dispensed with Kinetic Energy as a concept as it is utterly useless. All discussion now will be about Bill Curvature of spacetime although it isn't clear how Bill Curvature relates to everyday physics.

It's not even clear to me how I can calculate a velocity I have to ask Bill what the curvature and time is at any given point of space so I can calculate my velocity.

Is the problem clear now?

At the moment you have done a Marosz and broken everything and it's not terribly useful smile

So can we be serious now and either discuss how GR does it, or you need to start addressing issues.

Last edited by Orac; 07/28/15 02:00 AM.

I believe in "Evil, Bad, Ungodly fantasy science and maths", so I am undoubtedly wrong to you.
Orac #54223 07/28/15 02:17 AM
Joined: Aug 2010
Posts: 3,570
B
Megastar
Offline
Megastar
B
Joined: Aug 2010
Posts: 3,570
Originally Posted By: Bill S
Could you identify the "answer" to which you refer?


Apparently not.

I’m trying to learn and understand.

Entertaining as your circumlocutory derision may be, your comments are of little value unless you attach them to the “answer” you are criticising.

A plethora of deprecatory comments, and a surfeit of seemingly sneering emoticons is probably not the most productive way forward.

Let's either get back on track, or stop wasting our time.


There never was nothing.
Bill S. #54224 07/28/15 02:28 AM
Joined: May 2011
Posts: 2,819
O
Orac Offline OP
Megastar
OP Offline
Megastar
O
Joined: May 2011
Posts: 2,819
You posted this

Originally Posted By: Bill.S

1. Gravity has energy.
2. Gravity distorts spacetime.
3. Gravity creates more gravity.

In achieving 2 & 3, is the energy of gravity diminished, dissipated or exchanged in any way?

THE ANSWER IS YES ENERGY IS EXCHANGE IT HAS TO BE!!!!!!

Why ... well it's dam obvious ... Energy is what connects the curvature to the normal everyday physics as we know it, after all the curvature is supposed to be what produces the effect we measure isn't it or vice versa for that matter.

The only thing you can measure is energy, name something you measure that has no energy. That is the funny part of time if you have no energy you can't measure and time is meaningless and where you have gone. I think that area has a name usually covered by religion/philosophy as I can't test anything and I have to believe whatever you say.

If the energy isn't connected, you have Bill Curvature which is random but involves time and Energy which involves time but it isn't connected to the curvature ... say what. One may ask is Bill curvature of spacetime and normal physics time even the same thing if they aren't connected.

So at least in physics if you have time you have energy exchange. If we can measure something different then you have a difference in energy. So gravity produces an effect we can measure so it is exchanging energy. So if Bill curvature is caused by physics or is creating physics it can only do so by exchanging energy.

Now this has got totally stupid and gone Marosz can we get back to reality, we need energy exchange to define time at least in physics. I really didn't think I would have to explain that to you.

Now if you really really want to push the envelope and not have energy I at least need some observable I can test so this isn't a religion/philosophy thing.

Assuming you don't want to push the envelope Bill curvature is either caused by something in physics or something in physics causes Bill curvature it matters not which way. I supposedly can measure a cause and effect so therefore energy is being exchanged.

If you can't do either of above we are back to a Bill's law and I don't really care about Bill Curvature it rates about the same as invisible martians push things down as an explaination of gravity ..... AKA Newton gravity ... AKA you aren't offering any solution so why discuss it as there are plenty of religion/philosophy forums.

Lets see if I want to continue this .. here goes

So am I going to be able to measure something or some effect from Bill Curvature AKA are we exchanging energy?

Last edited by Orac; 07/28/15 07:22 AM.

I believe in "Evil, Bad, Ungodly fantasy science and maths", so I am undoubtedly wrong to you.
Orac #54226 07/28/15 05:26 AM
Joined: May 2011
Posts: 2,819
O
Orac Offline OP
Megastar
OP Offline
Megastar
O
Joined: May 2011
Posts: 2,819
Leap of faith we aren't doing philosophy or religion we are exchanging energy smile

I am going to stretch your thinking back from classic physics towards GR. Many of the concepts in GR does not require curved spacetime at all
https://en.wikipedia.org/wiki/Equivalence_principle

Quote:
Einstein combined (postulated) the equivalence principle with special relativity to predict that clocks run at different rates in a gravitational potential, and light rays bend in a gravitational field, even before he developed the concept of curved spacetime.

So what is the reason you need a curvature of spacetime, well to turn GR into a force that you see in classical physics.

Quote:
In general relativity, objects in free-fall follow geodesics of spacetime, and what we perceive as the force of gravity is instead a result of our being unable to follow those geodesics of spacetime, because the mechanical resistance of matter prevents us from doing so.

See the simplicity here there is nothing special about matter here it just can't be moved out of the way. So Einstein defined an equivalence of energy replacing your force with spacetime curve. That equivalence links the curvature to energy in a very precise way or to be exact what Einstein says is that the curvature of space-time and Newtonian stress-energy are the same thing.

You are trying to not talk about classic forces, energy and avoid stress energy in your Bill Curvature statements and I am going like what the hell sort of theory is this!!!!!!!

Now it does get more challenging when we can't simplify to a nice reference frame like above but SR provides most of the answers which GR adopts.

To me you are sort of coming at this backwards you are trying to curve spacetime without first working out why you want to do that and specifically trying to avoid incorporating classic physics.

Last edited by Orac; 07/28/15 08:50 AM.

I believe in "Evil, Bad, Ungodly fantasy science and maths", so I am undoubtedly wrong to you.
Orac #54227 07/28/15 10:38 AM
Joined: Aug 2010
Posts: 3,570
B
Megastar
Offline
Megastar
B
Joined: Aug 2010
Posts: 3,570
Thanks Orac. A few years ago I was trying (in my low-tech, hitch-hiker way) to argue that causing and maintaining spacetime curvature must involve energy exchange. I was being shot down from all angles. It seems that a more devious approach can elicit quite different results. To be fair, I don’t remember if you were among the shooters, but I have my suspicions. smile

I just need to find a bit of time to digest your last couple of posts; I’ll be back.

BTW. A silly thought:

Gravity: that which keeps everyone down.
Grabity: that which lets the few rise above the masses.


There never was nothing.
Bill S. #54228 07/28/15 11:34 AM
Joined: May 2011
Posts: 2,819
O
Orac Offline OP
Megastar
OP Offline
Megastar
O
Joined: May 2011
Posts: 2,819
Originally Posted By: Bill S.
Thanks Orac. A few years ago I was trying (in my low-tech, hitch-hiker way) to argue that causing and maintaining spacetime curvature must involve energy exchange. I was being shot down from all angles. It seems that a more devious approach can elicit quite different results. To be fair, I don’t remember if you were among the shooters, but I have my suspicions. smile

I can say I hope it would have not been me unless I really didn't understand what you were saying or had a bad hang over. If it was me I was wrong and I apologize because I could not have been more wrong, Einstein says you were right and I accept it smile

You may officially give me a hard time if I got it wrong and tell me to go fix up the rest of physics I just broke laugh

Here is what you were asking a more technical way
http://physics.stackexchange.com/questions/70993/how-energy-curves-spacetime
Lubos's answer is the most technically correct and he even tacks in the local feedback but I agree the most voted answer is the best layman answer, and best layman formula ever that says basically says what you did in the statement above.

You were right at least until I or someone else can disprove GR anyhow smile

I am hopeful you get the bit about why the tensor can't be measured directly as anything but zero but it will be very obvious in your surface gravity of a black hole at the event horizon you were doing. Bill G I don't think gets that by his answer to that problem and as I warned you the answer to that problem is actually funny.

Last edited by Orac; 07/28/15 12:18 PM.

I believe in "Evil, Bad, Ungodly fantasy science and maths", so I am undoubtedly wrong to you.
Orac #54230 07/29/15 04:20 PM
Joined: Aug 2010
Posts: 3,570
B
Megastar
Offline
Megastar
B
Joined: Aug 2010
Posts: 3,570
Originally Posted By: Orac
You have a curvature in spacetime in the presence of energy/matter but something has to invert time back to what it was originally. You can make it whatever you like space is elastic, there is a back pressure, the green alien inverts it back but something has to as you have spacetime as passive currently.


This makes little sense to me, unless you mean that as a gravitating body moves through spacetime it distorts spacetime; then something else must act on spacetime in order to restore it to its original, non-distorted, state. Is that what you are saying?


There never was nothing.
Bill S. #54234 07/30/15 04:17 AM
Joined: May 2011
Posts: 2,819
O
Orac Offline OP
Megastar
OP Offline
Megastar
O
Joined: May 2011
Posts: 2,819
Originally Posted By: Bill S.
This makes little sense to me, unless you mean that as a gravitating body moves through spacetime it distorts spacetime; then something else must act on spacetime in order to restore it to its original, non-distorted, state. Is that what you are saying?

Correct you must force it back or if space is passive have a radiation effect.

Lets take any random point in our solar system it's gravitional distortion is based on the effect it feels from every point in the universe. Lets deal with that not from the really really distant ones but the close ones but the lets say from earth to sun like distances.

You know the calculation the sun is 8 minutes away at the speed of light from earth which is the speed of gravity theoretically. So lets take a point right angles either side of the suns position right now and 8 minutes at the speed of light away. The deformation is some value x. Now roll forward one second. The sun has moved as we were initially right angles (the shortest distance) the distance to us must have increased slightly and so the gravity must drop by a small amount delta so we have x - delta. The key point here that point must not only distort it must undistort somehow.

We are back to the exact same situation as electric charge there are exactly three options here

1. The object is radiating gravity and space is passive. So the gravity at any point in space is the sum of the gravitational radiation of all objects. That is how classic physics taught electromagnetism so you end up with gravity looking like that radiating from everything. You can even end up drawing pretty radiation field lines like you did in classic electromagnetics. Big objects radiate the attractive force more than little objects because they are bigger and we have our theory.

2. The object is passive and space has an energy that can be deformed in the presence of mass/energy but will return back to normal when it moves. We saw this with Quantum Field Theory that particles become viewed as just a disturbance in the field and so gravity would be just some disturbance in some field. Underpinning this idea however is the the distortion of the field is unnatural and the field is fighting to return it to uniformity as the distortion carries energy. Hence a little mass distorts a little way a big mass further, you get your proportionality. In QFT we even had resonant points that could persist for long periods because the resonant point stopped that force being seen and we called them "real particles" and "virtual particles" were decayed by that force. In Electromagnetics we call it back EMF or counter-EMF, the opposition to deform the field. You can't have a field of this type without a back pressure, think about why.

3. Some mix of the above two schemes. We didn't really discuss this with QFT but there could be some sort of mix of both effects. The whole idea called heralding, it's sort of like phone ahead or notify in advance. In QM there are quite a few things that can be setup to be heralded so you know they are going to happen but as you can imagine this would make gravity really complex. This is like hidden variables explaining QM.

So number one would be how Newtonian physics would work with things radiating attractive force. GR fits into two and Einstein realized he needed a pressure to push spacetime back so he introduced the cosmological constant AKA dark energy.

The story is well covered in
https://en.wikipedia.org/wiki/Cosmological_constant

Einstein did it as the article says to "hold back gravity" and achieve a static universe.

Last edited by Orac; 07/30/15 06:58 AM.

I believe in "Evil, Bad, Ungodly fantasy science and maths", so I am undoubtedly wrong to you.
Orac #54235 07/30/15 04:22 AM
Joined: May 2011
Posts: 2,819
O
Orac Offline OP
Megastar
OP Offline
Megastar
O
Joined: May 2011
Posts: 2,819
My personal views, isolated because it is my view

From reading Einsteins work I get the impression he thought of energy as a "quantity" and the universe as a bath full of it because of the laws of conservation. As he viewed it that way he arrived at modelling the pressure as a perfect fluid. I think his logic was energy had some property internally that makes it want to spread evenly like a something dissolved in liquid solution. As energy clump together it created an a pressure which in a normal liquid solution you would call osmotic pressure ( https://en.wikipedia.org/wiki/Osmotic_pressure)

The reason Einstein made the stress tensor always zero is because he made it like a surface tension of zero in a fluid. No such fluid exists by the way so why do it you ask.

Well surface tension is not a property of materials but of interfaces between two (or more) materials. It is implicit in its definition that the interface separates two kinds of materials that behave differently (otherwise the interface would be just some imaginary surface inside the one material with no physical meaning). In real liquids that surface is indeed subject to blurring because of forces which is why no zero surface tension fluids exist.

Worth a quick look it's short
https://en.wikipedia.org/wiki/Perfect_fluid

I think Einstein wanted a clean division between the energy and its internal pressure and our physical world. It gets messy if you start modelling interface tension in GR and the maths blows up but some people have tried it. So Einstein wanted a clean divide between the energy domain and our physical world, I think the motivation was E=mc2 it's either one or other with c as a constant.

Do you understand now why you are never going to be able to measure the tensor directly as anything other than zero if Einstein is right?

However I also scratch my head if Einstein believed that of energy why did he have so much trouble with QM.

Anyhow there you have it why something called "Dark Energy" is actually needed in any non radiative version of gravity. GR is a non radiative version and has that back pressure inbuilt into it. If you make a field version of GR you must in build the back pressure of deforming the field. Finally the tensor is always zero in GR because of the clean symmetrical interface it defines.

Bonus now if anyone doing electronics asks you what the back-electromotive force or Lenz's law exists you can explain it smile

You know how I love cosmologists and consensus well apparently I am right to a science confidence of 99.996% smile
http://www.port.ac.uk/uopnews/2012/09/12/dark-energy-is-real-say-astronomers/

GR isn't my area and I can't go much deeper than this but I do know the basics but I could always be wrong !!!!!

If you understand all that you actually have got generalized field theory understood ... what a field is in physicality that is a whole other story smile

The other lesson is the one Bill G failed at... look at what is being said, not who said it. For example I always read Lubos Motl on GR even though he is a string theorist because he knows GR very very well, I have seen that on a number of physics forums. Don't take him as gospel but do read what he says very carefully.

Last edited by Orac; 07/30/15 07:36 AM.

I believe in "Evil, Bad, Ungodly fantasy science and maths", so I am undoubtedly wrong to you.
Orac #54239 07/30/15 09:47 PM
Joined: Aug 2010
Posts: 3,570
B
Megastar
Offline
Megastar
B
Joined: Aug 2010
Posts: 3,570
Originally Posted By: Orac
You know the calculation the sun is 8 minutes away at the speed of light from earth which is the speed of gravity theoretically. So lets take a point right angles either side of the suns position right now and 8 minutes at the speed of light away. The deformation is some value x. Now roll forward one second. The sun has moved as we were initially right angles (the shortest distance) the distance to us must have increased slightly and so the gravity must drop by a small amount delta so we have x - delta. The key point here that point must not only distort it must undistort somehow.


Surely this apparent movement is due to the rotation of the Earth. The angle between sun and Earth does not change significantly. Obviously the fact that the Earth's orbit around the sun is not circular changes the angle through the year, but your 1 sec interval is not going to mark a significant difference to that.


There never was nothing.
Orac #54240 07/30/15 09:56 PM
Joined: Aug 2010
Posts: 3,570
B
Megastar
Offline
Megastar
B
Joined: Aug 2010
Posts: 3,570
Originally Posted By: Orac
The problem is obvious if time wasn't restored back there would be this big time curvature tracks carved across space where planets/suns moved thru. It would be like a boat moving thru water and the water not levelling the wake left behind it.


Water pressure smoothes out the wake; why would vacuum pressure not smooth out the spacetime distortions?


There never was nothing.
Orac #54241 07/30/15 10:46 PM
Joined: Aug 2010
Posts: 3,570
B
Megastar
Offline
Megastar
B
Joined: Aug 2010
Posts: 3,570
Originally Posted By: Orac
.....there are exactly three options here.


Could there be a 4th?

The Universe is a gravitational field, particles of matter are distortions in that field. Energy is a distortion in the gravitational field. All other fields and particles are subsidiary to this.

Just a thought.


There never was nothing.
Bill S. #54242 07/31/15 01:24 AM
Joined: May 2011
Posts: 2,819
O
Orac Offline OP
Megastar
OP Offline
Megastar
O
Joined: May 2011
Posts: 2,819
Originally Posted By: Bill S.
Surely this apparent movement is due to the rotation of the Earth. The angle between sun and Earth does not change significantly. Obviously the fact that the Earth's orbit around the sun is not circular changes the angle through the year, but your 1 sec interval is not going to mark a significant difference to that.

I didn't introduce Earth there it was a blank empty piece of space? If you want something there lets put a neutron particle there as it will provide almost no other interactions. Sorry I only used earth/sun as a distance because I know you knew the time of flight of light. Sorry my for my bad English again, I concentrate on the translation so much I lost what I meant.

So the point is a blank empty piece of space yet the gravity field must reflect every gravity attraction from every point in the universe and it must go up and down.

Last edited by Orac; 07/31/15 06:29 AM.

I believe in "Evil, Bad, Ungodly fantasy science and maths", so I am undoubtedly wrong to you.
Bill S. #54243 07/31/15 01:31 AM
Joined: May 2011
Posts: 2,819
O
Orac Offline OP
Megastar
OP Offline
Megastar
O
Joined: May 2011
Posts: 2,819
Originally Posted By: Bill S.
Water pressure smoothes out the wake; why would vacuum pressure not smooth out the spacetime distortions?

Well two things

1.) You haven't introduced a vacuum pressure so far? Space has vacuum but no-one has ever measured a pressure because of next point.
2.) Vacuum pressure requires a positive pressure to already exist and you remove something in layman terms.

For the record the measured vacuum pressure in space near earth is 3x10E-9 Pascals and that is positive caused by the few stray particles and it is the wrong way it's going to keep spacetime curved.

You need a negative pressure so any ideas?

In meantime a hint: casimir force.
https://en.wikipedia.org/wiki/Casimir_effect

Quote:
The Casimir effect can be understood by the idea that the presence of conducting metals and dielectrics alters the vacuum expectation value of the energy of the second quantized electromagnetic field.Since the value of this energy depends on the shapes and positions of the conductors and dielectrics, the Casimir effect manifests itself as a force between such objects.

Casimir is again the idea that deforming a field involves energy and creates a force within the field. We know electomagnetic fields do it we can measure it and it's fully resolved with quantum electrodynamics.

Sorry had to bring QM in Bill G will be on me whistle

Last edited by Orac; 07/31/15 06:38 AM.

I believe in "Evil, Bad, Ungodly fantasy science and maths", so I am undoubtedly wrong to you.
Bill S. #54244 07/31/15 02:13 AM
Joined: May 2011
Posts: 2,819
O
Orac Offline OP
Megastar
OP Offline
Megastar
O
Joined: May 2011
Posts: 2,819
Originally Posted By: Bill S.
Could there be a 4th?

The Universe is a gravitational field, particles of matter are distortions in that field. Energy is a distortion in the gravitational field. All other fields and particles are subsidiary to this.

Just a thought.

That is actually the same as 2 ... and now you have converted a classic gravity field theory to a quantum gravity theory.

Remember we did that in QFT.


I believe in "Evil, Bad, Ungodly fantasy science and maths", so I am undoubtedly wrong to you.
Orac #54247 07/31/15 07:36 PM
Joined: Aug 2010
Posts: 3,570
B
Megastar
Offline
Megastar
B
Joined: Aug 2010
Posts: 3,570
Originally Posted By: Orac
That is actually the same as 2


I thought this went beyond 2.

In 2: "gravity would be just some disturbance in some field."

In 4: matter and energy are distortions in the gravitational field. Everything else arises as excitations of the gravitational field, which is fundamental. What initiates these excitations? Quantum uncertainty, of course. smile

Quote:
and now you have converted a classic gravity field theory to a quantum gravity theory.


Bill S cracks quantum gravity! or not! Might have to wait a bit for the Nobel.


There never was nothing.
Orac #54248 07/31/15 07:58 PM
Joined: Aug 2010
Posts: 3,570
B
Megastar
Offline
Megastar
B
Joined: Aug 2010
Posts: 3,570
Originally Posted By: Orac
Originally Posted By: Bill S
OK. Let’s look at three things I think we have said about gravity.

1. Gravity has energy.
2. Gravity distorts spacetime.
3. Gravity creates more gravity.

In achieving 2 & 3, is the energy of gravity diminished, dissipated or exchanged in any way?
Yes absolutely it is


This brings us back to questions I was asking a few years ago about where gravity’s seemingly inexhaustible supply of energy comes from. Why, for example, after thousands of millions of years of holding the moon in orbit, and keeping vast quantities of loose objects “stuck” to the Earth’s surface, does gravity show no signs of lessening. Among the responses I received then were things like: “No energy is expended”.

We now seem to have reached a point where we are saying that gravity does work, therefore it expends energy.


There never was nothing.
Bill S. #54249 08/01/15 02:44 AM
Joined: May 2011
Posts: 2,819
O
Orac Offline OP
Megastar
OP Offline
Megastar
O
Joined: May 2011
Posts: 2,819
Originally Posted By: Bill S.
Bill S cracks quantum gravity! or not! Might have to wait a bit for the Nobel.

I think your just behind Marosz in line for the Nobel smile


I believe in "Evil, Bad, Ungodly fantasy science and maths", so I am undoubtedly wrong to you.
Bill S. #54250 08/01/15 03:06 AM
Joined: May 2011
Posts: 2,819
O
Orac Offline OP
Megastar
OP Offline
Megastar
O
Joined: May 2011
Posts: 2,819
Originally Posted By: Bill S.
This brings us back to questions I was asking a few years ago about where gravity’s seemingly inexhaustible supply of energy comes from. Why, for example, after thousands of millions of years of holding the moon in orbit, and keeping vast quantities of loose objects “stuck” to the Earth’s surface, does gravity show no signs of lessening. Among the responses I received then were things like: “No energy is expended”.

We now seem to have reached a point where we are saying that gravity does work, therefore it expends energy.

Ok Gravity most certainly does expend energy as we can show it does work and exchanges energy.

The problem comes back to the tensor being zero, Bill G's little friend.

I usually don't like Sean Carroll but on this subject he has written the most honest evaluation
http://www.preposterousuniverse.com/blog/2010/02/22/energy-is-not-conserved/

So you can either build a whole massive amount of energy into the gravity field which you are never going to be able to measure OR just say energy is not conserved.

LM gives it a similar treatment in more technical language
http://motls.blogspot.com.au/2010/08/why-and-how-energy-is-not-conserved-in.html

Quote:
The main lesson here is that general relativity is not a theory that requires physical objects or fields to propagate in a pre-existing translationally invariant spacetime. That's why the corresponding energy conservation law justified by Noether's argument either fails, or becomes approximate, or becomes vacuous, or survives exclusively in spacetimes that preserve their "special relativistic" structure at infinity. At any rate, the status of energy conservation changes when you switch from special relativity to general relativity.


In simple terms Energy conservation does not hold in general relativity (GR) because it is replaced by Energy stress tensor which we set to zero and therefore does not include gravitational potential energy. Now you know the dirty little secret I was trying to get Bill G to understand.

Last edited by Orac; 08/01/15 03:07 AM.

I believe in "Evil, Bad, Ungodly fantasy science and maths", so I am undoubtedly wrong to you.
Orac #54253 08/01/15 10:47 PM
Joined: Aug 2010
Posts: 3,570
B
Megastar
Offline
Megastar
B
Joined: Aug 2010
Posts: 3,570
Originally Posted By: Orac
I usually don't like Sean Carroll


For me, one of Carroll's strong points is that he usually seems to be able to explain things in a way that I stand a good chance of understanding. I know that doesn't make him right (or wrong), but if I can understand it, I can form a judgement.

At a brief glance, this article seems to be saying that in physics you can say that energy is conserved, or it is not conserved, and be right whichever you say.

Hopefully I shall have time to read it properly tonight.


There never was nothing.
Orac #54254 08/01/15 11:00 PM
Joined: Aug 2010
Posts: 3,570
B
Megastar
Offline
Megastar
B
Joined: Aug 2010
Posts: 3,570
There's something from earlier in the thread I would like to return to.

Originally Posted By: Orac
How can you get a black hole if it doesn't go into runaway, that is like having an atomic bomb without critical mass ... think about it


In order to ensure we were talking about the same thing, you would have to be quite clear as to whether you are treating the centre of a BH as a singularity, or not.

We would also have to be sure that by “singularity” we understood the same thing.


There never was nothing.
Bill S. #54255 08/02/15 07:28 AM
Joined: May 2011
Posts: 2,819
O
Orac Offline OP
Megastar
OP Offline
Megastar
O
Joined: May 2011
Posts: 2,819
Originally Posted By: Bill S.
In order to ensure we were talking about the same thing, you would have to be quite clear as to whether you are treating the centre of a BH as a singularity, or not.

We would also have to be sure that by “singularity” we understood the same thing.

Can I suggest we do this a different way and we start with massive black holes because they are easier to follow and understand than a collapsing sun forming a black hole.

For example if we created a latex film and enclosed the area out to saturn with the sun as a central point and filled it with air to the atmospheric pressure of earth, a black hole would open up at the sun. Really doesn't take much matter in a confined area to form one. That is why anywhere that matter accumulates you almost always get a black hole, such at the centre of galaxies.

So the creation is interesting and in a lot of ways more interesting than the black hole itself and tells you things. It's up to you but the singularity argument is actually a side issue to other problems you will face.

The one I would first like to look at is infinite gravity and what it actually means and will it kill you.

Hint: GR says if you moved towards a supermassive black hole you would easily pass thru the event horizon and it's huge gravity probably barely noticing it to inevitably die horribly some time later. What does gravity on a human body actually mean and how much does it take to kill a human smile

hint 2: The gravity at the 4200 miles up the international space station circles at is 89% what is on earth. However the special path the ISS takes makes conditions there almost zero gravity. Could you do such a path around a black hole and what is the maximum gravity you could thus be exposed to, see gravity as a force is tricky something layman don't often get and an infinite force may not mean anything special smile

Reference if you need it:
http://profmattstrassler.com/articles-an...-space-station/

Hint 3: If you believe in singularities in a black hole. It is a general fact you will be torn apart by a black hole approximately a tenth of a second before you hit the singularity, independent of the mass of the black hole. Why? smile

Last edited by Orac; 08/02/15 01:14 PM.

I believe in "Evil, Bad, Ungodly fantasy science and maths", so I am undoubtedly wrong to you.
Orac #54257 08/02/15 02:16 PM
Joined: Aug 2010
Posts: 3,570
B
Megastar
Offline
Megastar
B
Joined: Aug 2010
Posts: 3,570
Originally Posted By: Orac
Can I suggest we do this a different way and we start with massive black holes because they are easier to follow and understand than a collapsing sun forming a black hole.


That’s OK with me. All I was doing was seeking clarification so as to avoid singularities causing problems later.

Do super-massive not have singularities at the centre?


There never was nothing.
Bill S. #54258 08/03/15 01:34 AM
Joined: May 2011
Posts: 2,819
O
Orac Offline OP
Megastar
OP Offline
Megastar
O
Joined: May 2011
Posts: 2,819
They are the same as solar mass black holes in make-up just very very big and theoretically born a different way. If solar mass black stars contain a singularity then so would super massive black stars.

As I said I would first like you to look at is very very large gravity sources (bordering on infinite) and what it actually means and will it kill you as it might shed light on what infinite gravity actually means.

The second issue I would like to cover is what a singularity looks like in a static black hole versus a rotating black hole do you know the layman science magazine view on the difference?

If you don't here is the reference:
https://en.wikipedia.org/wiki/Ring_singularity

Last edited by Orac; 08/03/15 03:40 AM.

I believe in "Evil, Bad, Ungodly fantasy science and maths", so I am undoubtedly wrong to you.
Orac #54259 08/03/15 11:01 PM
Joined: Aug 2010
Posts: 3,570
B
Megastar
Offline
Megastar
B
Joined: Aug 2010
Posts: 3,570
Originally Posted By: Orac
As I said I would first like you to look at is very very large gravity sources (bordering on infinite) and what it actually means and will it kill you as it might shed light on what infinite gravity actually means.


I know what you mean, so I’ll not argue with “bordering on infinite”.

you introduced rotating and non-rotating BHs as a second point, but it might be useful to include them under the first heading. It might be good to check first that my thoughts/understanding about these are bordering on correct.

Kerr BHs rotate; Schwarzschild BHs do not.
A Schwarzschild BH can have a “point” singularity at its centre.
A Kerr BH has a central ring, sometimes called a ring singularity.
A stellar BH must rotate, because the original star would have been rotating, and angular momentum is conserved.
Galaxies rotate, so it seems reasonable to assume that the BHs at their centres also rotate.
To date, there are no known examples of non-rotating BHs.

This links to the first point in that if there are Schwarzschild BHs, and if they have singularities at their centres, any close approach to the singularity would be fatal.

Quote:
….I would first like you to look at is very very large gravity sources (bordering on infinite) and what it actually means and will it kill you…


Get too close to a point singularity, where gravity goes to infinity, and it will turn you into spaghetti.


There never was nothing.
Bill S. #54260 08/04/15 02:57 AM
Joined: May 2011
Posts: 2,819
O
Orac Offline OP
Megastar
OP Offline
Megastar
O
Joined: May 2011
Posts: 2,819
Originally Posted By: Bill S.
Kerr BHs rotate; Schwarzschild BHs do not.
A Schwarzschild BH can have a “point” singularity at its centre.
A Kerr BH has a central ring, sometimes called a ring singularity.
A stellar BH must rotate, because the original star would have been rotating, and angular momentum is conserved.
Galaxies rotate, so it seems reasonable to assume that the BHs at their centres also rotate.
To date, there are no known examples of non-rotating BHs.

Absolutely correct to all.

We can add that we have never see a sun or planet that doesn't rotate, many consider it impossible for a celestial body to form as a direct consequence of the laws of gravity and conservation of angular momentum. It would follow a collapsing star should be rotating and one may ponder that there really could be such thing as a static back hole.

This is the sort of subtle problem I have with the usually naive discussions around black holes, reality and logic are usually the first victim of the discussion.

I wouldn't waste my time discussing static back holes unless you first convince me how one could form or that there is evidence of one.

Originally Posted By: Bill S.
This links to the first point in that if there are Schwarzschild BHs, and if they have singularities at their centres, any close approach to the singularity would be fatal.

.. snip

Get too close to a point singularity, where gravity goes to infinity, and it will turn you into spaghetti.

Correct you have the basics the thing that will kill you is the rate of change of gravity creating a tidal difference in you not gravity itself. Most layman don't get that there is no absolute value of gravity you could not survive provided you were in free fall. The formula is given in the wiki discussion
https://en.wikipedia.org/wiki/Spaghettification

So I completely agree with everything you have said but now want to look at more mundane physics infinities.

Lets start with the a simple one which is the formula for induction in electricity .... V=L(di/dt). That says voltage induced across a coil equals the inductance multiplied by the change in current, divided by the change in time. If the change in time goes zero the voltage produced goes to infinity.

That formula is correct to all know values and you see it in your car ignition system, all switch mode power supplies and most TV's and monitors where breaking current flowing in an inductor is used to create very high voltages and usually even extended higher by use of transformer as the inductive component.

Theoretically if you could open a switch instantly with (dt=0) you would and could perfectly insulate you would indeed generate infinite voltage. This is point I am trying to get you to think about infinities require theoretical conditions that can not exist in the real world. The fact a theory or law predicts an infinity does not mean it exists.

We saw just the subtle change of rotation changed what GR said about the singularity. GR is just another physics theory that predicts an infinity, for me I don't expect the infinity to ever be realized because I think there will be a number of factors in the real world that stop it.

Unfortunately practical reality doesn't sell glossy science magazines. What would be interesting would me trying to sell the story to layman, that their car ignition coil could open it's points in zero seconds and the atmosphere might be perfect for insulation and so they could die to the infinite voltage. No scientist can say I am wrong because that is what the theory and law says they would instead argue real world modifies the prediction smile

So I take a fairly standard approach when I see infinity in theories that it just means some high value that would be moderated by the real world. Hence I don't take all the hype and ventilation over gravity singularities too seriously, I would need to see experimental results saying it exists in reality.

Last edited by Orac; 08/04/15 05:12 AM.

I believe in "Evil, Bad, Ungodly fantasy science and maths", so I am undoubtedly wrong to you.
Orac #54262 08/04/15 06:16 PM
Joined: Aug 2010
Posts: 3,570
B
Megastar
Offline
Megastar
B
Joined: Aug 2010
Posts: 3,570
Originally Posted By: Orac
I wouldn't waste my time discussing static back holes unless you first convince me how one could form or that there is evidence of one.


No way! I can’t see how they could be a physical reality. I just wanted to be sure that if we were talking about singularities, they were ring singularities.

Quote:
Lets start with the a simple one which is the formula for induction in electricity .... V=L(di/dt). That says voltage induced across a coil equals the inductance multiplied by the change in current, divided by the change in time. If the change in time goes zero the voltage produced goes to infinity.

That formula is correct to all know values and you see it in your car ignition system, all switch mode power supplies and most TV's and monitors where breaking current flowing in an inductor is used to create very high voltages and usually even extended higher by use of transformer as the inductive component.


Therefore, voltage can be infinite for a period of no time. I have difficulty making a distinction between something that happens for a period of no time, and something that does not happen at all. Infinite voltage is evidently theoretically possible, but is not possible in our 3+1 dimensional Universe.

Quote:
Theoretically if you could open a switch instantly with (dt=0) you would and could perfectly insulate you would indeed generate infinite voltage. This is point I am trying to get you to think about infinities require theoretical conditions that can not exist in the real world. The fact a theory or law predicts an infinity does not mean it exists.


This is exactly what I have been criticised for saying for years. smile

Quote:
We saw just the subtle change of rotation changed what GR said about the singularity. GR is just another physics theory that predicts an infinity, for me I don't expect the infinity to ever be realized because I think there will be a number of factors in the real world that stop it.


The major one being that the “real world” which we experience is finite, so cannot “contain” infinity.

Quote:
Hence I don't take all the hype and ventilation over gravity singularities too seriously, I would need to see experimental results saying it exists in reality.


Good luck with that. smile


There never was nothing.
Bill S. #54267 08/05/15 02:35 AM
Joined: May 2011
Posts: 2,819
O
Orac Offline OP
Megastar
OP Offline
Megastar
O
Joined: May 2011
Posts: 2,819
I would say we probably have about the same view on this stuff.

My view is the more mundane infinities science run across they can easily identify the real world restrictions that will prevent it. In much of the spacetime discussion we don't know the problem well enough to know the restrictions but that doesn't mean there are no restrictions.

The telling point to me is we don't waste time arguing over infinite voltage, yet some sections of science waste time arguing over singularities. I think they missed the lesson that most theory predictions assume ideal unrealistic conditions in the real world your answer will be approximated smile

If it's any consolation we are not alone
http://www.askamathematician.com/2012/09/q-what-are-singularities-do-they-exist-in-nature/
Just our practicality doesn't provide stories for glossy science magazines or web sites.

Last edited by Orac; 08/05/15 02:55 AM.

I believe in "Evil, Bad, Ungodly fantasy science and maths", so I am undoubtedly wrong to you.
Orac #54269 08/05/15 01:01 PM
Joined: Aug 2010
Posts: 3,570
B
Megastar
Offline
Megastar
B
Joined: Aug 2010
Posts: 3,570
For AAM that seems a very sensible answer. I tend to be a bit suspicious of that forum. At

http://www.askamathematician.com/2011/07/q-does-light-experience-time/#comments

there’s this chap, Bill S who seems to be fielding a lot of the questions. I don’t think he knows much. laugh


There never was nothing.
Bill S. #54270 08/05/15 04:13 PM
Joined: May 2011
Posts: 2,819
O
Orac Offline OP
Megastar
OP Offline
Megastar
O
Joined: May 2011
Posts: 2,819
You should know the answer to that question with all your discussion we had above it's not hard.

The problem for a photon and time in classic physics is the E & B (electric and magnetic) fields can't be described in a reference frame of the photon at least not in a manner that makes any sense in our reference frame. So a photon can't be used as a reference frame for itself much less anything else in classic physics including GR.

You can resolve the paradox in Quantum Field Theory (QFT) because the photon is nothing but a manifestation of the fields. Time for the fields remains real and it can be used for a reference frame ... try it ... you should get an answer close to one you were given with hand waving using classic physics smile

Sometimes QM/QFT gives obvious answers that make more sense than classic physics.

You disappointed me with not rethinking that problem.

Trying to describe the world from a rainbow is always problematic laugh

You like the matrix, the photon never experiences anything because it isn't real ... it is the spoon !!!!!!

Last edited by Orac; 08/05/15 04:28 PM.

I believe in "Evil, Bad, Ungodly fantasy science and maths", so I am undoubtedly wrong to you.
Orac #54271 08/05/15 08:30 PM
Joined: Aug 2010
Posts: 3,570
B
Megastar
Offline
Megastar
B
Joined: Aug 2010
Posts: 3,570
The interesting thing about that thread is the variety of things it threw up, and the fact that “the Physicist” vanished. It is a question/answer forum, rather than a general discussion, so I think those who ask questions should have a reasonable expectation of an expert opinion, not just the thoughts of another lay-person.

Originally Posted By: Orac
You can resolve the paradox in Quantum Field Theory (QFT) because the photon is nothing but a manifestation of the fields. Time for the fields remains real and it can be used for a reference frame ... try it ... you should get an answer close to one you were given with hand waving using classic physics.


I suspect it would be a valuable exercise, but I’d need some direction if I were going to “try it”.


There never was nothing.
Bill S. #54272 08/06/15 02:06 AM
Joined: May 2011
Posts: 2,819
O
Orac Offline OP
Megastar
OP Offline
Megastar
O
Joined: May 2011
Posts: 2,819
Originally Posted By: Bill S.
I suspect it would be a valuable exercise, but I’d need some direction if I were going to “try it”.

Well lets start with "time" which in the classic physics you can conjecture if it exists for the photon or not.

QM does not give you a "no time" choice, that photons behaviour is described by a probability wave (as is everything) and that wave requires time. You do not get to conjecture if "time exists" it is implicit in the description. So when you send the photon thru a double-split experiment QM can predict what is going to happen because it is implicit time exists for the photon which describes it's probability wave ... no time ... no QM ... no probability wave ... no predicted answer.

That is why I get cross at people who say they accept QM but then talk about time being something that is "experienced" like the classic physics trash version. I had a go at Bill G a couple of times over that when he tried to pull his consensus science garbage. I don't care how many science magazines and/or sites run trash like that it's wrong unless you can overturn QM.

So if you accept QM then a photon experiences time it isn't up for conjecture. As to why GR breaks down I gave you the answer the photon or anything moving at the speed of light isn't a classic physics object and the theory isn't valid for it, hence SR/GR exclude a reference frame at the speed of light.

If you want it in science speak:
Quantum physics sees time as an intrinsic variable and not extrinsic as in classical relativistic physics.

Now in QFT you have fields everywhere and the probability waves must traverse that worldscape so time at a minimum must be everywhere the field is. You can also describe the E/B fields even when its moving at the speed of light (Quantum electrodynamics) something all classic theories fail at.

Now I can't help some people don't get classic physics is wrong and often misleading smile

An illusion can not describe itself hence my rainbow joke, what does a rainbow look like to itself?
Do you see why a photon can not classically describe itself? smile

Now you spent the time to follow the logic and evidence and you should now get the current science position on time. You can play silly games with time in classic physics but don't mistake that for thinking it is something science says is consistent with ALL THE DATA, which is a key point of science.


My complaint of the classic physics zealots is they exclude data and are no better than religious zealots excluding data they don't like. If one is a quantum zealot at least you are consistent with all the data and it's still science smile

Last edited by Orac; 08/06/15 04:13 AM.

I believe in "Evil, Bad, Ungodly fantasy science and maths", so I am undoubtedly wrong to you.
Orac #54273 08/06/15 09:09 PM
Joined: Aug 2010
Posts: 3,570
B
Megastar
Offline
Megastar
B
Joined: Aug 2010
Posts: 3,570
I think I get that, but as resident devil's advocate I'll have to give it some consideration. There must be something I can question. smile


There never was nothing.
Bill S. #54274 08/07/15 04:23 AM
Joined: May 2011
Posts: 2,819
O
Orac Offline OP
Megastar
OP Offline
Megastar
O
Joined: May 2011
Posts: 2,819
LOL no problem ... I am open to changing my mind if you can construct a good argument and highlight a breakdown in the logic.


I believe in "Evil, Bad, Ungodly fantasy science and maths", so I am undoubtedly wrong to you.
Orac #54275 08/07/15 12:15 PM
Joined: Aug 2010
Posts: 3,570
B
Megastar
Offline
Megastar
B
Joined: Aug 2010
Posts: 3,570
I did say "questions", not logical arguments. smile

I'd appreciate some maths help here, please.

A particle travelling at 0.9c from emitter to detector is timed by observer 1, who is stationary relative to both emitter and detector, as taking 1 sec to cover that distance.

Observer 2 is travelling (in the direction from detector to emitter) at 0.8c.

What time does observer 2 record for the particle?
What equation is used to determine this?

Of course you can see where this is going, but that's OK, its not a "try to catch Orac" exercise. smile


There never was nothing.
Bill S. #54276 08/08/15 12:14 PM
Joined: May 2011
Posts: 2,819
O
Orac Offline OP
Megastar
OP Offline
Megastar
O
Joined: May 2011
Posts: 2,819
Not hard to catch me out on Saturday smile

The setup on the experiment isn't very good because technically the particle is going to be observed flying at observer 2 at one speed and then moving away at another (Plays stereotypical racing car noise of approach an recede).

However ignoring you bad setup the formula for time dilation for observer 2 is

dilation time = time / sqrt(1-(v2/c2))

plugging in

dilation time = 1s / sqrt(1-((0.8*0.8)/(1*1)))

dilation time = 1s / 0.36 = 1.6667 sec

So observer 2 sees the particle take 1.6667sec emitter to detector (ignoring bad setup)

The other way of saying this is observer 2 would say the particle was doing 0.54c not 0.9c that observer 1 is seeing, so he is seeing it redshifted by quite a way.

Last edited by Orac; 08/08/15 12:17 PM.

I believe in "Evil, Bad, Ungodly fantasy science and maths", so I am undoubtedly wrong to you.
Orac #54277 08/08/15 01:02 PM
Joined: May 2011
Posts: 2,819
O
Orac Offline OP
Megastar
OP Offline
Megastar
O
Joined: May 2011
Posts: 2,819
I left the other answer but I just realized from what you said you actually wanted the calculation of the horrible setup you have.

Ok you will need to break the sitation into two the blue shift approach and the red shift recede

So lets map the distances from the emitter and get the time observer 2 passes the particle

Particle: (Simply moves away with time)
distance from emitter = 0.9c * t

Observer 2: (He starts at detector 0.9c away and reduces down)
distance from emitter = 0.9c - 0.8c * t

So they pass each other when time "t" is equal so we have
(0.9c * t) = 0.9c - (0.8c * t)

rearranging
(0.9c * t) + (0.8c * t) = 0.9c

simplifying steps
1.7c * t = 0.9c
t = 0.9c / 1.7c

t = 0.5294 seconds

So if observer 2 passes receiver at time particle leaves emitter they meet 0.5294 sec in journey

Lets just double check that

The particle will move 0.9c for 0.5294 = 0.47646c from the emitter
The observer will move 0.8c for 0.5294 = 0.42352c from detector

Add the two together and hopefully we get our 0.9c the emitter and detector are apart ... 0.47646+0.42352=0.89998 ..... woot no maths fail for me

So observer 2 will see the particle blue shifted for 0.5294 sec then it will pass him and red shift away


I believe in "Evil, Bad, Ungodly fantasy science and maths", so I am undoubtedly wrong to you.
Orac #54279 08/08/15 07:53 PM
Joined: Aug 2010
Posts: 3,570
B
Megastar
Offline
Megastar
B
Joined: Aug 2010
Posts: 3,570
Thanks Orac. Lots to think about there, not least the tendency of experts to over complicate!

What I said was:

"Observer 2 is travelling (in the direction from detector to emitter) at 0.8c."

I didn't say he was travelling between the two at any point. However, I take your point that I should have been more specific. I didn't say if observer 2 was travelling towards, or away from the set-up. My bad. frown


There never was nothing.
Orac #54280 08/10/15 05:19 PM
Joined: Aug 2010
Posts: 3,570
B
Megastar
Offline
Megastar
B
Joined: Aug 2010
Posts: 3,570
I predicted that you would see where this was going. It was nothing more serious than than to ask:

In relativity time dilation/length contraction works to the point where the speed of light is involved. It then breaks down.

The same rules of time dilation/length contraction seem also to apply to QM.

How does what happens at "c" differ between SR/GR and QM?


There never was nothing.
Orac #54281 08/10/15 06:00 PM
Joined: Aug 2010
Posts: 3,570
B
Megastar
Offline
Megastar
B
Joined: Aug 2010
Posts: 3,570
Back at the start of this thread you were looking for a “theory”. I don’t have theories; I lack the necessary maths/science to formulate one effectively. However, I’m willing to have a go at an idea. If nothing else, I can learn from having it dismantled.

Why do we not have runaway gravity?

This is the wrong question; it should be, “Do we have runaway gravity”. This has a straightforward answer: “Yes”.

Notwithstanding the fact that the Universe is expanding at an increasing rate, gravity is operating locally to cause matter to aggregate. One of the ways in which we see this happening is by black holes accreting material. True, this is a slow process, but on the timescale of the Universe processes do not need to be fast. The range of gravity is infinite, so however far apart the galaxy groups become, gravity will eventually catch up. This may be indistinguishable from the closed universe concept, but it has nothing to do with the basic geometry of the Universe. It is runaway gravity, caused by the fact that gravity creates more gravity, so gravity will eventually overcome any basic geometry.


There never was nothing.
Bill S. #54282 08/10/15 11:33 PM
Joined: May 2011
Posts: 2,819
O
Orac Offline OP
Megastar
OP Offline
Megastar
O
Joined: May 2011
Posts: 2,819
Originally Posted By: Bill S.
I don’t have theories; I lack the necessary maths/science to formulate one effectively. However, I’m willing to have a go at an idea. If nothing else, I can learn from having it dismantled.

You don't need maths/science to formulate an effective theory that is layman thinking. Einstein for example actually wasn't that good at mathematics he needed his old school teacher Hermann Minkowski to actually do the mathematics for GR. What you need is concise logical idea that holds together. I rarely ever use mathematics to "prove" anything even with experts. You should remember my exchange with PMB, he wasn't happy but I got him to show you the real problem not the problem via some mathematical abstraction. I often use mathematics to find problems but once you identify the problem it's usually easy to then work out an experiment or result that will falsify an idea.

Originally Posted By: Bill S.
Why do we not have runaway gravity?

This is the wrong question; it should be, “Do we have runaway gravity”. This has a straightforward answer: “Yes”.

Cool so now we have a working concept.

Originally Posted By: Bill S.
The range of gravity is infinite, so however far apart the galaxy groups become, gravity will eventually catch up.

You don't have a problem with this the whole range is infinite thing? Remember that means gravity has to propagate outwards as a negative attractive force like electric charge.

Is there an advantage over that formulation over doing the QFT thing and inverting it and putting a field everywhere and get rid of the infinity propagation, just a question and thought.

Originally Posted By: Bill S.
This may be indistinguishable from the closed universe concept, but it has nothing to do with the basic geometry of the Universe. It is runaway gravity, caused by the fact that gravity creates more gravity, so gravity will eventually overcome any basic geometry.

Okay I would like you to read an article from Ethan on Dark matter.
Strangely ignore the DM stuff and concentrate on understanding the normal matter behaviour.
https://medium.com/starts-with-a-bang/as...es-c5b6d90b1883

It hopefully will ring some bells.

One may also question if a big bang is possible under your idea, what would cause the matter to spread if it was already clumped in a big bag scenario with gravity as you describe?

Last edited by Orac; 08/11/15 04:12 AM.

I believe in "Evil, Bad, Ungodly fantasy science and maths", so I am undoubtedly wrong to you.
Orac #54283 08/11/15 08:31 PM
Joined: Aug 2010
Posts: 3,570
B
Megastar
Offline
Megastar
B
Joined: Aug 2010
Posts: 3,570
Originally Posted By: Orac
You don't have a problem with this the whole range is infinite thing?


What? Me! Have a problem with infinity? Seriously, though, I’m trying to use infinity as others use it.

I like Ethan’s article. So far I’ve only skimmed it, but the idea that gravity needs help in forming atoms, stars etc raises two questions for a start:

1. Is this main stream scientific thinking?

2. If so, why is it not more widely acknowledged?


There never was nothing.
Bill S. #54284 08/12/15 03:06 AM
Joined: May 2011
Posts: 2,819
O
Orac Offline OP
Megastar
OP Offline
Megastar
O
Joined: May 2011
Posts: 2,819
See you have progressed, there was a time you would take him as gospel. I take it you see more than few problems with that article.

"Is it main stream" would be a Bill G question, Ethan is part of the glossy science media so I guess it is smile

You realize you are doubting a qualified scientist in his area of expertize, Rede and Bill G will be after you smile

If you asked me as a janitor and rebel do I believe Ethan's answer then bluntly NO.

Working from first principles dark matter would clump together just not as fast or easily as matter. When normal matter is orbiting in an accretion disk the various interactions and collisions will cause the matter to heat up, and that heat will radiate energy away (the key words never mentioned). The result is the orbit decays and the process quickly escalates and that was main stream back in my day.

If dark matter doesn't interact with itself there is no mechanism to speed the accretion and hence no mechanism to to decay the orbit. That is the key difference between normal matter and dark matter and something completely lost in that whole article from Ethan.

This would present a view with dark matter that would orbit a black hole for all time, sort of like a world without friction. So actually over time you would end up with more and more dark matter trapped around black holes and the larger gravitational bodies.

Now that is actually a problem on two fronts
(i) Observationally there is a strong correlation between the rate of consumption of normal matter and the estimated mass gain of feeding black holes and little from dark matter.
(ii) Dark matter would and should end up as the dominate mass around a black hole

When you simulate it that is exactly what happens
http://www.dailygalaxy.com/my_weblog/2015/06/nasa-black-holes-are-dark-matter-concentrators.html

Observationally as Ethan says that is not what dark matter looks like in the universe, so how do we fix it. Well you create the concept that if you clump dark matter together in large enough density it self annihilates releasing positrons. So we go from absolutely dark matter to weakly interacting dark matter

Originally Posted By: http://www.dailygalaxy.com/my_weblog/2015/06/nasa-black-holes-are-dark-matter-concentrators.html

Over the past few years, theorists have turned to black holes as dark matter concentrators, where WIMPs can be forced together in a way that increases both the rate and energies of collisions.

Then if you go looking for that positron signature and you find it
http://www.americaspace.com/?p=33651

Now you are convinced aren't you smile

Last edited by Orac; 08/12/15 03:54 AM.

I believe in "Evil, Bad, Ungodly fantasy science and maths", so I am undoubtedly wrong to you.
Orac #54286 08/13/15 08:03 PM
Joined: Aug 2010
Posts: 3,570
B
Megastar
Offline
Megastar
B
Joined: Aug 2010
Posts: 3,570
Time is extra short at present, but I'm still here, just.

My main questions about Sean's article would be:

Does gravity need help to concentrate ordinary matter beyond the stage that Sean indicates?

If so, where is the evidence?


There never was nothing.
Bill S. #54287 08/14/15 12:36 AM
Joined: May 2011
Posts: 2,819
O
Orac Offline OP
Megastar
OP Offline
Megastar
O
Joined: May 2011
Posts: 2,819
I will give you the little joke on classical science zealots that don't believe in QM, that the universe actually can't exist then smile

I won't bother to write it all out again "ask a physicist" does a reasonable layman job
http://io9.com/5561717/ask-a-physicist-why-dont-collapsing-atoms-destroy-all-matter-in-the-universe

The technical name for it is degeneracy pressure.

The name doesn't translate well to me, that English thing again, but it is not up to me smile

The section on it in wiki isn't fantastic
https://en.wikipedia.org/wiki/Degenerate_matter

One of the funnier things to realize is the reverse situation in a solid.
If there is QM degeneracy pressure why doesn't a solid expand infinitely outward smile

Answer if you get stuck:
http://www.physicspages.com/2013/05/28/degeneracy-pressure-in-a-solid/

I hope you have a new appreciation for solids now. I would also hope you roll your eyes when someone says QM doesn't matter when things become large and solid, I have seen a couple of people do that on this forum.

The irony is the "classical physics world" is a delicate balance between various QM forces in remarkable fine tuning.

Our universe without QM wouldn't last very long (a few billionth of a second), yet some want it gone smile

The memo here is you can't tack QM into a classical world as a bolt on extra strange effect, you would have to completely replace it along with the classical world in a theory of everything.

Last edited by Orac; 08/14/15 04:55 AM.

I believe in "Evil, Bad, Ungodly fantasy science and maths", so I am undoubtedly wrong to you.
Orac #54288 08/14/15 01:53 AM
Joined: May 2011
Posts: 2,819
O
Orac Offline OP
Megastar
OP Offline
Megastar
O
Joined: May 2011
Posts: 2,819
David Kaplan has done a nice video on falling into a black hole but wont add to anything you already know.

https://www.quantamagazine.org/20150508-what-happens-when-you-fall-into-a-black-hole/


I believe in "Evil, Bad, Ungodly fantasy science and maths", so I am undoubtedly wrong to you.
Orac #54293 08/17/15 09:15 PM
Joined: Aug 2010
Posts: 3,570
B
Megastar
Offline
Megastar
B
Joined: Aug 2010
Posts: 3,570
https://en.wikipedia.org/wiki/Degenerate_matter

Quote:
In particular, the pressure remains nonzero even at absolute zero temperature.


Does this imply that if the pressure were zero it would, in principle, be possible to determine the position and momentum of a particle in contravention of the uncertainty principle?


There never was nothing.
Bill S. #54294 08/18/15 12:21 AM
Joined: May 2011
Posts: 2,819
O
Orac Offline OP
Megastar
OP Offline
Megastar
O
Joined: May 2011
Posts: 2,819
Yes and more importantly if you cool anything to absolute zero it would collapse, something that worried science in the 1920's with the universe temperature at 3K smile

So it is basically a statement of stability and QM still persisting even at absolute zero and hence the pressure remains and things don't collapse. QM even predicts how these things behave together as a solid and apart in so called Bose-Einstein condensates.

Some of the brighter students who haven't yet encountered QM but doing classical binding theories will often ask this question.
"Can Absolute zero temperature, cause the formation of a black hole?"
If you look carefully at what they are studying the reason for the question is obvious, and you sometimes get the next question.
"Is there such thing as Absolute hot temperature?"

Quantum physics formally sets infinitely negative to infinitely positive temperature range. The spin system in matter stops matter reaching either infinity. To beat the natural limits requires rather exotic experimental setups which has been done in the lab at least for colder than absolute zero. Such things are "toys" to prove things they could never occur naturally.

If time ever did stop you should now be able to work out what would happen to the universe or at least all the matter in it. Ponder for example people who want for time to really stop at the event horizon of a black hole, they do not really understand the implication smile

I would hope you also get why materials may expand or contract as they are cooled and you alter the balance of the opposing forces. Classical science long struggled with water expanding as it was cooled, they got that water changed into a hexagonal form but could never fully understand why. It also required a bit of hand waving (and sometimes funny wrong) as to how ice could generate massive forces to expand and explode glass containers in freezers etc. Today we know those forces can reach a mind numbing 300 Mega Pascals and what they are.

If you are interested in the classical version
https://en.wikipedia.org/wiki/Negative_thermal_expansion

Explaination in classical physics
https://en.wikipedia.org/wiki/Interatomic_potential

However note the warning at the bottom
Quote:
Classical interatomic potentials cannot reproduce all phenomena. Sometimes quantum description is necessary. Density functional theory is used to overcome this limitation.

I must say I am impressed that you are not only understanding the details but are able to extend and work out things that aren't explicitly stated. You are not jumping at individual new facts but looking careful how and if that fact fits into a framework, a sign you really are getting it.

Again my message is simple, QM is not an addition bolt on to classical physics but that classical physics is a sometimes valid simplification of a QM universe. None of this stuff is hard to understand it is only the mathematical calculations that are intense and why we still teach classic physics.

Last edited by Orac; 08/18/15 06:04 AM.

I believe in "Evil, Bad, Ungodly fantasy science and maths", so I am undoubtedly wrong to you.
Orac #54295 08/18/15 05:37 AM
Joined: May 2011
Posts: 2,819
O
Orac Offline OP
Megastar
OP Offline
Megastar
O
Joined: May 2011
Posts: 2,819
Another line you may choose to look at, so I will put in this new post.

Given what you have understood from above lets discuss an option QM throws up with black holes which rarely gets glossy media presentation

http://phys.org/news/2011-03-black-holes.html

You should sort of understand the process now that would drive the explosion. For completeness the details and mathematics is accurately done here
http://physics.stackexchange.com/questions/159937/why-would-a-black-hole-explode

So basically QM poses a possible view that a black hole can only exist so long as it can keep feeding on matter. Starved of matter it will evaporate energy until it reaches a critical size and explode.

Recycling at it's best smile

If that interests you then you can extend the theoretical construct to a different form of big bang
https://en.wikipedia.org/wiki/White_hole

I would be interested to know if you had even heard of the idea black holes might explode rather than just evaporate?

Last edited by Orac; 08/18/15 06:03 AM.

I believe in "Evil, Bad, Ungodly fantasy science and maths", so I am undoubtedly wrong to you.
Orac #54301 08/22/15 10:45 PM
Joined: Aug 2010
Posts: 3,570
B
Megastar
Offline
Megastar
B
Joined: Aug 2010
Posts: 3,570
The reason I have been absent from this thread for a while is not lack of interest.

your link: http://phys.org/news/2011-03-black-holes.html starts with a mention of Hawking radiation. In principle I know what it is, but one thing that puzzles me is that the event horizon of a BH is always presented as a zone in which conditions change slowly. Hawking radiation seems to require it to be a surface of such precise location that it can separate something as closely spaced as a particle/antiparticle pair. What am I missing?


There never was nothing.
Orac #54302 08/22/15 10:59 PM
Joined: Aug 2010
Posts: 3,570
B
Megastar
Offline
Megastar
B
Joined: Aug 2010
Posts: 3,570
https://en.wikipedia.org/wiki/White_hole

Interesting link. In a multiverse scenario, could a white hole simply be the "other side" of a BH, in another universe? Entropy would then be conserved in the multiverse, if not in individual universes.


There never was nothing.
Bill S. #54303 08/23/15 03:56 AM
Joined: May 2011
Posts: 2,819
O
Orac Offline OP
Megastar
OP Offline
Megastar
O
Joined: May 2011
Posts: 2,819
Originally Posted By: Bill S.
your link: http://phys.org/news/2011-03-black-holes.html starts with a mention of Hawking radiation. In principle I know what it is, but one thing that puzzles me is that the event horizon of a BH is always presented as a zone in which conditions change slowly. Hawking radiation seems to require it to be a surface of such precise location that it can separate something as closely spaced as a particle/antiparticle pair. What am I missing?

You are actually missing nothing and actually asking the sorts of questions that need to be asked as far as I am concerned smile

Hawkings himeself seems to have thought along the sort of thinking line with his grey holes.
http://news.discovery.com/space/no-black-holes-more-like-grey-holes-says-hawking-140124.htm

The problem to me is you create a particle/antiparticle pair those two pairs both create gravity under GR and I know you know that story. So one particle drops back in, one doesn't clearly you dissipate the gravity energy and we just did Hawking radiation with no mathematics and no concern what the interface looks like and our problem still exists.

Ok my thoughts the real problem to me everyone is dancing around is time=0 at the event horizon. That is the only reference frame available and it is strictly forbidden to use it under GR you saw that with the double slit experiment. So whatever GR is saying we can ignore exactly like the double slit situation, it fails because it is a classical theory.

What strikes me is we actually have another interface that is somewhat the same as this just a lot smaller at the atom level. What happens inside that interface is a very different to the outside and can not be described by classical rules. Classical physics sort of works down to that interface and then you have to just ignore classical physics.

I actually ponder whether the event horizon around a black hole is not just a larger version of the same thing, just ike the surface of earth is an interface and we need to adjust formula for inside (shell theorem). I would like to see a lot more justification discussion on why you can take GR inside the event horizon, it's like our crazy attempts to take classic physics inside the atom and inside the earth and you get stupidly wrong answers.

So your logic and understanding is what I call mainstream science and it goes from the atomic interface to the event horizon of a black hole. I have to stop at the event horizon as you have reached the limit of my understanding of GR and I really don't feel comfortable to go beyond it.

Last edited by Orac; 08/23/15 04:42 AM.

I believe in "Evil, Bad, Ungodly fantasy science and maths", so I am undoubtedly wrong to you.
Bill S. #54304 08/23/15 04:01 AM
Joined: May 2011
Posts: 2,819
O
Orac Offline OP
Megastar
OP Offline
Megastar
O
Joined: May 2011
Posts: 2,819
Originally Posted By: Bill S.
https://en.wikipedia.org/wiki/White_hole

Interesting link. In a multiverse scenario, could a white hole simply be the "other side" of a BH, in another universe? Entropy would then be conserved in the multiverse, if not in individual universes.

It's in the article Hawkings actually argued pretty much that

Quote:
This may imply that black holes and white holes are the same object. The Hawking radiation from an ordinary black hole is then identified with the white-hole emission.

His basis is exactly yours and you are now really showing you have pretty much understood it all.

That really was an extension exercise and I am so impressed you got it and made the connection.

See it really isn't that hard smile

Last edited by Orac; 08/23/15 04:41 AM.

I believe in "Evil, Bad, Ungodly fantasy science and maths", so I am undoubtedly wrong to you.
Orac #54315 08/25/15 03:19 AM
Joined: May 2011
Posts: 2,819
O
Orac Offline OP
Megastar
OP Offline
Megastar
O
Joined: May 2011
Posts: 2,819
Bill S ... this paper got written up by glossy science media today and I would like you to evaluate it.

So our background is as we discussed the real situation in a solid is you have two opposing QM forces both trying to do opposite things. Our glossy science media can't even give the reader that background, but you have covered it so at least will understand the context.

So here is the glossy science media article
http://phys.org/news/2015-08-iron-bar-capable-decision-making.html

Here is the actual paper
http://iopscience.iop.org/1367-2630/17/8/083023/article

Ok I would like you to evaluate both the report and original paper.

No hints this time smile

Last edited by Orac; 08/25/15 03:21 AM.

I believe in "Evil, Bad, Ungodly fantasy science and maths", so I am undoubtedly wrong to you.
Orac #54316 08/25/15 02:25 PM
Joined: Aug 2010
Posts: 3,570
B
Megastar
Offline
Megastar
B
Joined: Aug 2010
Posts: 3,570
Had a quick look at both links. Will have a better look as soon as I can. Hope you don't expect me to understand the maths in the actual paper. smile


There never was nothing.
Orac #54318 08/25/15 08:50 PM
Joined: Aug 2010
Posts: 3,570
B
Megastar
Offline
Megastar
B
Joined: Aug 2010
Posts: 3,570
Just had a look at the first link. My initial reaction was to look at the date in case it was 1st April.

The original article may show what is really going on, but Gell-Mann's "quantum flapdoodle" comes to mind.

Watch this space. laugh


There never was nothing.
Bill S. #54319 08/26/15 01:06 AM
Joined: May 2011
Posts: 2,819
O
Orac Offline OP
Megastar
OP Offline
Megastar
O
Joined: May 2011
Posts: 2,819
I had the same reaction smile

I will give you the background it comes from Quantum Information Science and I thought you might pick up the problem they were trying to understand and badly propose a solution to.

Ok this problem exists in both classical physics and quantum mechanics with solids. The question is how does a solid hold it's shape and volume given the fact it actually moves around with expansion and contraction. Having a rigid lattice structure does not solve the problem as you input or remove energy to flex the structure, that energy when restored must result in the structure returning to original position.

In solids the expansion and contraction may not be the same rate in every direction (even rate of movement is called isotropic). So as you cool the object how does it work out how much energy to return to each area. Think about cooling one end rather than the body as a whole and yet you get the same result when you let it reach equilibrium.

In classical physics you know the first order approximation as Hookes law
https://en.wikipedia.org/wiki/Hooke's_law

So one of the current interests in QM is to derive Hooke's Law on a microscopic level inside QM.

Note it is a "law" not a theory in classical physics there is no real complete explaination of why it works all you can do is a lot of hand waving and speculation. At school level you claim the energy just goes into the interatomic bonds (that is lie) wave hands and quickly move on. Teachers may need to make sure they have sent any smart students on an errand before discussing.

In QM the area is called Density Functional Theory (DFT) and as you see it's a theory not a law.
https://en.wikipedia.org/wiki/Density_functional_theory

I take it they are trying to adapt DFT into Quantum Information Theory but I am not convinced that model really works smile

Coming from a science modelling background lets just say there are models and then there are models.

Mind you, I choose to maintain my body shape by a highly efficient algorithm does sound cool laugh

Last edited by Orac; 08/26/15 02:33 AM.

I believe in "Evil, Bad, Ungodly fantasy science and maths", so I am undoubtedly wrong to you.
Orac #54341 08/27/15 03:28 PM
Joined: Aug 2010
Posts: 3,570
B
Megastar
Offline
Megastar
B
Joined: Aug 2010
Posts: 3,570
A crystal lattice forms because atoms combine to form molecules, and molecules combine to form a lattice. Without outside interference the process works faultlessly. If energy is removed, the lattice deforms. Even if the deformation is non-isotropic, the deformation in any direction is related to the bond strengths in the lattice. When the energy returns, why would the factors that originally formed the lattice not cause it to return to its original form?


There never was nothing.
Bill S. #54346 08/27/15 05:56 PM
Joined: May 2011
Posts: 2,819
O
Orac Offline OP
Megastar
OP Offline
Megastar
O
Joined: May 2011
Posts: 2,819
LOL thats the theory and it works really well if you say it fast wave hands and move on smile



https://en.wikipedia.org/wiki/Atomic_diffusion

That is why technically when you heat and cool anything over a large enough range it is never the same and why you get thermal fatigue even in the purest materials.

When you deal with complex materials you have micro crytals and this stuff gets even more mixed up and it has it's own discipline called material science.

The general problem in that is called heat treating
https://en.wikipedia.org/wiki/Heat_treating

Again what we think of as a very still lattice in classical physics isn't very still but it is a nice easy model to teach.


I believe in "Evil, Bad, Ungodly fantasy science and maths", so I am undoubtedly wrong to you.
Orac #54348 08/27/15 09:38 PM
Joined: Aug 2010
Posts: 3,570
B
Megastar
Offline
Megastar
B
Joined: Aug 2010
Posts: 3,570
I had a feeling you would say I was thinking like a geologist. smile

The only real thought I have given to lattice vibration (and phonons) is in relation to the apparent slowing of light in a medium. Obviously I need to do some more thinking.


There never was nothing.
Bill S. #54352 08/28/15 01:37 AM
Joined: May 2011
Posts: 2,819
O
Orac Offline OP
Megastar
OP Offline
Megastar
O
Joined: May 2011
Posts: 2,819
In them rocks the process would be slow probably negligible the problem really opens up in metals especially conducting and malleable ones.

So consider a pure alloy metal so there are no macroscopic or microscopic discontinuities (like crystals etc) it is a pure metal lattice.

The only real mechanism for failure then is dislocation.
https://en.wikipedia.org/wiki/Dislocation

You are here => Until the 1930s, one of the enduring challenges of materials science was to explain plasticity in microscopic terms.

Your problem is your perfect metal lattices should be much stronger than you will measure.

Quote:
As shear modulus in metals is typically within the range 20 000 to 150 000 MPa, this is difficult to reconcile with shear stresses in the range 0.5 to 10 MPa observed to produce plastic deformation in experiments.


Dislocation theory from 1934 on sorts out all the problems and it is relatively easy to understand in classical terms. Inbuilt in the classical view however is the thing just wants to stay how it is sitting there quietly on the desk with no forces at play.

In the Quantum world view it is not quite so easy to work out what is going on. How does one describe a dislocation in quantum terms?

So the magic you are after is a Quantum Theory of Dislocation Motion in Metals and now you have gone well outside my areas of expertise smile

Last edited by Orac; 08/28/15 01:44 AM.

I believe in "Evil, Bad, Ungodly fantasy science and maths", so I am undoubtedly wrong to you.
Orac #54353 08/28/15 03:27 AM
Joined: Aug 2010
Posts: 3,570
B
Megastar
Offline
Megastar
B
Joined: Aug 2010
Posts: 3,570
I recall that about 20 years ago there was work being done involving the lattice nonlinear Schrödinger equation and the Peierls-Nabarro barrier potential. Would that have anything to do with a Quantum Theory of Dislocation Motion in Metals?


There never was nothing.
Bill S. #54354 08/28/15 04:22 AM
Joined: May 2011
Posts: 2,819
O
Orac Offline OP
Megastar
OP Offline
Megastar
O
Joined: May 2011
Posts: 2,819
Yes that would be the stuff and other than recognize what they are dealing with I don't know a huge amount of material science stuff. Basically they would be attacking the orbital stability within the lattice structure in the normal QM realm.

At a guess they will model it as lots of little harmonic oscillators so you have atom oscillations and the lattice oscillations superimposed over the top. Ultimately the atoms around on the surface wont be fully balanced so you need to model them and superimpose that over the top of the internal result. What you end up with is a model of seething vibrating complex oscillations in quantum fields and you and I would see the result as a solid in classical physics. It is probably a lot worse than that and I will have missed important stuff but that would be my initial guess.

In sort of layman terms those larger resonances will have a sort of inertia effect and resist the solid changing shape the old conservation of energy again. It would really only want to change shape if it was energy effective to do so as the resonance has to collapse to a new one.

That is why you can do weird things with metamaterials as there is a pronounced composite effect as well as an atomic effect and that isn't obvious when you look at it from a classical viewpoint. Your are also getting into areas as superconductive materials etc and it's not fully complete yet.

Last edited by Orac; 08/28/15 05:11 AM.

I believe in "Evil, Bad, Ungodly fantasy science and maths", so I am undoubtedly wrong to you.
Orac #54367 08/29/15 03:05 AM
Joined: May 2011
Posts: 2,819
O
Orac Offline OP
Megastar
OP Offline
Megastar
O
Joined: May 2011
Posts: 2,819
Now this is a cool experiment I love it

http://phys.org/news/2015-08-quantum-motion.html


Last edited by Orac; 08/29/15 03:06 AM.

I believe in "Evil, Bad, Ungodly fantasy science and maths", so I am undoubtedly wrong to you.
Orac #54369 08/29/15 02:01 PM
Joined: Aug 2010
Posts: 3,570
B
Megastar
Offline
Megastar
B
Joined: Aug 2010
Posts: 3,570
Am I right in thinking that this technique reduces the “quantum jitters” in one direction such that this reduction could, in principle, be used to improve measurement, but that the overall motion remains unchanged, so the uncertainty is not diminished?


There never was nothing.
Bill S. #54374 08/30/15 03:25 AM
Joined: May 2011
Posts: 2,819
O
Orac Offline OP
Megastar
OP Offline
Megastar
O
Joined: May 2011
Posts: 2,819
Yes correct the energy in the vibrations remains unchanged so it has to increase in the unconstrained directions.

The overall uncertainty is the same, you traded more uncertainty in one direction for less in another which might help improved accuracy in the one direction if you can utilize it in an experimental setup.

Interesting week looks like we have the first crack in the standard model as well those details will be released as they clear review, so should be an interesting month.

Last edited by Orac; 08/30/15 03:49 AM.

I believe in "Evil, Bad, Ungodly fantasy science and maths", so I am undoubtedly wrong to you.
Orac #54375 08/30/15 03:42 PM
Joined: Aug 2010
Posts: 3,570
B
Megastar
Offline
Megastar
B
Joined: Aug 2010
Posts: 3,570
Originally Posted By: Orac
Interesting week looks like we have the first crack in the standard model


Is that the same lepton universality break that BaBar thought they had discovered? If so, I guess this is the first "confirmation" of their finding.

Big fun!


There never was nothing.
Bill S. #54379 08/31/15 02:50 AM
Joined: May 2011
Posts: 2,819
O
Orac Offline OP
Megastar
OP Offline
Megastar
O
Joined: May 2011
Posts: 2,819
Yes two more experiments are showing the same result, they agree with each other but not the standard model.

First one is out Sept 4th.

http://www.sciencealert.com/the-lhc-finds-evidence-of-particle-activity-beyond-the-standard-model

What most think this detail on is
https://en.wikipedia.org/wiki/Strong_CP_problem

Now if you take the propose solutions to the strong CP problem and you get something resembling what is described in Next-to-Minimal Supersymmetric Standard Model.

Yes it's a string theory model but it will tell you what to expect.
https://en.wikipedia.org/wiki/Next-to-Minimal_Supersymmetric_Standard_Model

You might want to look at it's other prediction
https://en.wikipedia.org/wiki/Neutralino

Whatever the case I think most are happy they have new physics to play with smile

Last edited by Orac; 08/31/15 03:41 AM.

I believe in "Evil, Bad, Ungodly fantasy science and maths", so I am undoubtedly wrong to you.
Orac #54393 09/03/15 03:04 AM
Joined: May 2011
Posts: 2,819
O
Orac Offline OP
Megastar
OP Offline
Megastar
O
Joined: May 2011
Posts: 2,819
Worth a read from Sabine

http://backreaction.blogspot.com.au/2015/09/loops-and-strings-and-stuff.html

She has come around to the same point and problem Lubos Motl put me in when I wanted QM and a classical theory of gravity. I can have one or the other but not both. Strangely I probably end up exactly where she is that I still don't believe the string theory story but the mathematics of it may be very useful in making predictions.

We all complain string theory can be a theory of anything but that is factually not true it CAN'T ACTUALLY make the standard model, it predicts it has to be wrong. The anti-string theory lobby have had great delight in pronouncing string theory dead because of that fact and as the standard model results initially strengthened. Now we appear to have cracks in the standard model and if it drops back to a string theory model the payback on the anti-string lobby will be large. There will be certain people like Peter Woit going for cover.

Lubos would have both Sabine and My head stuck up on a pole as a warning to other scientists but it is the case we can always be convinced by a compelling argument.

Lets see what the next results show and I will give you my thoughts.

Last edited by Orac; 09/03/15 03:13 AM.

I believe in "Evil, Bad, Ungodly fantasy science and maths", so I am undoubtedly wrong to you.
Orac #54394 09/03/15 12:46 PM
Joined: May 2011
Posts: 2,819
O
Orac Offline OP
Megastar
OP Offline
Megastar
O
Joined: May 2011
Posts: 2,819


I believe in "Evil, Bad, Ungodly fantasy science and maths", so I am undoubtedly wrong to you.
Orac #54397 09/04/15 08:45 PM
Joined: Aug 2010
Posts: 3,570
B
Megastar
Offline
Megastar
B
Joined: Aug 2010
Posts: 3,570

http://backreaction.blogspot.com.au/2015/09/loops-and-strings-and-stuff.html

Interesting read, indeed.

I guess you'll be adding "semi-shittical sentence" to your English vocabulary. smile


There never was nothing.
Orac #54443 09/15/15 05:01 PM
Joined: Aug 2010
Posts: 3,570
B
Megastar
Offline
Megastar
B
Joined: Aug 2010
Posts: 3,570
If you really have gone, Orac, I hope you are still looking at SAGG.

I want to say thanks for all the times you have made me think. True, you can be a pain in the arse at times, and some of the things you said sounded like "flapdoodle", but that has to be balanced against the positives.


There never was nothing.
Orac #54833 11/24/15 09:58 PM
Joined: Aug 2010
Posts: 3,570
B
Megastar
Offline
Megastar
B
Joined: Aug 2010
Posts: 3,570
I met this recently, and wonder if it could be an explanation as to why we don’t see run-away gravity.

Quote:
Gravitons participate in the gravitational field, which means that a graviton can emit gravitons. But remember that 4-momentum is conserved. A "spray" of gravitons from a decay collectively has the same 4-momentum which means that the gravitational field strength has not increased.


There never was nothing.
Bill S. #54839 11/25/15 03:24 AM
Joined: May 2011
Posts: 2,819
O
Orac Offline OP
Megastar
OP Offline
Megastar
O
Joined: May 2011
Posts: 2,819
Sigh, ok lets do this quickly from the top, as we haven't even discovered a graviton and it's all speculation smile

Gravitons would be heralded as particles in a field which would be effected by energy and/or mass. So given energy is a "fictional" quantity technically you get the situation where a movement of energy or mass creates a runaway effect creating more gravitons. It is the situation that a country decided to leave it's printing presses on for it's money.

The post you have given simply says that is okay because the observer will see that the new created gravitons have the same combined currency. So in your money world if a country keeps printing presses on the currency devalues relative to some other country.

So I guess you could say if I devalued the USA dollar by 50%, my dollar is worth less but the entire GDP of USA is still the same when expressed in Euros but twice as much when expressed in USA dollars. That is the same as your post statement above.

So although correct it doesn't solve the problem because the gravitons would continue the process until there was infinite of them all worth nearly nothing. The same thing happens in economics and it is called hyperinflation, if you really leave the money press on.

So lets look at the proposed process, a graviton in a small space has something happen to make another graviton pop into existance near it. That energy in the space section causes yet another graviton to pop into existence. This cycle continues infinitely and we would say the graviton theory is nonrenormalizable.

The usual way to stop the runaway process is to invoke planck distance. That two gravitons can't exist in the same planck distance and string theory for example would give you a physical reason for why that is.

The alternative answer is the process is somehow held in check and as we haven't discovered a graviton that is a bit hard to even guess at what.

All pure speculation take whichever answer you like best.

Last edited by Orac; 11/25/15 03:26 AM.

I believe in "Evil, Bad, Ungodly fantasy science and maths", so I am undoubtedly wrong to you.
Orac #54860 11/28/15 06:32 PM
Joined: Aug 2010
Posts: 3,570
B
Megastar
Offline
Megastar
B
Joined: Aug 2010
Posts: 3,570
Could there be two distinct concepts being confused here?

1. A graviton could give rise to another (albeit lesser) graviton, thus increasing the total gravitational energy.

2. A graviton might convert into a number of lesser gravitons, thus increasing the number of gravitons, but not the total gravitational energy.

If 1 is the case, how could this contribute to run-away gravity?

If 2 is the case; where does the additional energy come from?


There never was nothing.
Bill S. #54868 11/29/15 01:09 AM
Joined: May 2011
Posts: 2,819
O
Orac Offline OP
Megastar
OP Offline
Megastar
O
Joined: May 2011
Posts: 2,819
The answers would depend if gravity is a Quantum field or not

Quote:
1. A graviton could give rise to another (albeit lesser) graviton, thus increasing the total gravitational energy.

Not sure what you mean, lesser particle meaning a different particle? If you mean a graviton with a different value, in a Quantum field it's not possible as it would be quantized. For example every electron has exactly the same value there is no such thing as a lesser value electron. All particles in Quantum fields are quantized to exact identical values and as that process extends up, you build the concept of an atomic table where each atom of each element is identical except for isotopes.

Quote:
2. A graviton might convert into a number of lesser gravitons, thus increasing the number of gravitons, but not the total gravitational energy.

Really same as above, explain what you mean by lesser graviton?

In a no Quantum field situation I guess a graviton could be any size it would be controlled by whatever theory was at play. So I guess under such an idea it could vary from nothing to the size of a black hole unless the theory somehow imposed size constraints. Is that the situation you mean?

I can't answer the last bit unless I understand what you meant above.

The graviton page has been brought up todate and it would be worth a review
https://en.wikipedia.org/wiki/Graviton

The bit that might interest you is how hard gravitons are going to be to detect individually and possible problems detecting even a gravity wave.

Last edited by Orac; 11/29/15 01:35 AM.

I believe in "Evil, Bad, Ungodly fantasy science and maths", so I am undoubtedly wrong to you.
Orac #54872 11/29/15 06:40 PM
Joined: Aug 2010
Posts: 3,570
B
Megastar
Offline
Megastar
B
Joined: Aug 2010
Posts: 3,570
I expressed that badly. I was thinking along these lines:

A photon is the quantum of the EM force, so it cannot be divided, but a photon can give rise to two photons carrying less energy.

A graviton would be the quantum of gravity. Could a graviton (hypothetically) divide to become two gravitons, each carrying less energy? The quote “Gravitons participate in the gravitational field, which means that a graviton can emit gravitons”, suggests that this might be the case.

If gravity creates gravity, and gravitons participate in this, does one graviton create another? If so, the process of creating more gravity must mean that there is more gravitational energy in the system after graviton A has created graviton B than there was before. Where would the additional energy come from?


There never was nothing.
Bill S. #54880 11/30/15 03:54 AM
Joined: May 2011
Posts: 2,819
O
Orac Offline OP
Megastar
OP Offline
Megastar
O
Joined: May 2011
Posts: 2,819
Originally Posted By: Bill S.
A photon is the quantum of the EM force, so it cannot be divided, but a photon can give rise to two photons carrying less energy.

Be careful here, let me elaborate.

A photon like all elementary particles was thought not to divid at all because of feature of renomalization of a quantum filed called Ward–Takahashi identity ( Ward-Takahashi identity ), In laymans terms it's completely balanced. So for a long time it was thought it could not and would not split ... ever.

In 1970 an observation of splitting a photon was first observed by passing a laser thru a crystal lattice. The process is called Spontaneous parametric down-conversion ( SPDC description).

That work lead to the realization that while light is passing thru a medium you can play with QM statistics in the waveform and once it exits the medium when it reforms it can have new characteristics.

You may remember the photonic molecules of 2013 which in the science magazines seemed to cause excitement because someone thought they resemble the starwars lightsabre. (http://www.gizmag.com/photonic-molecules-pave-the-way-for-quantum-computers-and-lightsabers/29924/)

The reason for the caution is you need a medium in which the photons interact there is no situation they are ever known to do it in a vacuum (Ward–Takahashi holds).

Originally Posted By: Bill S.
A graviton would be the quantum of gravity. Could a graviton (hypothetically) divide to become two gravitons, each carrying less energy? The quote “Gravitons participate in the gravitational field, which means that a graviton can emit gravitons”, suggests that this might be the case.

So now when we look at this suggestion, if we are making a graviton a spin 2 quantum field particle we have to assume the same rules apply. So gravitons as per photons are never going to split in the vacuum of space but possibly could in the presence of matter or other suitable mediation layer.

Originally Posted By: Bill S.
If gravity creates gravity, and gravitons participate in this, does one graviton create another? If so, the process of creating more gravity must mean that there is more gravitational energy in the system after graviton A has created graviton B than there was before. Where would the additional energy come from?

You are looking at that, wrong the Quantum Statistics of the wave are being effected by the mediating layer. As per SPDC of a photon splitting the energy and indeed all QM statistics are preserved and that is why the two photons are actually known to be entangled.

SPDC is used often as a source of entangled photons for exactly that reason that the entanglement is guaranteed. So if a QM graviton existed you would expect the same behaviour and the split graviton pair should be entangled.

Last edited by Orac; 11/30/15 03:58 AM.

I believe in "Evil, Bad, Ungodly fantasy science and maths", so I am undoubtedly wrong to you.
Orac #54892 12/01/15 05:45 PM
Joined: Aug 2010
Posts: 3,570
B
Megastar
Offline
Megastar
B
Joined: Aug 2010
Posts: 3,570
Originally Posted By: Orac
You are looking at that, wrong


No surprise there. smile

Quote:
the Quantum Statistics of the wave are being effected by the mediating layer. As per SPDC of a photon splitting the energy and indeed all QM statistics are preserved and that is why the two photons are actually known to be entangled.


I think I follow that, but don't see the link to gravity creating more gravity.


There never was nothing.
Bill S. #54894 12/02/15 05:52 AM
Joined: May 2011
Posts: 2,819
O
Orac Offline OP
Megastar
OP Offline
Megastar
O
Joined: May 2011
Posts: 2,819
Originally Posted By: Bill S.
I think I follow that, but don't see the link to gravity creating more gravity.

It won't create more gravity because the energy would be conserved. The splitting is very controlled and not something that can lead to runaway.

Last edited by Orac; 12/02/15 06:37 AM.

I believe in "Evil, Bad, Ungodly fantasy science and maths", so I am undoubtedly wrong to you.
Orac #54896 12/02/15 04:15 PM
Joined: Aug 2010
Posts: 3,570
B
Megastar
Offline
Megastar
B
Joined: Aug 2010
Posts: 3,570
Originally Posted By: Orac
It won't create more gravity because the energy would be conserved.


That was how I interpreted it, but that seems to say that although gravity creates more gravity, gravitons play no part in it.


There never was nothing.
Bill S. #54903 12/03/15 04:31 AM
Joined: May 2011
Posts: 2,819
O
Orac Offline OP
Megastar
OP Offline
Megastar
O
Joined: May 2011
Posts: 2,819
Originally Posted By: Bill S.
That was how I interpreted it, but that seems to say that although gravity creates more gravity, gravitons play no part in it.

Thus you have proved to yourself the real problem that has stumped everyone with quantum gravity.

What is going on is with all the other fundamental particles they don't self interact. Lets break this down to a really simple body example

So when I add energy to an electric field on a body it increases the field by a proportional amount and the energy adds to gravity of the body (E=MC2).

Mathematically: Field increase = constant * Energy

You can see that in the classical formula's of force as the force is directly related to the field strength and energy
F magnetic = qB
E electric = qE

Now try it with gravity

So when I add energy to a gravity field on the body it increases the field by a proportional amount and the energy
adds to the gravity of the body ..... WOAH say what!!!!!!.

Mathematically: Field increase = constant1 * Energy + (Taylor series expansion(Field increase * constant2 * Energy)).

Background: https://en.wikipedia.org/wiki/Taylor_series

Each time you add energy to the gravity field energy a percentage is added into the field itself. That percentage
is a taylor iteration of the percentage to infinity because it self adds.

So lets look at this in classical physics
https://en.wikipedia.org/wiki/Gravitational_potential

I took some liberties and hand waving shortcuts but I got to the same point
Quote:
The integrand can be expanded as a Taylor series in Z = r/|x|, by explicit calculation of the coefficients.

I note they also give the other result I was going to handwave my way to:
Quote:
In general relativity, the gravitational potential is replaced by the metric tensor. When the gravitational field is weak and the sources are moving very slowly compared to light-speed, general relativity reduces to Newtonian gravity, and the metric tensor can be expanded in terms of the gravitational potential.

So now we have the final nightmare piece that our horrible taylor series expansion result is going to also self interact
based on its speed thru space .... mathematics nightmare.

So we have a taylor expansion (speed) of another taylor expansion (energy)

This is called an Analogue version of gravity and complex analysis of it is the only other current option besides string theory. Sabine has a basic background (http://backreaction.blogspot.com.au/2014/02/what-is-analogue-gravity-and-what-is-it.html)

I will however warn you that such a classical answer is at odds with QM. Lubos gives you a simple enough approach here (http://motls.blogspot.com.au/2012/01/why-semiclassical-gravity-isnt-self.html?)


I believe in "Evil, Bad, Ungodly fantasy science and maths", so I am undoubtedly wrong to you.
Orac #54904 12/03/15 11:46 PM
Joined: Aug 2010
Posts: 3,570
B
Megastar
Offline
Megastar
B
Joined: Aug 2010
Posts: 3,570
Orac, I haven't had time to follow those links yet, but I would like to check that I am on the right path. I never think I understand something unless I can explain to someone else; so please bear with my attempt.

I gravitons exist, this would seem to indicate that a quantum theory of gravity is, in principle, a distinct possibility in the near future. However, it is experimentally established (and experiment trumps theory) that gravity creates more gravity, which presents a couple of serious questions.

1. If gravity creates gravity, why doesn’t every gravitating body spontaneously develop into a black hole?

2. Can the introduction of gravitons to our theory of gravity provide an answer to Q1?

A graviton can “split” into gravitons of lower energy, but a graviton cannot produce another graviton of any energy if the total energy is greater after the reaction than it was before, as this would involve creating energy.

Certainly this would prevent run-away gravity, but it would also prevent quantized gravity from producing more gravity, which, as we have seen, would be contrary to physical evidence.

It follows, therefore, that if gravity creates gravity, gravitons cannot play a part in this process without violating the law of conservation of energy.


There never was nothing.
Bill S. #54906 12/04/15 03:44 PM
Joined: May 2011
Posts: 2,819
O
Orac Offline OP
Megastar
OP Offline
Megastar
O
Joined: May 2011
Posts: 2,819
Originally Posted By: Bill S.
1. If gravity creates gravity, why doesn’t every gravitating body spontaneously develop into a black hole?

Correct in fact in theory according to GR anything over 22 micrograms can collapse to a black hole. The question why it doesn't is the one I thought you might be interested in following.

You sort of have all the pieces conservation of energy, curved spacetime, and Spin of particles both classical spin and QM angular momentum to reach at least the most basic solution.

So conceptually you could imagine a spin trying to twist space with it's motion and creating torsion in space itself. There is a long list of people who looked at this being a solution to our problem above.

Einstein-Cartan theory

It is not the most favoured way currently to solve the problem but it is one easy enough for you to understand because it is entirely classical. The solution clearly isn't compatible with the full QM theory but its fine in the QM spin consideration.

Quote:
Torsion also requires fermions to be spatially extended.[11] Such particles cannot be pointlike, which avoids the formation of singularities in black holes and removes the ultraviolet divergence in quantum field theory.

Hopefully you get what happens that your matter has to have a classical spacetime volume it can't be a true point and hence no runaway collapse.

So you have reached the simplest viable process to stop our matter instantly dropping into a black hole.

In fact under the theory the smallest black hole you can create requires 1.1245 x 10E16 kg to overcome the torsional force.

It does come with other predictions listed in the link, I will leave you to mull over.

Originally Posted By: Bill S
2. Can the introduction of gravitons to our theory of gravity provide an answer to Q1?

No they are just the force carrier particle of the field they don't change the problem.

Originally Posted By: Bill S
A graviton can “split” into gravitons of lower energy, but a graviton cannot produce another graviton of any energy if the total energy is greater after the reaction than it was before, as this would involve creating energy.

Yes but the process is highly unlikely without finding a mediation scheme for a gravitional field. No such mediation is known either physically or conceptually to be viable and it would lead to a scheme of gravitational shielding.

Why we are dealing with speculation along this line you might as well look at virtual black holes by Mr Hawking.

Last edited by Orac; 12/04/15 04:26 PM.

I believe in "Evil, Bad, Ungodly fantasy science and maths", so I am undoubtedly wrong to you.
Orac #54926 12/08/15 06:19 PM
Joined: Aug 2010
Posts: 3,570
B
Megastar
Offline
Megastar
B
Joined: Aug 2010
Posts: 3,570
Originally Posted By: Orac
You sort of have all the pieces conservation of energy....


I think this is the point at which I am stuck.

We established that a graviton could not give rise to another graviton (or gravitons) if the total energy of the daughter graviton(s) was greater than that of the parent. Conservation of energy, and all that.

However, we seem to be saying that if we ignore gravitons, gravity creates more gravity.

So where does the additional energy come from, and why does the process not violate conservation of energy?


There never was nothing.
Bill S. #54931 12/09/15 09:01 AM
Joined: May 2011
Posts: 2,819
O
Orac Offline OP
Megastar
OP Offline
Megastar
O
Joined: May 2011
Posts: 2,819
Originally Posted By: Bill S.
However, we seem to be saying that if we ignore gravitons, gravity creates more gravity.

So where does the additional energy come from, and why does the process not violate conservation of energy?

The problem comes from classical physics versus GR/QM.

Try the generalized FAQ on the problem
http://math.ucr.edu/home/baez/physics/Relativity/GR/energy_gr.html

We have been thru this but energy in classical physics is poorly defined and not even consistent. In trying to drag your answers back to classical physics the answer becomes yes/no/maybe/never/I cant work it out ... choose which one applies.

Above in your quote you change from graviton energy in QM (very well defined definition of energy) to an ambiguous definition in classical physics or possibly GR it isn't clear to me which you want.

The problem is in classical physics the answer is whatever you want ... pick your own answer !!!!.

At the level you are working classical physics breaks down and there is no patch you can make for it.

Sort of along this line look at the one-minute physics video

Last edited by Orac; 12/09/15 09:11 AM.

I believe in "Evil, Bad, Ungodly fantasy science and maths", so I am undoubtedly wrong to you.
Orac #54933 12/09/15 11:29 PM
Joined: Aug 2010
Posts: 3,570
B
Megastar
Offline
Megastar
B
Joined: Aug 2010
Posts: 3,570
What you seem to be saying is that gravity creates gravity, and in classical/GR physics that is no problem because energy is poorly defined, so it doesn’t matter if energy appears to be created.

In QM, energy is well defined, so it cannot be created. This leaves a problem explaining how gravity creates gravity.

Two questions arise from this:

1. If we live in a quantum universe, how can gravitational energy be created?

2. We don’t have a quantum theory of gravity, so how do we know how QM would influence the creation of gravitational energy?


There never was nothing.
Orac #54934 12/10/15 04:16 AM
Joined: Mar 2006
Posts: 4,136
P
Megastar
Offline
Megastar
P
Joined: Mar 2006
Posts: 4,136
what a BS video.

but I love to see that you guys are really trying and
that does mean something.

so as for your discussion about gravity = energy and momentum
what happens when a atom is drained of almost all of
its energy and as a result all motion within the atom slows to a almost complete stop , are we now to believe that the atom looses a proportional amount of its gravity since gravity is no
longer due to mass but due to a combination of its energy (which has been drained ) and its momentum ( which has also been drained)

?

whats next , I wonder.

I guess that as the atom regains its (energy) heat its gravity also proportionately returns or how do you guys work around that one?

also , the energy of an atom can be bumped up so that the
orbit of the electrons of the atom become further and further
away from the center of the atom , wouldn't your brand of fizzics require the atom to acquire more gravity because its
overall energy and its overall momentum within the atom have increased.

do you guys have a workaround for that one?

if QM is better than or more correct than classical physics
then QM must also overlap into the classical world as in
the two examples above.

else it would simply be philosophical.

is QM actual or is QM philosophy.

just curious because when I see things like the video you posted a link to I become afraid and worried about the future.



3/4 inch of dust build up on the moon in 4.527 billion years,LOL and QM is fantasy science.
paul #54935 12/10/15 05:34 AM
Joined: May 2011
Posts: 2,819
O
Orac Offline OP
Megastar
OP Offline
Megastar
O
Joined: May 2011
Posts: 2,819
Originally Posted By: paul
so as for your discussion about gravity = energy and momentum what happens when a atom is drained of almost all of its energy and as a result all motion within the atom slows to a almost complete stop , are we now to believe that the atom looses a proportional amount of its gravity since gravity is no longer due to mass but due to a combination of its energy (which has been drained ) and its momentum ( which has also been drained)

Sorry Paul you make me laugh at times with your science smile

How does an atom become drained of energy, never seen that ... explain please?

Originally Posted By: paul
is QM actual or is QM philosophy.

QM is neither from a science level QM does not explain anything it simply creates a framework that predicts answers nothing more nothing less.

Science's only roll is to be useful and predict results. In that regard QM is 100% and has never yet been shown to give a wrong answer.

Unfortunately for all that it doesn't explain why things behave like they do.

On the other scale we have Classical physics which gives countless wrong answers which is why we don't use it as it isn't terrible useful beyond stuff layman may encounter.

Probably only 2 or 3% of the planet believe in QM or even understand it enough to make a judgement and yet we don't care because it is more useful than classic physics. It is the understanding behind a lot of science advancement and to be blunt that is why no-one cares what the 97% others think including the religious factions or people like yourself.

Someone always funds us because someone always wants the technical edge and that is why we don't really care what anyone else thinks. We aren't here to convert people and we care little if they believe in what we do, but they always want our developments.

Last edited by Orac; 12/10/15 05:49 AM.

I believe in "Evil, Bad, Ungodly fantasy science and maths", so I am undoubtedly wrong to you.
Bill S. #54936 12/10/15 06:20 AM
Joined: May 2011
Posts: 2,819
O
Orac Offline OP
Megastar
OP Offline
Megastar
O
Joined: May 2011
Posts: 2,819
Originally Posted By: Bill S.
What you seem to be saying is that gravity creates gravity, and in classical/GR physics that is no problem because energy is poorly defined, so it doesn’t matter if energy appears to be created.

Yes that is a fair summation

Originally Posted By: Bill S.
In QM, energy is well defined, so it cannot be created. This leaves a problem explaining how gravity creates gravity.

We went thru that under QFT it will be spin 2 boson and will self interact as well as a number of predictable properties but there are things not predictable which needs more theory. So we know why and how it creates more gravity, what we don't know is what stops runaway collapse. No theory of quantum gravity at the moment can predict or produce stability.

Originally Posted By: Bill S.
1. If we live in a quantum universe, how can gravitational energy be created?

The particles of the other forces in an area of space cause curvature of spacetime and create the presence of gravitons. If the gravitons appeared without the energy/momentum presence there would be nothing to stop a pile of them appearing in an empty space and the universe instantly collapsing.

In a really basic laymans terms it sort of like the other forces coax or squeeze the gravitons into existence which we can then measure.

Originally Posted By: Bill S.
2. We don’t have a quantum theory of gravity, so how do we know how QM would influence the creation of gravitational energy?

Ask the problem in reverse how is the gravitation force going to work of deal with a mass particle held in entanglement in two places at once.

Is the gravity at one or other position or split between the two different physical locations? How does it do this if it isn't quantum in nature itself?

You probably get why there is a lot of interest in that problem and experiments around it.


I believe in "Evil, Bad, Ungodly fantasy science and maths", so I am undoubtedly wrong to you.
Orac #54937 12/10/15 08:40 PM
Joined: Aug 2010
Posts: 3,570
B
Megastar
Offline
Megastar
B
Joined: Aug 2010
Posts: 3,570
Originally Posted By: Orac
Originally Posted By: Bill S
In QM, energy is well defined, so it cannot be created. This leaves a problem explaining how gravity creates gravity.
We went thru that under QFT it will be spin 2 boson and will self interact as well as a number of predictable properties but there are things not predictable which needs more theory. So we know why and how it creates more gravity, what we don't know is what stops runaway collapse. No theory of quantum gravity at the moment can predict or produce stability.


This probably answers the question for someone with more knowledge of QM than I have. I can’t make the link between being a spin 2 boson that will self interact, and being able to – seemingly – violate conservation of energy.

I couldn't explain that link to someone else, and unless I can do that I don't consider that I understand whatever it is.

Perhaps we could take one step at a time.

Assume our theory of gravity includes gravitons, which are the particles that mediate the gravitational force.

If this gravity gives rise to more gravity, would that not mean that graviton A must be able to give rise to graviton B, such that the energy of A+B is greater than the energy of A alone?


There never was nothing.
Orac #54938 12/10/15 08:53 PM
Joined: Mar 2006
Posts: 4,136
P
Megastar
Offline
Megastar
P
Joined: Mar 2006
Posts: 4,136
Quote:
Sorry Paul you make me laugh at times with your science smile

How does an atom become drained of energy, never seen that ... explain please?


Don't Worry, that's entirely understandable as this stuff
is found in the realm of actual science and real physics , and it just doesn't blend in well with the fantasy science and fizzix that you have your head stuck into most of the time
... heres a non layman explanation that you might be capable of comprehending and understanding at the same time in this universe.




3/4 inch of dust build up on the moon in 4.527 billion years,LOL and QM is fantasy science.
paul #54939 12/10/15 10:00 PM
Joined: Aug 2010
Posts: 3,570
B
Megastar
Offline
Megastar
B
Joined: Aug 2010
Posts: 3,570
Interesting video, Paul.

A couple of things I'm not absolutely clear about.

When an atom absorbs a photon it acquires more energy. The impact slows it down, so it loses kinetic energy. Is the kinetic energy lost greater than the energy of the acquired photon, or is something else going on?

Where does the energy go? Does the laser beam become hotter? Where does the heat go?

No doubt all these questions will be answered, and possibly someone may comment on the fact that the non-layman said the atoms are in a quantum state. Oops!


There never was nothing.
Bill S. #54941 12/11/15 02:46 AM
Joined: Mar 2006
Posts: 4,136
P
Megastar
Offline
Megastar
P
Joined: Mar 2006
Posts: 4,136
Quote:
When an atom absorbs a photon it acquires more energy.

yes , it acquires the energy of the photon.
the direction of the atom and the direction of the laser beam
determine the amount of energy over and above the energy of the photon itself that the atom acquires just like in a classic physics impact , like pool balls colliding on a pool table sort of. <--- layman description but not a guess or speculation.

Quote:
The impact slows it down, so it loses kinetic energy.


yes , the impact slows the atom down , just like in classical physics.

theres a lot more energy in a head on collision.

Quote:
Is the kinetic energy lost greater than the energy of the acquired photon,


I couldn't tell you without knowing the energy of the photon and the energy of the impact and the energy of the atom before and after the impact.

from what I understand from the experiments is that
the only time that the atom will absorb the photon is when
the atom is moving towards the laser beam and the frequencies
of the atom and the laser are exact , which is why he dwelt
on the direction in the video because direction is important
just like in classic physics.

we know that when an atom absorbs a photon the atom becomes
excited , we know that the state of the atom changes , we
know that a atoms frequency changes , and we know that the
electrons orbits move outward from the center.
in this excited state the atom has gained energy.
(heated)
then

the atom stabilizes , a photon is emitted , the state changes , the frequency changes , the electrons move closer in towards the center.
(cooled)

in order for the atom to slow down further and further ( or cool down more and more ) energy must be lost from the atom itself repeatedly and the only way possible for that to occur is if the atom emits a photon of a higher energy than the photon that impacted the atom initially.

because a normal change from an excited state to a stabilized state places the energy of the atom back to its initial energy , its frequency returns to its initial frequency and its electrons orbits return to there initial orbits.

and with this experiment we see that energy is lost
because we see the atom slowing down
and the only element involved that could possibly account
for the lost energy would be the emission of a higher charged photon.

so that is where the energy goes !

when the atom emits the higher charged photon the electrons
move closer in to the center than they were initially before
the impact.

as the atom reaches its slowest energy levels in the millionth
of a degree K its mass has not changed , it still has its electrons , they are simply much closer to the center and moving much slower in there orbits than before the experiment began.

and if my brand of physics is correct then its gravity
has also remained the same.

thus , therefore , and hithertooooo

gravity is due to mass , not energy and momentum as the
QM'ers faith would have us believe or speculate or guess or
philosophize.

Quote:
the fact that the non-layman said the atoms are in a quantum state. Oops!


probably peer pressure... possibly job security ... perhaps
fear of being labeled a heretic and shunned by the QM congregation because he seems like a truly smart smart person not a stupid smart person.

LOL










3/4 inch of dust build up on the moon in 4.527 billion years,LOL and QM is fantasy science.
paul #54942 12/11/15 05:20 AM
Joined: May 2011
Posts: 2,819
O
Orac Offline OP
Megastar
OP Offline
Megastar
O
Joined: May 2011
Posts: 2,819
ROFL I am enjoying this, I love the electron orbitals moving position that is cool except a few problems with the idea which obviously escape you laugh

The bit I really love and need a description, is what exactly does a "COMPLETELY ENERGY DRAINED ATOM" look like.

So does nothing move, the electrons sit right against the nucleus? Does it cease to exist ... can I get a description?

The other obvious question is why atomic energy interactions are quantized, why aren't they linear or logarithmic like classical stuff?

Last edited by Orac; 12/11/15 05:28 AM.

I believe in "Evil, Bad, Ungodly fantasy science and maths", so I am undoubtedly wrong to you.
Bill S. #54943 12/11/15 06:24 AM
Joined: May 2011
Posts: 2,819
O
Orac Offline OP
Megastar
OP Offline
Megastar
O
Joined: May 2011
Posts: 2,819
Originally Posted By: Bill S.
Perhaps we could take one step at a time.

Assume our theory of gravity includes gravitons, which are the particles that mediate the gravitational force.

If this gravity gives rise to more gravity, would that not mean that graviton A must be able to give rise to graviton B, such that the energy of A+B is greater than the energy of A alone

The problem is you keep trying to bring it back to ONE graviton sitting there which is by itself stable, in the same way a car sitting on a hill can be stable. This isn't the problem the next step is.

If I give the car a push then I get a problem. So lets do the same I push more energy into the fields near the graviton such that another graviton appears BUT now my two gravitons must also attract each other.

Mathematically:
X energy gives rise to the first graviton
2X energy gives rise to two gravitons as we have quantization

BUT if two gravitons also attract each other, you now go into runaway.

What we can look at is the strong force in the nucleus which also has a self interacting property and it's force carrier particles the gluon. So what stops runaway in that situation you may ask?

Well the force is opposed by the electromagnetic force repelling the protons. So our nucleus at this level is viewed by the electromagnetic force of the protons trying to push each other apart and opposed by the strong force trying to pull it together.

See the difference there is something opposing the self interaction. Compare that to gravity.

Quarks don't appear free in nature, because their force does not diminish with distance, as you separated two the force between them quickly heads to infinity. What is predicted is color confinement which prevents such a thing.

So what you are probing is a particles self energy and how that relates to energy in general (https://en.wikipedia.org/wiki/Self-energy).

For a graviton you can't do that problem in classical physics because how do we determine the gravitational field of a particle, since under Heisenberg uncertainty principle we can't know the position and velocity together.

Last edited by Orac; 12/11/15 08:25 AM.

I believe in "Evil, Bad, Ungodly fantasy science and maths", so I am undoubtedly wrong to you.
paul #54944 12/11/15 05:30 PM
Joined: Aug 2010
Posts: 3,570
B
Megastar
Offline
Megastar
B
Joined: Aug 2010
Posts: 3,570
Originally Posted By: Paul
because a normal change from an excited state to a stabilized state places the energy of the atom back to its initial energy , its frequency returns to its initial frequency and its electrons orbits return to there initial orbits.


Surely the whole point of laser cooling is that the atom does not return to its original frequency between impacts. This was explained in the video, as was the fact that it is kinetic energy that has to be reduced. The bit that was not explained (or perhaps I missed)was what happened to all the energy being introduced to the system, and the energy being removed.

Quote:
the only element involved that could possibly account
for the lost energy would be the emission of a higher charged photon.


Where would this extra-energetic photon go. Saying it is emitted by an individual atom does not account for its loss in terms of the system.


There never was nothing.
Orac #54945 12/11/15 05:35 PM
Joined: Aug 2010
Posts: 3,570
B
Megastar
Offline
Megastar
B
Joined: Aug 2010
Posts: 3,570
Originally Posted By: Orac
The problem is you keep trying to bring it back to ONE graviton sitting there which is by itself stable, in the same way a car sitting on a hill can be stable. This isn't the problem the next step is.

If I give the car a push then I get a problem. So lets do the same I push more energy into the fields near the graviton such that another graviton appears BUT now my two gravitons must also attract each other.


Now you are talking about introducing extra energy. In the gravity creates gravity situation, where does that come from?


There never was nothing.
Orac #54947 12/11/15 07:21 PM
Joined: Mar 2006
Posts: 4,136
P
Megastar
Offline
Megastar
P
Joined: Mar 2006
Posts: 4,136
Quote:
I love the electron orbitals moving position that is cool


Yeah , that is groovy , isn't it , that was my idea to have them moving around like that.


3/4 inch of dust build up on the moon in 4.527 billion years,LOL and QM is fantasy science.
Bill S. #54948 12/11/15 07:28 PM
Joined: Mar 2006
Posts: 4,136
P
Megastar
Offline
Megastar
P
Joined: Mar 2006
Posts: 4,136
Quote:
Surely the whole point of laser cooling is that the atom does not return to its original ........



Quote:
in order for the atom to slow down further and further ( or cool down more and more ) energy must be lost from the atom itself repeatedly and the only way possible for that to occur is if the atom emits a photon of a higher energy than the photon that impacted the atom initially.

because a normal change from an excited state to a stabilized state places the energy of the atom back to its initial energy , its frequency returns to its initial frequency and its electrons orbits return to there initial orbits.

and with this experiment we see that energy is lost
because we see the atom slowing down
and the only element involved that could possibly account
for the lost energy would be the emission of a higher charged photon.

so that is where the energy goes !




Quote:
Where would this extra-energetic photon go. Saying it is emitted by an individual atom does not account for its loss in terms of the system.


in the experiment it is not the system that caused the cooling
that is the focus , and as far as I can tell the photon is immediately out of focus as soon as it is emitted.

in the experiment the focus is on the slowing or cooling of the atom itself and everything outside its physical boundaries are of no concern.




3/4 inch of dust build up on the moon in 4.527 billion years,LOL and QM is fantasy science.
paul #54949 12/11/15 08:57 PM
Joined: Aug 2010
Posts: 3,570
B
Megastar
Offline
Megastar
B
Joined: Aug 2010
Posts: 3,570
Perhaps I missed something. Where in the Video did it say that the atom returned to its former frequency between hits?


There never was nothing.
Bill S. #54950 12/11/15 11:04 PM
Joined: Mar 2006
Posts: 4,136
P
Megastar
Offline
Megastar
P
Joined: Mar 2006
Posts: 4,136
that may not have been in the video I don't remember , but
that is the way that absorption and emission normally occurs.

I was just pointing that out because the atom in the video
was not following the normal standard procedure explained
by science books ( before the fantasy took hold ).

I'm not sure how the QM fantasy faries explain it to their congregation today because orac doesn't seem to have
heard of it happening that way.

I would suggest that this part of science may have been
excluded so that one or more of the fantasies could emerge
as the method of choice because they couldn't get this to
fit in the scheme of things.


3/4 inch of dust build up on the moon in 4.527 billion years,LOL and QM is fantasy science.
Bill S. #54951 12/11/15 11:18 PM
Joined: Mar 2006
Posts: 4,136
P
Megastar
Offline
Megastar
P
Joined: Mar 2006
Posts: 4,136
ok, I just watched it again and at 03:48 into the video
there is a part where he is explaining how the photon is absorbed and the atom goes to a excited state.

ok , I finished it and it didn't talk about the atom going to
ground state or stabilizing , it used to be easier to find
a video or a web page that explained absorption and emission
of a photon by an atom but since history is also written by the victor ( the BS QM congregation ) even in science it seems , I cant really find it.

the QM faith must have been planning the proposed

(gravity = energy and momentum) BS for some time now.

an observation that shows that the atoms shrink in size
due to cooling is like in
the cooling of steel atoms for instance during sub zero temperatures in the winter causes the steel to become brittle and easier to break , because the electrons in the atoms have moved slightly inwards towards the center and the molecular bond is weakened.

couldn't find anything about this on Wikipedia either
molecular bond gets redirected to the covalent bond page
where they talk about QM crap.

but youtube has it in a video.



to me the spooky part is that the web sites and the videos
don't talk about why the molecular bond becomes weak they only say that it becomes weak.

obviously theres something else in there that the QM church
doesn't want its believers / followers / worshipers to be aware of.

I wonder what that could be?












3/4 inch of dust build up on the moon in 4.527 billion years,LOL and QM is fantasy science.
Orac #54952 12/12/15 01:55 AM
Joined: Mar 2006
Posts: 4,136
P
Megastar
Offline
Megastar
P
Joined: Mar 2006
Posts: 4,136
Quote:
QM is neither from a science level QM does not explain anything it simply creates a framework that predicts answers nothing more nothing less.


so QM is a prediction service that predicts?
does it predict a single answer or a huge range of answers?
a single answer is an answer.
a huge range of answers is a guess.

Quote:

Science's only roll is to be useful and predict results. In that regard QM is 100% and has never yet been shown to give a wrong answer.


so obviously QM has never answered with a single answer
and has only guessed with a huge range of answers.
like throwing a ball and guessing where it will land.

Quote:

Unfortunately for all that it doesn't explain why things behave like they do.


you would think that QM never being wrong in its guesses
would have built up a base of data that would have the
capabilities of explaining why things behave like they do.

which tells me that they are wrong more than they are
right ... ie

if the range is from 1 to 1000 and 555 is the result
then QM was right 1 time and wrong 999 times.

Quote:

On the other scale we have Classical physics which gives countless wrong answers which is why we don't use it as it isn't terrible useful beyond stuff layman may encounter.


non laymen use classical physics to build the equipment
that QM uses to do things such as the einstein boseman condensate , without classical physics quantum physics
could do nothing except guess and never would know if 1
of the answers in the predicted range of answers is the
result.

and if quantum physics isn't fake then why does it use
fake math?

since it does nothing except predict results using
math then the use of fake math in QM proves that quantum
physics is fake and its results are faked by association.

claiming a close proximity to a observed result
as a answer is the same thing that occurs in classic
mechanics without using fake math.

when you say "Classical physics which gives countless wrong answers" I simply think about the horde of wrong answers
in the range of answers in QM.

Im going to predict that if I throw a ball into the air
the ball will land somewhere on the earth.

my predictions so far are 100% correct!

I know this because I observed everything in the cosmos
moving away from the ball as I threw it ... and I observed the ball as it lost its gravity due to its motionless in my reference frame as it appeared to stop in midair for a
moment in time before the ball gained its gravity back
and accelerated faster because its momentum was increasing
as the cosmos moved back toward the ball due to its increasing momentum and kinetic energy or its gravity.

so that explains why a falling object accelerates while
the cosmos is falling towards it.

brilliant!




3/4 inch of dust build up on the moon in 4.527 billion years,LOL and QM is fantasy science.
paul #54953 12/12/15 06:15 AM
Joined: May 2011
Posts: 2,819
O
Orac Offline OP
Megastar
OP Offline
Megastar
O
Joined: May 2011
Posts: 2,819
ROFL I think I will just ignore the crackpot religious nutter rant ... QM ate baby jesus and I hate it smile

In such long posts you would think you could get one fact right so that it didn't come across quite as stupid.

Like the dinoaurs on the arc, Paul that is so wrong it isn't worth responding too smile

Last edited by Orac; 12/12/15 07:22 AM.

I believe in "Evil, Bad, Ungodly fantasy science and maths", so I am undoubtedly wrong to you.
Bill S. #54954 12/12/15 07:02 AM
Joined: May 2011
Posts: 2,819
O
Orac Offline OP
Megastar
OP Offline
Megastar
O
Joined: May 2011
Posts: 2,819
Originally Posted By: Bill S.
Now you are talking about introducing extra energy. In the gravity creates gravity situation, where does that come from?

If you don't introduce energy nothing happens anywhere in the universe. You are a bit like the QM joke waiting for the girl to appear on your barstool smile

So lets do this again at a layman level, a flat empty section of vacuum of space is exactly that flat and empty. Virtual particles may briefly pop in and out of existence but on the whole the section remains flat and empty.

We can describe gravity as particles responding to the presence of spacetime curvature, or we can describe them as resulting from the exchange of virtual gravitons. It doesn't matter how we do it we need to introduce Energy into the section of spacetime to curve it and make them appear.

So lets slowly introduce some energy some way (say put some small energy there say an electron) enough energy to create a single graviton. Nothing exciting happens the graviton is our force carrier and the only thing it can exert a force on is thing that has our energy and so it gives mass to that thing.

The problem comes when we introduce a bit more energy and we get a second graviton. Now both our gravitons react with with whatever energy source we introduced but they will now also see each other as each graviton carries energy they react on each other. Nothing unusual there you would know it from electrons repels an electrons, protons repelling protons etc. The naive prediction would be that the two gravitons should therefore clump together creating even higher energy density at a specific point and given enough gravitons the whole thing could go into runaway with just the interaction of the gravitons creating more gravitons.

It's a horrible simplification but it will do for now. Pick the story up from there and explain your problem?


I believe in "Evil, Bad, Ungodly fantasy science and maths", so I am undoubtedly wrong to you.
Orac #54955 12/12/15 06:13 PM
Joined: Mar 2006
Posts: 4,136
P
Megastar
Offline
Megastar
P
Joined: Mar 2006
Posts: 4,136
Quote:
If you don't introduce energy nothing happens anywhere in the universe.


what about a star?

does a star require energy to be introduced before things
happen inside the star?

does any atom require energy to be introduced before things
happen inside the atom?

if the entire universe consisted of nothing more than a
single atom that atom would constantly be in motion.

Quote:
QM joke waiting for the girl to appear on your barstool


that comment sums up the entirety of QM in my opinion.

time passes .......

Quote:
So lets slowly introduce some energy some way (say put some small energy there say an electron) enough energy to create a single graviton. Nothing exciting happens the graviton is our force carrier and the only thing it can exert a force on is thing that has our energy and so it gives mass to that thing.


so a single electron stumbles in the scene , and creates
a graviton , then the graviton gives mass to the thing.


1) a single electron carries either a negative or a positive
charge.

2) does a graviton carry a charge?

3) does the graviton get its charge from the single electron
if the graviton gets its charge from the single electron?

4) does the graviton carry the same charge that the
electron carries if the graviton carries a charge?

5) does the thing carry a charge?

6) when the graviton gives the thing mass what form of mass
does the graviton give the thing.
a) virtual mass
b) mass

7) where does the graviton get the mass that it gives to the thing?

8) if gravity = energy and momentum then why are you giving
mass to the thing?

9) if mass no longer has a function in QM and is replaced
by energy and momentum why do you even mention mass at all?

10) I wonder if there is a good video to watch on youtube
or Netflix or the cartoon channel that can help me to
understand QM , one that I cant find a reason to stop
watching it in the first few minutes if I can first
get past the title.




3/4 inch of dust build up on the moon in 4.527 billion years,LOL and QM is fantasy science.
Orac #54956 12/13/15 01:53 AM
Joined: Aug 2010
Posts: 3,570
B
Megastar
Offline
Megastar
B
Joined: Aug 2010
Posts: 3,570
Originally Posted By: Orac
So lets do this again at a layman level, a flat empty section of vacuum of space is exactly that flat and empty. Virtual particles may briefly pop in and out of existence but on the whole the section remains flat and empty.


OK so far.

Quote:
We can describe gravity as particles responding to the presence of spacetime curvature,


As we started with a flat empty section of vacuum of space, would I be right in thinking that the curvature you have introduced is caused by the particles which subsequently respond to the curvature?

Quote:
or we can describe them as resulting from the exchange of virtual gravitons.


If we regard the particles as resulting from the exchange of virtual gravitons, wouldn’t the gravitational “field” have to be in the flat empty section of vacuum of space first? How do you square that with the idea that gravity is the curvature of spacetime caused by the presence of particles?


There never was nothing.
paul #54957 12/13/15 04:02 AM
Joined: May 2011
Posts: 2,819
O
Orac Offline OP
Megastar
OP Offline
Megastar
O
Joined: May 2011
Posts: 2,819
Paul as you are asking semi-intelligent questions I will respond

Originally Posted By: paul
1) a single electron carries either a negative or a positive charge.

An electron only ever carries negative charge there is an antimatter equivalent of an electron called a positron that carries positive charge. A layman would most run across those if they were ill an in hospital with a Positron emission tomography scanner or PET scanner for short. In normal matter the positive charged particle is called a proton.

Even in that basic statement there are lots of problems for your Paul science such as the existence of antimatter, why different particles for each charge smile

Originally Posted By: paul
2) does a graviton carry a charge?

It depends on what you mean by charge. In the strict electrical sense, no as charge is a very specific term of an electric field. However sometimes layman don't mean it explicitly in an electrical field and in that sense it is a form of charge. I prefer the word "statistic" for layman because it doesn't tend to lead layman into confusion as they have no precise understanding of statistic. So an electron carries an electrical statistic and a graviton would carry a gravitational statistic.

Originally Posted By: paul
3) does the graviton get its charge from the single electron if the graviton gets its charge from the single electron?

No Paul gravitons have no real relationship to electrons they are related in only that they are particles in physics. So for layman apples and oranges have the same relationship they are both fruit.

Originally Posted By: paul
4) does the graviton carry the same charge that the electron carries if the graviton carries a charge?

Repeat of above question an no they aren't directly related

Originally Posted By: paul
5) does the thing carry a charge?

Repeat of question again

Quote:
6) when the graviton gives the thing mass what form of mass does the graviton give the thing.

The gravitron doesn't really physically give a "physical thing" it provides a force that you measure as mass. Lets see a layman would see a magnet do the same thing providing a force without actually transferring anything.

The mass is provided by the force, talking about virtual mass or real mass makes no sense. I think you were thinking something solid and physical was exchanged it isn't.

Quote:
7) where does the graviton get the mass that it gives to the thing?

It doesn't its simply a force between and object and a patch of space so in that respect not really different to the old school classical version of gravity.

Quote:
8) if gravity = energy and momentum then why are you giving mass to the thing?

Anything with energy in a section of space creates gravity. The addition of momentum is done because there are things that only move at the speed of light but have a property that loosely translates as mass. That addition has nothing to do with GR/QM it was classical physics that added that trying to cover a bit more ground. Again here we don't really use those terms scientifically we use them only in trying to simply this stuff down for layman.

Quote:
9) if mass no longer has a function in QM and is replaced by energy and momentum why do you even mention mass at all?

Again you got that statement completely wrong. So correcting it, mass is created in space anywhere that energy exists in both GR and QM. It is not optional but demanded under both frameworks..

I will ignore the last stupidity but to say it is a bit rich from a guy who can fit billions of animals and dinosaurs on a boat ... now that was a good movie smile

Last edited by Orac; 12/13/15 04:04 AM.

I believe in "Evil, Bad, Ungodly fantasy science and maths", so I am undoubtedly wrong to you.
Bill S. #54958 12/13/15 04:15 AM
Joined: May 2011
Posts: 2,819
O
Orac Offline OP
Megastar
OP Offline
Megastar
O
Joined: May 2011
Posts: 2,819
Originally Posted By: Bill S.
As we started with a flat empty section of vacuum of space, would I be right in thinking that the curvature you have introduced is caused by the particles which subsequently respond to the curvature?

Yep you covered both options and we don't know which way it is. If we jump to your last question, we can add or even if it occurs at all.

Originally Posted By: Bill S.
If we regard the particles as resulting from the exchange of virtual gravitons, wouldn’t the gravitational “field” have to be in the flat empty section of vacuum of space first?

Correct and that is in line with QFT and the other fields. So here we have assumed a fully QM gravitational solution.

Originally Posted By: Bill S.
How do you square that with the idea that gravity is the curvature of spacetime caused by the presence of particles?

The extreme versions of quantum gravity and GR don't work together, they are competing descriptions of gravity. The quantum version mathematics returns the same answers as GR in the normal regions, they diverge widely in the macro and micro extremes without resolution on which is correct.

Last edited by Orac; 12/13/15 04:18 AM.

I believe in "Evil, Bad, Ungodly fantasy science and maths", so I am undoubtedly wrong to you.
Orac #54963 12/13/15 08:17 PM
Joined: Aug 2010
Posts: 3,570
B
Megastar
Offline
Megastar
B
Joined: Aug 2010
Posts: 3,570
Again, can we take one question/one answer at a time?

If we start with our flat, empty space and disregard virtual particles and anything from outside, unless/until we need it; is there a gravitational field?

My inclination is to say "no", but there is no point in following this line if I'm wrong.


There never was nothing.
paul #54966 12/13/15 11:41 PM
Joined: Dec 2010
Posts: 1,858
B
Megastar
Offline
Megastar
B
Joined: Dec 2010
Posts: 1,858
Paul, you say you think GR is a fake. Orac gives you good responses to your statements, but I wanted to respond to your overall idea. I'm not going to get into a discussion of this, I am just going to give this one response. I will give an example of GR that I think is awfully hard to shrug off.

Nuclear weapons!

The development of nuclear weapons was based purely on QM. The people who designed the bombs we used on Japan were chosen for their back ground in QM. Now if QM is fake, then nuclear weapons are fake, and the destruction of Hiroshima and Nagasaki were fakes. Hiroshima and Nagasaki were destroyed so I don't see how anybody can claim that QM is fake.

Bill Gill


C is not the speed of light in a vacuum.
C is the universal speed limit.
Bill S. #54967 12/14/15 01:40 AM
Joined: May 2011
Posts: 2,819
O
Orac Offline OP
Megastar
OP Offline
Megastar
O
Joined: May 2011
Posts: 2,819
Originally Posted By: Bill S.
My inclination is to say "no", but there is no point in following this line if I'm wrong.

There is no right or wrong answer as we don't know.

If you assume a fully QFT solution there must be a field there which is not "turned on" for the want of better words. However it is still a big question whether gravity is a quantum field.

So depending what answer you chose it changes what logically that answer dictates must be there.

Bill G, I totally agree there is no viable alternative however I expect a predictable result. Paul will make childish claims like that it somehow paid someone to say that while ignoring the blatant fact there is no actual science organization that controls science that could ever profit or even care. It's one of those spontaneous conspiracies that millions of people decided to fool the world over the same thing for no obvious reason smile

We don't need to convert people and I don't really care what he thinks, it isn't going to change science. I only have issues with Paul when he makes up lies and fabrications claiming that science says something it doesn't. He does it continually with things he doesn't like such as evolution and QM.

Last edited by Orac; 12/14/15 07:33 AM.

I believe in "Evil, Bad, Ungodly fantasy science and maths", so I am undoubtedly wrong to you.
Orac #54968 12/14/15 02:54 PM
Joined: Aug 2010
Posts: 3,570
B
Megastar
Offline
Megastar
B
Joined: Aug 2010
Posts: 3,570
Originally Posted By: Paul
Originally Posted By: Bill S
Perhaps I missed something. Where in the Video did it say that the atom returned to its former frequency between hits?
that may not have been in the video I don't remember , but
that is the way that absorption and emission normally occurs.


True; but it does say that the frequency of the laser has to be adjusted after each impact, which would not be necessary if the atom reverted spontaneously to its former state.


There never was nothing.
Bill S. #54969 12/14/15 03:57 PM
Joined: May 2011
Posts: 2,819
O
Orac Offline OP
Megastar
OP Offline
Megastar
O
Joined: May 2011
Posts: 2,819
When he gets past all that, for a real laugh let him try and come up with a way the latest cooling trick works (AKA the quantum refrigerator).

http://physicsworld.com/cws/article/news/2013/mar/12/quantum-refrigerator-is-efficient-and-reusable

I am dying to hear him explain how it works without quantum tunneling .. should be epic like dinosaurs and Noah smile

Oh keep an eye out this week Dark Matter still a no show at improved sensitivity and the big rumour that CMS and ATLAS have a new bump (http://motls.blogspot.com.au/2015/12/an-atlas-charged-higgs-boson-excess.html). Jester dropped a hint (http://resonaances.blogspot.com.au/) => So far these searches have provoked little public interest, but that may change soon.

Last edited by Orac; 12/14/15 04:09 PM.

I believe in "Evil, Bad, Ungodly fantasy science and maths", so I am undoubtedly wrong to you.
Bill S. #54970 12/14/15 04:11 PM
Joined: Mar 2006
Posts: 4,136
P
Megastar
Offline
Megastar
P
Joined: Mar 2006
Posts: 4,136
bill s

I wasnt saying that the atom in the experiment was returning
to its former ground state.

I was saying how a atom would normally return to its ground
state.

and the frequency of the laser must be adjusted to a lower frequency because the atoms frequency has lowered because
it has slowed / cooled.


3/4 inch of dust build up on the moon in 4.527 billion years,LOL and QM is fantasy science.
Orac #54971 12/14/15 04:16 PM
Joined: Aug 2010
Posts: 3,570
B
Megastar
Offline
Megastar
B
Joined: Aug 2010
Posts: 3,570
Originally Posted By: Orac
There is no right or wrong answer as we don't know.


Great. Now we have two possible tracks to follow, but first we have to be absolutely sure we are not talking about “nothing”. We have space and time and the laws of physics. Right?

A. Just flat spacetime; no fields.

B. Flat spacetime + unknown fields. You said the fields would be turned off; this must mean there is no Higgs field, because that is always turned on. Is that going to cause a problem if we introduce an electron?


There never was nothing.
Bill #54972 12/14/15 05:21 PM
Joined: Mar 2006
Posts: 4,136
P
Megastar
Offline
Megastar
P
Joined: Mar 2006
Posts: 4,136
I would like to see where you found that information
I checked the manhattan project wiki page and the word
quantum doesn't seem to be on the page.

https://en.wikipedia.org/wiki/Manhattan_Project

I also clicked on the origins link in the page.

https://en.wikipedia.org/wiki/Manhattan_Project#Origins

I then preceded to check the individual pages of each of
the people underlined below who are credited with the discovery of nuclear fission.

Quote:
The discovery of nuclear fission by German chemists Otto Hahn and Fritz Strassmann in 1938, and its theoretical explanation by Lise Meitner and Otto Frisch, made the development of an atomic bomb a theoretical possibility.


and none of the above listed people seem to have used quantum anything in the discovery of nuclear fission or making the development of a nuclear bomb a theoretical possibility
because the word quantum is also not on any of their wiki pages which tells me that they did not use quantum anything
in their work or experiments.

x otto hann
x fritz strassman
x lise meitner
x otto frisch

perhaps yourself or orac can place a link that tells just
how some type of quantum something was actually used in
the making of the atom bomb.

but the most important link would be a link that states that

Quote:
The development of nuclear weapons was based purely on QM.


from what I have already read the "purely based" part
seems to be out of place or misrepresented.

where did you read this at?

I did find this on the otto Frisch wiki page however that
clearly describes how fission was first found in an experiment

https://en.wikipedia.org/wiki/Otto_Robert_Frisch#Overview

Quote:
During the Christmas holiday in 1938 he visited his aunt Lise Meitner in Kungälv. While there she received the news that Otto Hahn and Fritz Strassmann in Berlin had discovered that the collision of a neutron with a uranium nucleus produced the element barium as one of its byproducts. Hahn, in a letter to Meitner, called this new reaction a "bursting" of the uranium nucleus. Frisch and Meitner hypothesized that the uranium nucleus had split in two, explained the process, estimated the energy released, and Frisch coined the term fission to describe it


the above really sounds like this experiment was the base of
any future results of nuclear experiments that split atoms , or cause nuclear fission , and would be the experiment and
the physics that nuclear weapons were purely based on.







3/4 inch of dust build up on the moon in 4.527 billion years,LOL and QM is fantasy science.
paul #54973 12/14/15 06:27 PM
Joined: Aug 2010
Posts: 3,570
B
Megastar
Offline
Megastar
B
Joined: Aug 2010
Posts: 3,570
Originally Posted By: Paul
...Frisch coined the term fission to describe it


Would I be right in inferring that it is not your intention to suggest that fission did not occur before the term was coined?


There never was nothing.
Bill S. #54974 12/15/15 03:27 AM
Joined: May 2011
Posts: 2,819
O
Orac Offline OP
Megastar
OP Offline
Megastar
O
Joined: May 2011
Posts: 2,819
Originally Posted By: Bill S.
B. Flat spacetime + unknown fields. You said the fields would be turned off; this must mean there is no Higgs field, because that is always turned on. Is that going to cause a problem if we introduce an electron?

LOL no the Higgs is always on in any patch of spacetime it's the one field with a constant value always as you know. So I am not sure how we got back to this "nothing" thing. There isn't a section of spacetime with a "nothing" concept, you want that go outside the universe.


I believe in "Evil, Bad, Ungodly fantasy science and maths", so I am undoubtedly wrong to you.
paul #54975 12/15/15 03:36 AM
Joined: May 2011
Posts: 2,819
O
Orac Offline OP
Megastar
OP Offline
Megastar
O
Joined: May 2011
Posts: 2,819
Originally Posted By: paul
I would like to see where you found that information
I checked the manhattan project wiki page and the word
quantum doesn't seem to be on the page.

ROFL you are so funny it hurts ... like a gift that keeps on giving laugh

Yep no QM mentioned on page and that means it wasn't involved apparently, I guess that is one up from a global science conspiracy. Now all you need to do is invent some physics that will create the energy smile

Hey I bet if you search Leo Szilard (born Leo Spitz) who owned the patents on the atomic chain reaction and in effect owns the patents on all atomic bombs, it probably doesn't mention the word QM either. His good friend Enrico Fermi who you could also search and who actually built the first nuclear reactor unfortunately you will probably find the word QM because he went on to expand the understanding and is a really important player in modern atomic science.

Nah you wouldn't bother trying to understand things your belief depends on you not understanding how this all ties together.

I did say to Bill G that you would try and invent some unbelievable stupidity. Hey I guess it doesn't involve dinosaurs and a boat so we are on the improve.

Last edited by Orac; 12/15/15 03:53 AM.

I believe in "Evil, Bad, Ungodly fantasy science and maths", so I am undoubtedly wrong to you.
Bill S. #54976 12/15/15 03:55 AM
Joined: Mar 2006
Posts: 4,136
P
Megastar
Offline
Megastar
P
Joined: Mar 2006
Posts: 4,136
I'm going to have to read up on that because before there
was the coined term "fission" what did the scientist think
was going on inside the sun...

obviously they didn't know before the term was coined
because there was no term for what they found was occurring.

and that might be interesting to look into.

so yes there was no fission before the term was coined.
they must not have known what the sun was doing all those
years !!

and I'm going to look up what term was used before the
term "fission" was coined.

ok , I looked for it but couldn't find much more than
the normal fussion stuff and the only places that I could find that recognized fission in the sun was nasa and some answers web site.

I suppose that they don't seem to think that the fission
is really important because the fussion.












3/4 inch of dust build up on the moon in 4.527 billion years,LOL and QM is fantasy science.
Orac #54978 12/15/15 04:03 AM
Joined: Mar 2006
Posts: 4,136
P
Megastar
Offline
Megastar
P
Joined: Mar 2006
Posts: 4,136
well , post a link orac , it would take much less effort
on your part because you already know where to look and
who to look for.

just post a link to some web site where some evidence of
the below can be found , you can leave out the other lip
service unless that's all that you have and you rely on that
as what you purely base your argument on.

Quote:
The development of nuclear weapons was based purely on QM.


shouldn't be too hard for you to find that.


3/4 inch of dust build up on the moon in 4.527 billion years,LOL and QM is fantasy science.
paul #54979 12/15/15 04:18 AM
Joined: May 2011
Posts: 2,819
O
Orac Offline OP
Megastar
OP Offline
Megastar
O
Joined: May 2011
Posts: 2,819
Originally Posted By: paul
I'm going to have to read up on that because before there
was the coined term "fission" what did the scientist think
was going on inside the sun...

obviously they didn't know before the term was coined
because there was no term for what they found was occurring.

Oh wow you actually cottoned onto why the atomic weapon page doesn't mention QM.

Okay lets give you the modern answer for what they were playing with which is called the strong nuclear force
https://en.wikipedia.org/wiki/Strong_interaction
Quote:
The binding energy that is partly released on the breakup of a nucleus is related to the residual strong force and is harnessed in nuclear power and fission-type nuclear weapons.

It requires the theory of quantum chromodynamics(QCD) and if QM is wrong you need a new explaination of atomic explosions.

You starting to see why we don't give a rats what all the layman believe and ignore them, and why you are giving me constant humour.

As Bill G correctly stated no QM, no atomic bomb, at least not without inventing some energy from somewhere else.

Last edited by Orac; 12/15/15 04:39 AM.

I believe in "Evil, Bad, Ungodly fantasy science and maths", so I am undoubtedly wrong to you.
Orac #54980 12/15/15 04:56 AM
Joined: Mar 2006
Posts: 4,136
P
Megastar
Offline
Megastar
P
Joined: Mar 2006
Posts: 4,136
not sure why you posted a link to that page for.

Quote:
Before the 1970s, physicists were uncertain about the binding mechanism of the atomic nucleus.


also ,

Quote:
This article needs additional citations for verification. Please help improve this article by adding citations to reliable sources. Unsourced material may be challenged and removed. (November 2015)


nov 2015 , not to long ago was it ... ?

if you want me to agree that bills comment was correct
then someone should post something verifiable.

else ... just more lip service propping up what you have
heard others say.


3/4 inch of dust build up on the moon in 4.527 billion years,LOL and QM is fantasy science.
paul #54981 12/15/15 09:14 AM
Joined: May 2011
Posts: 2,819
O
Orac Offline OP
Megastar
OP Offline
Megastar
O
Joined: May 2011
Posts: 2,819
Originally Posted By: paul
if you want me to agree that bills comment was correct then someone should post something verifiable.

else ... just more lip service propping up what you have
heard others say.

I don't need to convert you, believe whatever you want, hell you believe lots of other what I would call unbelievable stuff smile

You asked me for us to explain it and I did, you feigning understanding as a way to save the argument, not my problem. What you should have got is why you look really really silly to any scientist or uni student, when you start with I don't believe in QM. Most will switch off, walk away or treat you like an imbecile. If your religion embodies that sort of belief you better stick to marketing it to the less educated section of population.

Einstein never believed in QM either and science just doesn't care and it is one of the interesting backdrops for science as to how a genius can fail to get it. In his defense the evidence is not as overwhelming as it now is, but still most other scientists of the time did get it. So given all that do you really think science or any of us care what some religious poster on a forum called Paul thinks? What we do instead is get great amusement from you smile

Last edited by Orac; 12/15/15 09:39 AM.

I believe in "Evil, Bad, Ungodly fantasy science and maths", so I am undoubtedly wrong to you.
Orac #54982 12/15/15 02:23 PM
Joined: Dec 2010
Posts: 1,858
B
Megastar
Offline
Megastar
B
Joined: Dec 2010
Posts: 1,858
I really don't know why Paul doesn't believe in QM. I sort of understand his refusal to believe in evolution. That conflicts with his strongly held religious beliefs. But as far as I know the Bible doesn't have anything to say about physics. So there really isn't any biblical reason for him to refuse to accept QM.

Bill Gill


C is not the speed of light in a vacuum.
C is the universal speed limit.
Orac #54983 12/15/15 04:38 PM
Joined: Mar 2006
Posts: 4,136
P
Megastar
Offline
Megastar
P
Joined: Mar 2006
Posts: 4,136
1)
Quote:
I don't need to convert you, believe whatever you want


2)
Quote:
hell you believe lots of other


3)
Quote:
what I would call unbelievable stuff


4)
Quote:
you look really really silly to any scientist or uni student, when you start with I don't believe in QM.


5)
Quote:
If your religion embodies that sort of belief


6)
Quote:
Einstein never believed in QM either


the above points to QM being a belief.

its not a matter of belief for me , and you hit the nail
on the head when you wrote.

Quote:
Paul thinks


that is the exact reason why I think that QM is BS
there really is no reason to think that QM is scientific
it is a belief just like evolution , and science is not about
belief.

I asked for a link to a web site for verification of bill's
claim and neither you or bill were able to post a link , verification of a claim is important in science even if QM
doesn't like verification , same for evolution ... else science is a religion.

so who really looks silly to the readers.








3/4 inch of dust build up on the moon in 4.527 billion years,LOL and QM is fantasy science.
Bill #54984 12/15/15 05:04 PM
Joined: Mar 2006
Posts: 4,136
P
Megastar
Offline
Megastar
P
Joined: Mar 2006
Posts: 4,136
Quote:
I really don't know why Paul doesn't believe in QM


because QM uses the same fake math that Einstein used.

the math that includes the division of (c/any amount)
is strictly fake , and its intended purpose is to ensure that
all future calculations that have any relation to the speed
of light will enforce its demands that nothing can travel
faster than the speed of light.

which is also BS.

ie ... I claim that nothing can cost more than 1 dollar!!!

heres the math that proves my theory is correct ...

cost = 1 dollar / 1 dollar = 1 dollar
cost = 1 dollar / 100 dollars = 0.01 dollars
cost = 1 dollar / 1,000,000 dollars = 0.000001 dollars
cost = 1 dollar / 100,000,000,000 = 0.00000000001 dollars

the above math shows that my theory is correct and that
nothing can cost more than 1 dollar.

aint I a smart person ?

we believe know that plenty of things cost more than 1 dollar because
inflation has increased all prices.

but we also know believe that nothing can cost more than 1 dollar.

Quote:
I sort of understand his refusal to believe in evolution. That conflicts with his strongly held religious beliefs.


its not my belief in religion that tells me that evolution
is BS it is the fact that there is no proof of evolution.






3/4 inch of dust build up on the moon in 4.527 billion years,LOL and QM is fantasy science.
Orac #54985 12/15/15 07:17 PM
Joined: Aug 2010
Posts: 3,570
B
Megastar
Offline
Megastar
B
Joined: Aug 2010
Posts: 3,570
Originally Posted By: Orac
Originally Posted By: Bill S
first we have to be absolutely sure we are not talking about “nothing”.
So I am not sure how we got back to this "nothing" thing. There isn't a section of spacetime with a "nothing" concept, you want that go outside the universe.


I was trying to make sure we didn’t get back to talking about nothing.

Quote:
LOL no the Higgs is always on in any patch of spacetime it's the one field with a constant value always as you know.


Quote:
If you assume a fully QFT solution there must be a field there which is not "turned on" for the want of better words


I know the Higgs is always on, which is why I wondered how that fitted in with your saying the fields would not be “turned on”.


There never was nothing.
paul #54986 12/15/15 09:07 PM
Joined: Dec 2010
Posts: 1,858
B
Megastar
Offline
Megastar
B
Joined: Dec 2010
Posts: 1,858
Originally Posted By: Paul
cost = 1 dollar / 1 dollar = 1 dollar

Wrong!
1 dollar / 1 dollar = (1 * dollar) / (1 * dollar) = (1 / 1) * (dollar / dollar) = 1 * 1 = 1

Don't forget to include the units in the calculation, that is elementary algebra.

You had better learn a bit more math before you start running down other peoples math.

Bill Gill


C is not the speed of light in a vacuum.
C is the universal speed limit.
Bill #54989 12/16/15 02:22 AM
Joined: Mar 2006
Posts: 4,136
P
Megastar
Offline
Megastar
P
Joined: Mar 2006
Posts: 4,136
Quote:
1 dollar / 1 dollar = (1 * dollar) / (1 * dollar) = (1 / 1) * (dollar / dollar) = 1 * 1 = 1


1 what?

1 dollar?

your problem is that you left out the unit that you needed
to find in the equation which is the cost unit.
by writing the equation the correct way as below you can
see that the cost unit is important and is the reason why
the equation was used.


Quote:
cost = 1 dollar / 1 dollar = (1 * dollar) / (1 * dollar) = (1 / 1) * (dollar / dollar) = 1 * 1 = 1


but you left out the cost unit in your answer causing your
answer to be incorrect.

Don't forget to include the units in the calculation, that is elementary.

also don't forget to include the units in the answer you
provide as some may not understand what the 1 is supposed
to represent when determining the cost of something.

that's like going to buy a new car and the man says he will
sell you a new car for 20 , so you pull out a 20 dollar bill
and say sold...


suppose I go back in all the post you have made and used
math , would they all be algebra , or can you divide without
using algebra?

Quote:
cost = 1 dollar / 1 dollar = 1 dollar


what if I would have written it this way

Quote:
cost = 1 dollar / 1 = 1 dollar


then you could say 1 what , it could be a penny you know
that's why its always best to use units when possible
to avoid confusion.


cost as in how much something cost

cost = 1 dollar --- RIGHT ANSWER

not

cost = 1 ----------- WRONG ANSWER



3/4 inch of dust build up on the moon in 4.527 billion years,LOL and QM is fantasy science.
paul #54990 12/16/15 04:01 AM
Joined: Dec 2010
Posts: 1,858
B
Megastar
Offline
Megastar
B
Joined: Dec 2010
Posts: 1,858
Originally Posted By: Paul

suppose I go back in all the post you have made and used
math , would they all be algebra , or can you divide without
using algebra?

Quote:

cost = 1 dollar / 1 dollar = 1 dollar


what if I would have written it this way

Quote:

cost = 1 dollar / 1 = 1 dollar


then you could say 1 what , it could be a penny you know
that's why its always best to use units when possible
to avoid confusion.


cost as in how much something cost

cost = 1 dollar --- RIGHT ANSWER

not

cost = 1 ----------- WRONG ANSWER

And you are starting off all wrong again. The cost of something is how much you pay for it. If you are looking for the cost of an individual item then you divide what you pay for it by how many.

If you said:
cost = $1 / 1 = $1 where the 1 is the number of items.
you would be calculating the cost of one item, and the cost would be $1.

cost = $1 / $1 = $1

is meaningless. A more correct use would be for example if I had $1 and I wanted to know how many dime novels I could buy.

So I divide the amount I have by the price of 1 dime novel.
Dime novels are 1 dime per dime novel = (1 * dime) / dime novel.
That is:
Number of Dime Novels(N) = $1 / 1 dime /dime novel
To spread it out.
N = 1 * dollar / 1 * dime / dime novel

Well, a dime is 10 cents or .10 dollar.

so we have
N = 1 * dollar / .10 * dollar / dime novel = 10 dime novel

Notice that with this correct way of making a calculation I wind up with a correct answer, which is the number of novels you can buy for $1. Your way of calculating the cost is meaningless.

Bill Gill


C is not the speed of light in a vacuum.
C is the universal speed limit.
Bill #54991 12/16/15 04:17 PM
Joined: Mar 2006
Posts: 4,136
P
Megastar
Offline
Megastar
P
Joined: Mar 2006
Posts: 4,136
I think were vectoring away from the point that I was
attempting.

you are wanting to change my math that I use in my theory
that states that nothing can cost more than 1 dollar aren't you?

Quote:
(c/any amount)
is strictly fake , and its intended purpose is to ensure that
all future calculations that have any relation to the speed
of light will enforce its demands that nothing can travel
faster than the speed of light.

which is also BS.

ie ... I claim that nothing can cost more than 1 dollar!!!

heres the math that proves my theory is correct ...

cost = 1 dollar / 1 dollar = 1 dollar
cost = 1 dollar / 100 dollars = 0.01 dollars
cost = 1 dollar / 1,000,000 dollars = 0.000001 dollars
cost = 1 dollar / 100,000,000,000 = 0.00000000001 dollars

the above math shows that my theory is correct and that
nothing can cost more than 1 dollar.

aint I a smart person ?


my point is that 1 dollar divided by any number of whole dollars will always be either 1 dollar or less than 1 dollar.

because that is the way I designed my math that supports
my theory , also the answer must always be given in units of dollars.

this can also be written as follows in order to always show that nothing can ever cost a whole dollar.

cost = 1 dollar / ( 1 - any number you can dream up ) = less than 1 dollar.

the above formula will never show a cost of 1 whole dollar
it will always show a cost less than 1 dollar just like the math in Einstein's math and in QM's math.

as in
c = c / ( 1 - any number you can dream up ) = less than c







3/4 inch of dust build up on the moon in 4.527 billion years,LOL and QM is fantasy science.
Orac #54992 12/16/15 05:22 PM
Joined: Mar 2006
Posts: 4,136
P
Megastar
Offline
Megastar
P
Joined: Mar 2006
Posts: 4,136
let me try these in a sensible manner with sensible answers
to your three questions.

Quote:
1.) How does gravity get out of a black hole.


that's pretty much a stupid question but as usual coming from
a believer in QM its the norm.

answer: it was always there because of the mass that accumulated inside the center of the galaxy , it hasn't left
its there because of the mass inside the black hole.

Quote:
2.) Why is the speed of gravity restricted to the speed of light


the bounds of a gravity field is restricted only to
the mass that causes the gravity field.

Quote:
3.) What is opposing the black hole so that the black hole gravity does not go to infinity (What limits the collapse).


there is a constant collapse inside a black hole as long
as the black hole is feeding.

the gravity and pressures inside a black hole squeeze
the atoms inside it together so much that the electrons in
the atoms collapse inwards and these collapses cause energy
to be released from the atoms and this energy builds
up inside the mass until the energy can escape the gravity
of the mass.

light cannot escape the gravity because light has mass.

the excess energy escapes via gamma ray burst.



3/4 inch of dust build up on the moon in 4.527 billion years,LOL and QM is fantasy science.
Orac #54993 12/16/15 07:30 PM
Joined: Aug 2010
Posts: 3,570
B
Megastar
Offline
Megastar
B
Joined: Aug 2010
Posts: 3,570
Thread drift is a wond’rous thing,
It flits from there to here,
It obfuscates
A thread’s dictates
And makes its goals unclear.


There never was nothing.
Bill S. #54994 12/16/15 11:33 PM
Joined: Dec 2010
Posts: 1,858
B
Megastar
Offline
Megastar
B
Joined: Dec 2010
Posts: 1,858
That's why I wish the forum was still threaded. Then you could see which line came from which reply. That way you could easily tell what a reply was referring to and could keep at least one of the threads on topic.

Bill Gill


C is not the speed of light in a vacuum.
C is the universal speed limit.
Bill S. #54998 12/17/15 06:31 AM
Joined: May 2011
Posts: 2,819
O
Orac Offline OP
Megastar
OP Offline
Megastar
O
Joined: May 2011
Posts: 2,819
Originally Posted By: Bill S.
I was trying to make sure we didn’t get back to talking about nothing.

Cool then we aren't going into classic science babel which I feared.

Originally Posted By: Bill S.
I know the Higgs is always on, which is why I wondered how that fitted in with your saying the fields would not be “turned on”.

Yeah it's getting really tricky with you now because you understand it well enough to catch me out when I simplify too much. I was being lazy with the description and yeah it's wrong at heart. You got me with my loose and fast classical description of energy in the other thread as well.

You are getting to the point you are so concise, I think we will need to switch frameworks to either QM or SR with more controlled descriptors. I don't think you can go much further in classical descriptions without these sorts of problems. You basically have everything except the mathematics sorted now, so you will be able to work with the change of descriptions.

Any statement I now make with classical physics you will now punch a hole in using the advanced facts you now understand. Essentially you have got to the point you must realize the classical description is totally inconsistent.

With you I probably need refinement of the terms Energy, Matter, Measurement and if you push much further reality.

I guess we could just do these as they become a problem so I guess the first up is energy. Have you reached the point you are happy to change energy to an operator (or currency) rather than something "real"? So energy like a US dollar only has meaning and value to the person using,measuring,counting and interacting with it. You could also go down the path of a promissory note between things and the universe which is a bit more GR like (some feel it has classical physics comfort) and it doesn't really matter the frameworks all converge when we get further on.

Last edited by Orac; 12/17/15 07:23 AM.

I believe in "Evil, Bad, Ungodly fantasy science and maths", so I am undoubtedly wrong to you.
Orac #55004 12/19/15 11:27 PM
Joined: Aug 2010
Posts: 3,570
B
Megastar
Offline
Megastar
B
Joined: Aug 2010
Posts: 3,570
Originally Posted By: Orac
Have you reached the point you are happy to change energy to an operator (or currency) rather than something "real"?


I thought I had reached that point until I looked up "operator". One look at the maths, and I was not so sure. smile


There never was nothing.
Page 1 of 10 1 2 3 9 10

Link Copied to Clipboard
Newest Members
debbieevans, bkhj, jackk, Johnmattison, RacerGT
865 Registered Users
Sponsor

Science a GoGo's Home Page | Terms of Use | Privacy Policy | Contact UsokÂþ»­¾W
Features | News | Books | Physics | Space | Climate Change | Health | Technology | Natural World

Copyright © 1998 - 2016 Science a GoGo and its licensors. All rights reserved.

Powered by UBB.threads™ PHP Forum Software 7.7.5